Vous êtes sur la page 1sur 154

TABLE OF CONTENTS

To my mother lluminada,
my ro,ife Imelda,
anil our Children Kim D eunice,
KenDainiet,
and KarlaDenise

CHAPTER 2
Principles of Hydrostatics
Unit Pressure ..............;
............27
Absolute and Gage Prcssures.. ............29
Variations in Pressure ............ .........31
Pressure below I .rr urs of I)rtlerent LiryurJs...... .................32
Pressure Hea.l. ............
Manonr.,tcrs ........ .......... 3+
SOLVED PR(}TIT-LMS
SUPPI t:MtNTARY I'I{UBLEMS ...........69 to72
lt TABLE OF CONTENTS TABLE OF CONTENTS iii

CHAPTER 3 CHAPTER 5
Total Hydrostatic Force on Surfaces................ .......73 Fundamentals of Fluid Flow
Total Hydrostatic Force on Plane Surface ................73 Discharge. ...........247
Properties of Common Geometric Shapes... ........76 Definition of Terms... ...........241
Total Hydrostatic Force on Curved Surface,..... .......78 Energy and Head .................244
Power and Efficiency..........., ..................245
Dams ..................... 81 Bernoulliis Energy Theorem... ...............246
Types of Dams ...................81 Energy and Hydraulic Grade Lines .....248
Analysis of Gravity Dams........ ........... 84 SOLVED PRO8LEMS................ 250to 273
Buoyancy. ............. SS SUPPLEMENTARY PROBLEMS .................................. 27 4 to 27 6
Archimedes' Principles ................. ......88
Statical Stability of Floating Bodies ........90
Stress on Thin-Walled Pressure Vessels ................,..96
Cylindrical Tank.......... ......96
Spherical Shell ......... ............:................98
Wood Stave Pipes......................:..... .....98
SOLVED PROBLEMS.................
SUPPLEMENTARY PROBLEMS .................................. 19 6 to 200
iv TABLE OF CONTENTS TABLE OF CONTENTS V

:
I
i
\
I

{
.t
I TABLE OF CONTENTS FLUID MECHANICS CHAPTER ONE
Properties of Fluids
VI & HYDRAULICS

CHAPTER 9
I lytlroclynamics........'........'
Ij,rrcc against Fixed Flat Plates
"""""""""'
"""""""
551
551
Chapter I
Irorcc a[ainst Fixed Curved Vanes """' 553
Properties of Flui ds
I

Ijorce a[ainst Moving Vanes ..'......""":"' """"""""' 554


Work Done on Moving Vanes """"" 555
Force Developed on Closed Conduit "' 556 FLUID MECHANICS & HYDRAULICS
Drag and Lifi............. """"""557 Fltid Mechanlcs is a physical science dealing with the action of fluids at rest or
Terminal Velocity.... """" 559 in motion, and with applications and devices in engineering using fluids.
Water Hamrner... """"""""" 560 Fluid mechanics can be subdivided into two m4jor areas, Jluid stntics, which
563 to597 deals with fluids at rest, and fluid dynamics, concerned with fluids in motion.
SOLVED PROBLEMS""""""""
The term hydrodynanrics is applied to the flow of liquids or to low-velocity gas
SUPPLEMENTARY PROBLEMS .'.. " " " " " " " " " " " " " " " 5 97 to 598 flows where the gas can be considered as being essentially incompressible.

Hydratilics deals with the application of fluid mechanics to engineering devices


involving liquids, usually water or oil. Hydraulics deals with such problems
APPENDIX as the,flow of fluids through pipes or in open channels, the design of storage
Properties of Fluids on,t (-nnrrprqin'n
l- and Conversion Factors ""!"""";'.r' .......8g9 dams, pumps, and water furbines, and with other devices for the control or
Table A - 1: Viscosity and Density of Water at l atm""" """"S99
use of liquids, iuch as nozzles, valves, jets, and flowmeters.

Table A - 2:Viscosity and Density of Air at 1 atm"""""""""'


600

TableA_3:PropertiesofCommonLiquidsat].atm&20.C..601
Table A - 4: I'roperties of Comrnon Gases at 1 atm & 20'C ""' 601 TYPES OF FTUID
Table A - 5: Surface Tensiory Vapor Pressure, Fluids are generally divided into two categories: ideal fluids and real fluids.
ancl sound SPeed of Water ""'602
Table A - 6: Properties of Standard Atmosphere """"""""""' 603 Ideal Jluids
. Assumed to have no viscosity (arr.l lrence, no resistance to shear;
Table A - 7: Coriversion Factors from BG to SI Units """"""" 604 . Incompressible
Table A - 8: Other Conversion Factors """""""""' 605 . Have uniform velocity when flowing
. No friction between moving layers oi fluid
. No eddy currents or turbulence

INDEX I - IV ReaI fluids


. Exhibit infinite viscosities
. Non-uniform velocity distribution when flowing
. ComPressible
. Experience friction and turbulence in flow
CHAPTER ONE FLUID MECHANICS CHAPTER ONE
2 Properties of Fluids
FLUID MECHANICS
& HYDRAULICS & HYDRAULICS Properties of Fluids 3
[{t'al fltrids arc fttrther divided into Newtonian
fluids and non-Nerutonian fhrirls. where: p = absolute pressure of gas in Pa
R = gas constant Joule / kg-"K
Most l'luid problems assume real fluids with Newtonian characteristics for For air:
<'ortvenierrce. This assumption is appropriate for water, air, gases, steam, and R= 287 l/kg- "K
.thcr simple fluids like alcohol, gasoline, acid solutions, etc. However, R ='1,7L6 lb-ft/siug-sP
oKelvin
slurries, pastes, gels,' suspensions may not behave according to simpre fluid T: absolute temperature in
rcltrtionships. "K: "C + 273
"R="F+460

Table 1 - 1r Approximate Room-Temperature


Densities of Common Fluids

Fluid p in kg/mr
't.29
Air (STP)
Air (21'F, a 1tm) 1..20
Alcohol 790
Ammonia 602
Gasoline 720
Pseudoplastic Fluids Fluids Glvcerin 1.,260
Mercury 13,600
Water 1,000
Figure 1 - 1: Types offluid

MASS DENSITY, p (RHO) SPECTFTC VOLUME, Vs


The density of a fluid is its mass per unit of volume. Specific volume, %, is the volume occupied by a unit mass of fluid

massof fluid, M
Eq. 1- 1.
volume, V

Units:
English : slugs/ft3 Note: psrug, = ptbnf g
Metric : gram/cm3 UNIT WEIGHT OR SPECIFIC WEIGHT, Y
SI: k8lms Specific weight or unit weight, y, is the weight of a unit volume of a fluid.

lior an ideal gas, its density can be found from the specific gas constant and weightof fluid, W
Eq.1, - 4
icleal gas law:
volume,V'

v:Pg 1-5
o=
'RT
P
Eq.1. -2
CHAPTER ONE FLUID MECHANICS FLUID MECHANICS CHAPTER ONE
4 Properties of Fluids & HYDRAULICS & HYDRAULICS Properties of Fluids 5
U rrits: the upper plate will adhere to it and will move with the same velocity U while
Iinglish : lb/ ftt the fluid in contact with the fixed plaie will have a zero velocity. For small
Metric : dyne/cm3 values of U and y, the velocity gradient can be assumed to be a straight line
SI : N/m3 or kN/m3 and F varies as A, U and y as:

^AUFU
loc
-
yAy
SPECIFIC GRAVITY L) dv
but (from the figure)
Specific gravlty, s, is a dimensionless ratio of a fluid's density to some dy
standard reference density. For liquids and solids, the reference density is
water at 4' C (39.2' F). 5A : Sheuri.tg stress, r
T oc
dv Or T =k-.dv
-dv
Pliuui.t dv
Eq.1-6
P water where the constant of proportionality k is called the dynamic of
absolute viscosity denoted as p.
In gases, the standard reference to calculate the specific gravity is the density dv
of air. 'dy

. __.::_
Pgas
1-8
s = Eq. "I _ 7 t-l=--:--.---
' dv/dy Eq.
P air

For water at 4oC: where:


y = 62.41b / tF = 9.81 kN/mr r : shear stress in lb/ftz or Pa
p = 1.94 slugs/ft3 = 1000 kg/mt p = absolute viscosity in lb sec/ft2 (poises) or Pa-sec.
s=1.0 y = distance between the plates in ft or m
U: velocity in ft/s or m/s
VISCOSIW, p (MU)
The property of a fluid which determines the amount of its resistance to KINEMATIC VISCOSITY v (NU)
shearing forces. A perfect fluid would have no viscosity. Kinematic viscosity is the ratio of the dynamic viscosity of the fluid, p, to its
tnass density, p.
Consider two large, parallel Area = A U
plates at a small distance y
apart, the space between
them being filled with a fluid.
Consider the upper plate to
be subject to a force F so as to where:
move with a constant velocity Lr = absolute viscosity in Pa-sec.

U. The fluid in contact with p = density in kg/m3


fixed plate
CHAPTER ONE FLUID MECHANICS FLUID MECHANICS CHAPTER ONE
6 Properties of Fluids & HVDRAULICS & HYDRAULICS Properties of Fluids 7
Table 1 - 2: Common Units of Viscosity Capillarity
Svstem Absolute, p Kinematic, v
lb-sec/ ftz
English (slus/ft-sec) fA/ sec

dyne-s/cmz cm2/s
Metric (ooise) (stoke)
Pa-s
s.l mz/s
(N-s/rnz)

Note:
1 poise = 1 dyne.s/cm2 = 0.1 Pa-sec (1 dyne = 10-5 N)
1 stoke = 0.0001 m2ls

(a) Adhesion > cohesion (b) Cohesion > adhesion

SURFACE TENSION o (SIGMA)


Capillarity (Capillary nction) is the name given to the behavior of the liquid in a
The membrane of "skin" that seems to form on the free surface of a fluid is thin-bore tube. The rise or fall or a fluid in a capillary tube is caused by
due to the intermolecular cohesive forces, and is known as surface tension. surface tension and depends on the relative magnitr"rdes of the cohesion of the
Surface tension is the reason that insects are able to sit on water and a needle is liquid anct the adhesion of the liquid to the walls of the containing vessel.
able to float on it. Surface tension also causes bubbles and droplets to take on Liquids rise in tubes they wet (adhesion > cohesion) and fall in tubes they do
a spherical shape, since any other shape would have more surface area per not wet (cohesion > adhesion). Capillary is important when using tubes
unit volume. smaller than about 3/8 inch (9.5 mm) in diameter.

Pressure inside a Droplet of Liquid:

'4o '
P=T Eq.1 - 10 For complete wetting, as with water on clean glass, the angle e is 0o. Hence
the formula becomes
where:
o = surface tension in N/m
d = diameter of the droplet in m
p= gage pressure in Pa
where:
/l = capillary rise or depression in m
y : unit weight in N/m3
d = diameter of the tube in m
o = surface tension in Pa
I CHAPTER ONE
Properties of Fluids
FLUID MECHANICS
& HYDRAULICS
FLUID MECHANICS
& HYDRAULICS
CHAPTER ONE
Properties of Fluids I
Table I - 3: Contact Angles, 0 _ stress =
rLs=--------- _ Ap D^ 1 1r
1-15
,- Eq.
Materials Angle,0 strain LV
mercury-glass 140' v
water-paraffin 107"
dn
water-silver 90' orEs=-..'.--
- dv/v Eq. 1-16
kerosene-glass 26"
slvcerin-elass 19"
water-glass 00

ethvl alcohol-glass 00
PRESSURE DISTURBANCES
Pressure disturbances imposed on a fluid move in waves. The velocity or
celerity of pressure wave (also known as acoustical or sonic aelocity) is
COMPRESSIBILIW, expressed as:
B
of compressibillty) is the fractional
C' ompressibilify (also known as the cofficient

change in the volume of a fluid per unit change in pressure in a constant- tr; tr
c=.1 " = l-
p Eq."I_17
temperature process. I lFp
AV
-\/Y 1
'LPEB =j-
B= Eq.1-13
PROPERTY CHANGES IN IDEAL GAS
For any ideal gas experiencing any process, the equation of state is given by:
dv /v
^- a -
orP:-----;- Eq.1, - 1.4
qp
PrVt - PzVz Eq.1-18
Tt Tz
where:
AV = change in volume When temperature is held constant, Eq. 1 - l8 reduces to (Botlle's l.arc)
V = original volume
Ap = change in Pressure ptVt=pzVz Eq.1-19
dV /V = change in volume (usually in percent)

When temperature is held constant (isothermal condition), Eq. 1 - 18 reduces


lo (Clurle's Lato)

BULK MODULUS OF ELASTICITY' E8 vt : v, Eo.L -20


The bulk moclulus ot t,lasticity r,f the lluicl expresses the compressibility
of the T1 Tz
fluid. It is the ratio ol the change in urrit pressure to the corresponding
volume change Per unit ot vt'lume'
q{

lli

to CHAPTER ONE
Properties of Fluids
FLUID MECHANICS
& HYDRAULICS
FTUID MECHANICS
& HYDRAULICS
CHAPTER ONE
Properties of Fluids tl
li
For Adiabatic or Isentropic Conditions (no heat exchanged) Table 1 - 4r Typical Vapor Pressures
i
pt Vtk = pz Vzk 8q.1, - 21 Fluid kPa,20"C
mercury 0.000173
or Pt\u = u = Constant Eq.1, - 22
turoentine 0.0534
\vz ) Pt water 2.34
ethvl alcohol 5.86
k-^L
ether 58.9
t2 lPzlk
and
r, - l.p,l Eq.1-23 butane
Freon-12
278
584
DroDane 855
where: ammonla 888
pr = initial absolute pressure of gas
pz= final absolute pressure of gas
Vi = initial volume of gas
Vz = final volume of gas
Tr = initial absolute temperature of gas in .K (.K =.C + 273)
Solved Problems
Tz = final absolute ternperafure of gas in .K
lc = ratio of the specific heat at constant pressure to the specific heat at Problem 1- 1
constant volume. Also known as adiabatic exponent. A reservoir of glycerin has a mass of '1,200 kg and a volume of 0.952 cu. m.
liincl its (a) weight, W, (b\ untt weight, y, (c) mass density, p, and (d) specific
gravity (s).

VAPOR PRESSURE Solution


Molecular activity in a liquid will allow some of the molecules to escape the (a) Weight *-_** t
liquid surface. Molecules of the vapor also condense back into the liquid. The (1,200)(9.81)
vaporizalion and condensation at constant temperature are equilibrium Weight, W=77,772 N or 11.772 kN
processes. The equilibrium pressure exerted by these free molecules is known
as the papor pressure or saturation pressure.
(1,) Unit weight, , =
some liquids, such as propane, butane, ammonia, and Freon, have significant
+
_ 77.772
vapor pressure at normal temperatures. Liquids near their boiling point or
0.952
that vaporizes easily are said to aolatile liquids. other liquids such as mercury,
Unit weight, y:72.366 kN/m3
have insignificant vapor pressures at the same temperature. Liquids with low
vapor pressure are used in accurate barometers.

The tendency toward vaporization is dependent on the temperature of the (,') Density, p= M
liquid. Boiling occurs when the liquid temperature is increased to the point v
12oo
that the vapor pressure is equal to the local ambient (surrounding) pressure. DensitY, P =
Thus, a liquid's boiling temperature depends on the local ambient pressure, as 0.952
well as the liquicl's tendency to vaporize. Density, p = 1,,260.5 kglm3
CHAPTER ONE
t2 Properties of Fluids
FLUID MECHANICS
& HYDRAUTICS
FLUID MECHANICS
& HYDRAULICS
CHAPTER ONE
Properties of Fluids r3
(d) Specific gravity, s =
Pglv Solution
Pwater (a) W=*g=22(9.75)
. 1.260.5
Spectrlc gravrty, s = -:-
W= 214.5 N
' 1,000
Specific gravity, s ='L.26 (b) Since the mass of an object is absolute, its mass will still be 22kg

Problem 1- 2 Probtem 1- 5
The specific gravity of certain oil is 0.82. Calculate its (a) specific weight in What is the weight of a 45-kg boulder if it is brought to a place where the
lblft3 and kN/m3, and (b) mass density in slugs/ft3 and kglm'. acceleration due to gravity is 395 m/s per minute?

Solution Solution
(a) Specific weight, y : ywater x s W= Mg
Specific weight, y = 62.4 x 0.82 = 51.168 tb/ft3 m/s r lmin
Specific weight, y = 9.81 x 0.82 = 8.044 kN/mr 'c=3g5 min 60sec
g = 6.583 m/s2
(b) Density, p = p*ot.,.x s
Density, p = 1.94x 0.82 = 1.59 slugsftl
w= 4s(6.s83)
W=296.25N
Density, p = 1000 x 0.82 = 820 kglm3

Problem l - 5
Problem 1- 3
A liter of water weighs about 9.75 N. Compute its mass in kilograms.
If the specific volume of a certain gas is 0.7848 m3/kg, what is its specific
weight?
Solution
Solution
W
Mur, = Vr=
1
-
s
p
4.71
Mass = '_-'i" 11
9.8L
,
' v, 0.7848
Mass = 0.994 kg
p = 1.2742k9/mu

specific weight,,
Problem 1 - 4 ]=l.irtn *n.r.,
If an object
has a mass of 22 kg at sea level, (a) what will be its weight at a Specific weight y = 12.5 N/m3
point where the acceleration due to gravity g = 9.75 m/ sz? (b) What will be its
mass at that point?
l4 CHAPTER ONE
Froperties of Fluid5
FLUID MECHANICS
a nvonAuucs
Fr.urD MEcHnrrircs
& HVDRAULICS
CHAPTER ONE
Properties of Fluids
l5
Problem 1- 7 $olution
l,Vhat is the specific weight of air at 480 kPa absolute and 21'C?
Density, p = I
1 : {
I
Solution - 13.7
g;81,
T=q"8
p = 1.397 kglxrf
p
= 4=
'RT where R=287J/kg:K
t)
l Densitv; o = '
480xL03 , , : RT
287(21.+273) (205+L0'l'.325)x 103
'J,.397 Note: P.tm = 101.325 kPa
=
p= 5.689kg '

-RAr.r?3)
Gas constant, R = 778.87 1441. "K
5.689 x 9.8L
y = 55.81 N/rn3 ,

1. 10
io kept at:a pressure of 200 iiPa absolute and a temperatirre of 30bC in
a

Problem 1- 8 Jiter container. What is the mass of air?


Find the mass density of helium at a temperature of 4 "C and a pressure of 184
kPa gage, if atmospheric pressure is 10'1'92 kPa. (R = 2A79 J /kg :, 'K)
Solution o=
.RTP
n
Densitv. - -I- 200 x 103
r
_-_r. 6
RT 287(30 + 273)
p=p ag"+p*^ rp*2.3kg/m3
184+ 101.92
?=285.92kPa ,'ltdass= pxV'
9
,1..'
T= 4+ 273= 277"K '', ,=2.3xffi
It,hfass = 1.L5 kg
. 285.92x103
- J o=
Densrtv.
" 2,079(277)
Density, p = 0. 4g65kg6.f'
t-11
tank 80 cm in diameter and 90 cm high is filled with a liquid
- and the liquid weighed 420 kg. ,The weight of thb empty tank is 4(
Problem 1- 9
io the unit weight of:the liquid in kN/m3
and 205 kPa gage, the specific weight of a certain gas,was 13.7 N/mr.
.)t.32"C
I )etermine the gas constant of this gas.
1t
t6 CHAPTER ONE
Properties of Fluids
FLUID MECHANICS
& HYDRAULICS
FLUID MECHANICS
& HYDRAULICS
CHAPTER ONE
Properties of Fluids l7
Solution Solution
M dP
o: v Es=
dv
dp=pz-pt
/v
420- 40
= 840 kg/m:
f (0.8)'(0.e0) h=o
y=pg dp=p,
= 840(9.81) = 8240.4 N/m3 dV = Vz- Vt
y = 8.24 kN/ms dV=-0.6%V=-0.006V

F - Pz
Problem L - Lz " 0.006v /v
-.^

A lead cube has a total mass of 80 kg. what is the length of its side? sp. gr. of pz= 0.0132GPa
lead =-11.3. pz=13.2MPa

Solution
Let L be the length of side of the cube: Problem 1- 15
Water in a hydraulic press, initially at 137 kPa absolute, is subjected to a
M= pV pressure of 176,280 kPa absolute. Using Ee = 2.5 GPa, determine the
33 = (1000 x 11.3) Ls
percentage decrease in the volume of water.
L= 0.192 m = 19.2 cm
Solution
r- dP
Problem 1- 13 - dv/v
A liquid compressed in a container has a volume of 1 liter at a pressure of 1
MPa and a volume of 0.995 liter at a pressure of 2 MPa. The bulk modulus of (116,280-137)x103
2.5x10e=-
elasticity (Er) of the liquid is: dv /v
dV
Solution = -0.0465
v
-- dP 2-1 dV
= 4.65"ludecrease
dv/v (0.ee5-7)/1, v
Ea = 200 MPa

Prcblem 1- 16
Problem ! - 14 lf 9 m3 of an ideal gas at24 oC and 150 kPaa is compressed to 2 ma, (n) what is
\Alhat pressure is required to reduce the volume of water by 0.g percent? Bulk the resulting pressure assuming isothermal conditions. (b) What would have
modulus of elasticity of water, Ee = 2.2GPa. heen the pressure and temperature if the process is isentropic. Use k = 1.3
IB CHAPTER ONE
Properties of Fluids
FLUID MECHANICS
& HYDRAULICS
FLUID MECHANICS
& HYDRAULICS
CHAPTER ONE
Properties of Fluids l9
ll
Solution Problem 1- 18
I
(n\ For isothermal condition: Two large plane surfaces are 25 mm aPart and the space between them is filled
pr Vt : pz. Vz with a liquid of viscosity p = 0.958 Pa-s' Assuming the velocity gradient to be
tts0(9) = pz (2) a straight line, what force is required to pull a very thin plate of 0.37 m2 area al
a constant speed of 0.3 m/s if the plate is 8.4 mm from one of the surfaces?
Pz = 675 kPa abs

(b) For isentropic process: Solution


r_ Fr+Fz
r-
p'rVtk = pzVzk
150(9)'.3 = p2 (2)r.t
pz = 7,O60 kPa abs ' u/v
T

F /A 25r

Tr= lPz / \(k-l)/k


I
' u/v
T1 lp' .J
F= VUA
l j v
T. (
:tt 1.060 \(r'3-r)/
24+273 [ tso J 11 -
0.958(0.3)(0.37)
00166 - o.4 iri
Tz = 466.4'K or 193.4'C
0.958(0.3)(0.3n
F -+-12.o6N -,
rz- 0.00g4
Probfem I - L7 F=6.4+12.66
If the viscosity of water at 70 "C is 0.00402 poise and its specific gravity is 0.978 F = 19.05 N
determine its absolute viscosity in Pa - s and its kinematic viscosity in m2/s
and in stokes.
Problem I - 19
Solution
A cylinder of 725 mm radius rotates concentrically inside a fixed cylinder o
Absolute viscosity: 130 mm radius. Both cylinders are 300 mm long. Determine the viscosity o
0.1 Pa-s the liquid which fills the space betweep the cylinders if a torque of 0.88 N-m i
u = 0.00402 poise x
lpoise rerluired to maintain an angular velocity of 2n radians/sec Assume th,
I = 0.000402 Pa - s vekrrrity gradient to be a straight line

Kinematic viscosity:
v=:= u 0.000402
p (1000 x 0.978)
v = 4.11 x1V7 mzfs

l stoke
v = 4.11 x 1o-7 n'z/ s*
0.0001m2/s
v = 4.11 x 1(}3 stoke

i
I
CHAPTER ONE FLUID MECHANICS ONE
Ia t.
FLUID MECHANICS CHAPTER
20 Properties of Fluids & HYDRAULICS & HYDRAULICS Properties of Fluids
Solutlon [tr, = 0]
fixed
Wsin0-F =0
u= u/v
' Nt = o.oo5 m
F =Wsin0
i*-\.* F'= 176.58 sin 15'
U--ra u =-olles F =tA

U = 0.1,25(2n) rotating
rylinder
U = 0.785 m/s fixed cylinder lF.=tA=ug-Al
v
V = 0'005 m
t76.5|sin 15o = 0.0si4 (0.3)
Torque = F(0.125) ojmL
Torque: tA (0.125) U = 5.614 m/s
pr = 5.614 m/s
0.88 = r [2n(0.12s)(0.3)l (0.125)
t = 29.88 Pa L=0.3m
liquid
29.88
Problem 1- 21
t,.= o.zsslo.oo5 Estimate the height to which water will rise in a capillary tube of dianreter 3
0.005
p = 0.19 Pa-s mm. Use a = 0.0728 N/m and y = 9810 N/m3 for water.
0.13 m

Solution
Problem 1- 20 Note: 0 = 90o for water in clean tube
An 18-kg slab slides down a 15' inclined plane on a 3-mm-thick film of oil Capillaryrlse,h= I
with viscosity Lr = 0.0814 Pa-sec. If the contact area is 0.3 m2, find the terminal yd
velocity of the slab. Neglect air resistance. 4(0'0728)
CaPillary rise,l = e810(0.003)
Solution Capillary rise, /r = 0.0099 m = 9.9 mrn
W = 18(9.81) = 176.58 N

s
y = 0.003 m
Problem t - 22
llrtimate the capillary depression for mercury in
dlameter. Use o = 0.514 N/m and 0 = 140".
a glass capillary tube 2 mm in

plane lolutlon
4ocos0 4(0'514Xcos140')
yd -
0= Capillary rise, h =
(9810x13.6X0.002)
Capillary rise, I = -0.0059 m (the negative sign indicates capillary depression)
Terminal velocity is attained when the sum of all forces in the direction of
mction is zero. Capillary depression, lr = 5.9 mm
CHAPTER ONE
CHAPTER ONE FLUID MECHANICS FLUID MECHANICS 23
22 Properties of Fluids & HYDRAULICS d nYoRnuucs or"p-"l,;;
"?il;'
Problem 1- 23 Problem L - 26
What is the value of the surface tension of a small drop of watgr 0,9 Tall A sonar transmitter operates at 2 impulseS per second. lf the device is held tb
tf'ta t"ifu.. of fresh water'(Er = 2'04 x10e P;) and the echo is received
pidwal'
diameter which is iri contact with air if the pressure within the droplet is 561
Pa? between impulses, how deep is the water?

Solution Solution
4a The velociry of the pressure wave (sound wave) is:
P= -.:d
lr
ILB
4o '- {;
56"1=
0.0003
o = 0.042 N/m =7,428m/ s

the echo is received


Problem L - 24
way between impulses, tlien
An atomizer forms water droplets 45 pm in diarneter. Determine the excess
total time of kavel of sound,
pressure within these droplets using o = 0.0712 N/m. 1/z(0.5) = % sec and the total
covered is 2h, then;
Solution

p=
4o 2]t=ct
d Eh ='1,,428(1/4
4(0'0712) 'li = L78,5 m
p= = G,329 pa
45 x-10-o

t -27
Problem t - 25 pressure will 80 "C water boil?
pressure of water at 80oC = 47.4l<Pa)
bistiiled water stands in a glass tube of 9 mm dfmeter at a height of 24 mm
What is the true static height? Use o = 0.0742 N/m.

Solution will boii if the atmospheric pressure'equals the vapor piessure


, 4ocos0
n=- water at 80 "C will boil at 47.AkPa
'yd
where 0 : 0" for water in glass tube

4(0'0742)
tI= = o.oo336 m = 3.35 mm
9810(0.009)

True static height = 24 - 3.36


True static,height = 20.54 mm
CHAPTER ONE
2+ Properties of Fluids
FLUID MECHANICS
6. HYDRAULICS
FLUID MECHANICS CHAPTER ONE
zaE
J
& HYDRAULICS Properties of Fluids
upplementary Problems Problem 1 - 33
(a) If 12 m3 of nitrogen at 30oC and 125 kPa abs is permitted to expand
Problem 1- 28 isothermally to 30 m3, what is the resulting pressure? (b) What would the
what would be the weight of 1 3-kg mass or a planet where pressule and iemperature have been if the process had been isentropic?
the acceleration
due to gravity is 10 m/s2? Ans: (a) .50 kPa abs
Ans:30 N (b) 34.7 kPa abs; -63'C

Problem 1- 29 Problem 1 - 34
A vertical cylindrical tank with a diameter of 12 m and a depth of 4 m is filled A square block weighing 1.1 kN and 250 mm on an edge slides down an
with water to the top with water at 20"C. If the water is heated to 50.C, how incline on a film of oil 6.0 pm thick. Assuming a linear velocity profile in the
much water will spill over? Unit weight of water at 20'C and 50oc is 919 oil and neglecting air resistance, what is tl're terminal velocity of the block?
kN/m3 and 9.69 kN/m3, respectively. T'he viscosity of oil is 7 mPa-s. Angle of inclination is 20o.
Ans:4.7 m3 Ans'.5.1.6 m/ s

Problem 1- 3O Problem I - 35
A rigid steel container is partially filled with a liquid at 15 atm. The volume of llenzene at 20'C has a viscosity of 0.000651 Pa-s. What shear slress is required
the liquid is 1.23200 L. At a pressure of 30 atm, the volume of the liquid is to deform this fluid at a strain rate of 4900 s-1?
1.23100 L. Find the average bulk modulus of elasticity of the liquid over the
Atts: r -- 3.19 Pa
given range of pressure if the temperature after compression is allowed to
return to its initial value. What is the coefficient of compressibility?
Ans" Ea = 1872 GPa; 0 = 0.534 GPa-' Problem 1 - 36
A shaft 70 mm in diameter is being pushecl at a speecl of 400 mm/s through a
lrt'irring sleeve 70.2 mm in diameter and 250 mm long. The clearance, assumed
Problem t - 31
ltlliform, is filled with oil at 20.C with v = 0.005 m2/s and sp. gr. = 0.9. Find
Calculate the density of water vapor at 350 kPa abs and 20'c if its gas constant llrc force exerted by the oil in the shaft.
is 0.462 ppu-6r/kg-"K
Atzs: 987 N
Ans:2.59 kg/ m,

Problem t - 37
Problem 1- 32 'l'w<r clean parallel glass plates, by a distance d ='J..5 mm, are ctipped
separated
Air is kept at a pressure of 200 kPa and a temperature of 30oC in a 500-L ln n bath of water. How far does the water rise due to capillary action, if o =
contajner. What is the mass of the air? (),()730 N/m?
Ans: 1.15 kg Ans:9.94mn
CHAPTER ONE FLUID MECHANICS FLUID MECHANICS CHAPTER T\vO
26 Properties of Fluids & HVDRAULICS & HYDRAULICS Principles of Hydrostatics Z I
Problem 1- 38
Iiincl the arrgle the surface tension film leaves the glass for a vertical tube
irnmersed in water if the diameter is 0.25 inch and the capillary rise is 0.08
inc-'h. Use o = 0.005 lb/ft.
Chapter 2
Arts:64.3" Principles of Hydrostatics
Problem 1 - 39
UNIT PRESSURE OR PRESSURE, p
What force is required to lift a thin wire ring 6 cm in diameter from a water
Pressure is the force per unit area exerted.by a liquid or gas on a body or
surface at 20'C? (o' of water at 20"C = 0.0728 N/m). Neglect the weight of the
surface, with the force acting at right angles to the surface uniformly in all
ring.
directions.
Ans:0.0274N

Force. F
P= .--- A.
' Area, Eq.2-1

ln the English system, pressure is usually measured in pounds per square inch
(psi); in international usage, in kilograms per square ce.timeters (k[/cm2), or
in atmospheres; and in the international metric system (sI), in Newtons per
Bquare meter (Pascal). The unit atmosphere (atm) is defined as a pressure of
1.03323 kg/cm2 (74.696 lb/inz), which, in terms of the conventional mercury
batometer, corresponds to 760 mm (29.921in) of mercury. The unit kilopascal
(kPa) is defined as a pressure of 0.0102 kg/crft (0.1a5 lb/sq in).

PASCAL'S LAW
Ittrst:nl's lazo, cleveloped by French mathemati cian Blaisc pnscal,states that the
l'lrcssure on a fluid is equal in all directions and in all parts of the container. In
Figure 2 -'1,, as liquid flows into the large container at the bottom, pressure
pusl'res the liquid equally up into the tubes above the container. The liquid
tlgcs to the same level in all of the tubes, regardless of the shape or angle of the
ttrbc.
CHAPTER T\YO FLUID MECHANICS FLUID MECHANICS CHAPTER T\vO
2B Principles of Hydrostatics 6. HYDRAULICS & HYDRAUTICS Principles of Hydrostatics 29
ABSOLUTE AND GAGE PRESSURES
;. ;ifi*iiM;s.il
Gage Pressure (Relative pressure)
Gage pressures are presswes above or below the atrnosphere and can be
measured by pressure gauges or manometers. For srnall pressure differences, u-
a
fube manometer is used. It consists of a u-shaped tube
-ith o.ru end connected
the container and the other open to the atnosphere. Filled with a liquid, such
to
as
water, oil, or mercury, the difference in the liquid surface levels in the two
manometer legs indicates the pressure difference from local ahnospheric
conditions. For higher pressue differences, a Bourdon gauge, named after the
French inventor Eugdne Bourdon, is used. This consists of i hollow metal
tube
with an oval cross sectiory bent in the shape of a hook. one end of the tube is
Figure 2 - 1: Illustration of Pascal's Law closed, the other open and connected to the measurernent region.

The laws of fluid mechanics are observable in many everyda-v situations. For
example, the pressure exerted by water at the bottom of a prnd will be the Atmospheric Pressure & Vacuum
same as the pressure exerted by water at the bottom of a much narrower pipe, Atttrospluic Pressute is the pressure at any one point on the earth's surface from the
provided depth remains constant. If a longer pipe filled with water is tilted so weight of the air above it. A anarum is a space that has all matter removed from it.
that it reaches a maximum height of 15 m, its water will exert the same It is impossible to create a perfect vacuum in the laboratory; no matter how
pressure as the other examples (left of Figure 2 - 2). Fluids can flow up as well aclvanced a vacuum system is, some molecules are always present in the vacuum
as down in devices such as siphons (right of Figure 2 - 2). Hydrostatic force ett'a, Even remote regions of outer space have a small amount of gas. A vacuurn
causes water in the siphon to flow up and over the edge until the bucket is
r:an rrlso be described as a region of space where the pressure ii less than the
empty or the suction is broken. A siphon is particularly useful for emptying nrrrnral atmospheric pressure of 760 mm (29.9 n) of mercury.
containers that should not be tipped.
l-Jtrtlt.r Normal conditions at sea level:
It"r" = 2166lb/ft2
= 1.4.7 psi
= 29.9 inches of mercury (hg)
= 760 mm Hg
= 101.325 kPa

r, =Pi=P. lbrolute Pressure


Alrrrrtrrlr. pressure is the pressure above absolute zero (unumnt)
Figure 2 - 2: Illustration of Pascal's Law

r Ab5(llute zero is attained if all air is removed.


I Aurhtte pressure can never be negative. It is the lowest possible pressure attainable.
r lll€ nrtallest gage pressure is equai to the negative of the ambient atmospheric pressure.
CHAPTER T\vO FLUID MECHANICS FLUID MECHANICS GHAPTERTWO 2 t
30 Principles of Hydrostatics & HYDRAULICS & HYDRAULICS Principles of r
Hydrostatics '
VARHTIOI{S IN PRESSUR,E
Consider any two points (1 & 2), whose difference in elevation is Il, to lie in the
sB.6 ends of an eiemeniary prism having a cross-sectional area 4 and a length of L.
Standard
atmosphere = Since this prism is at rest, all forces acting upon it must be in equilibrium'

Free liquid surface


-40 gage
Current atmosphere = 100 abs

pressurcS
Absolute zero = -101.325 gage
or -100 gage

All pressure units in kPa

Figure 2 - 3: Relationship betvveen absolute and gage pressures


=Lsine
Note: Unless otherwise specified in this book, the term pressure signifies gage pressure.

MERCURY BAROMETER
A mercury barometer is an accurate and relatively
simple way to measure changes in atmospheric
pressure. At sea level, the weight of the atmosphere
forces mercury 760 mm (29.9 in) up a calibrated
A
Atmospheric Flgure 2 - 4: Forces acting on elementary prism
glass tube. Higher eleVations yield lower readings p le tsu re
because the atmosphere is less dense there, and the to liquid to zero gage press{Jre or with
thinner air exefts less pressure on the mercury, w Note: Free Uquid Surface refers
atnospheric pressure only.
surface subject

Wlth reference to Figure 2 - 4:


w-f v
1r'11= y @Ll

ANEROID BAROMETER [lF, - 0]


In an aneroid barometer, a Fz-Fr=Wsin0
partially evacuated metal drum P24-Pta=Y(aL)sin0
expands or contracts in response p2-p| =1Lsin0 butLsin0=h
to changes in air pressure, A
series of levers and springs
translates the up and down =yh .2-3
movement of the drum top into
the circular motion of the
pointers along the aneroid Threfore; the difference in pressure behueen any huo points in a lnmogeneous fuid
barometer's face. 4l nst ia equnl to thc product of the unit weight of the fluid (^il to tlu aerticnl distnnn
(h) llrhueen the points.
CHAPTER T\vO
32 Princlples of Flydrostatics
FLUID MECHANICS
& HY'DRAULICS
FTUID MECHANICS CHAPTER T\vO - -
& HYDRAULICS Principles of Hydrostatics 55
Also:
Consider the tank shown to be filled with liquids of different
clensities and
with air at the top under a gage pressure of pa, the pressure at the
bottom of
the tank is:
This means that arty change in pressure nt point 1. tuould cnu.se an equal chantge at
point 2. Thereforq a pressufe applied at any point in a Liquid at rest is h,! p'= y1h1 + yl lb + ^bhs +
trqnsmitted equally and undiminished to every other point in the liquid. =]i

Let us assume that point o in Figure 2 - 4 lie on the free liquid surface, then 'PRessunr nrno
the gage pressure pt is zero and Eq. 2 - 4 becomes:
'Pressure head is the height "rt" of a corumn of homogeneous liquid of unit
weight y that will produce an intensity of pressure p.
o -- ---
= zoh

This means that the pressure at any point "rt" beruo a


free liquid surface is equnl to
tlrc product of the unit weight of the
fluid (y) antt h.

Convert Pressure head (height) of liquid A to liquid


consider that points o and o in Figure 2 - 4keon the same elevation, such B
that h = 0; then Eq. 2 - becomes:

This means that the prcssure along the same horizontal plane in a homogeneous
fluid
at rest are equal. convert pressure head.(height) of any riquid to water,
iust murtipry its
Cht by its specific aravrty

Pressure below Layerc of Different Liquids

h2

hr

Fuotom
CHAPTER TWO FTUID MECHANICS FLUID MECHANICS CHAPTER TU(/O 2q
34 Principles of Hydrostatics. & HYDRAULICS & HVDRAULICS Principles of Hydrostatics J J
MANOMETER Steps in Solving. Manometer Problems:
A ntunometer is a tube, usually bent in a form .of a U, corttaining a liquid of t. Decide or, ,f," fluid in feet or meter, of which the heads are to be
known specific gravity, the surface of which moves proportionally to changes expressed, (water is most advisable).
of pressure. 'lt is used to measure pressure 2. Starting from an end point, number in order, the interface of different
fluids:
3. Identify points of equal pressure (taking into account that for a
Types of Manometer homogeneous fluid at rest, the pibssure along the same horizontal plane
Open Type - has an atmospheric surface in one leg and is capable ot are equal). Label these points with the same number.
measuring gage preisu res. 4. Proieed from level to level, adding (if going down) or subtracting (il
Differential Typ" without an atmospheiic surface and capable of going up) pressure ,heads as the elevation decreases or increases,
i; respectively with due regard for the specific gravity of the fluids.
measuring only differences of pressure.
Piezometer - The sirnplest form of open manometer. lt is a tube'tapped hto a i
wall of a container or conduit"for the purpose of measuring pressure. The
fluid in the container or qonduit rises in this tube to form a free surface

Limitations of Piezometer: PrOblems


. Large pressures in the lighter liquids require long tubes
. Gas pressrtres can not be measured because gas can not form a free 2-'
suiface a depth of liquid of L m causes a pressure of 7 kPa, what is the specifi<
ty cif the liquid?

Pressure, p=lh
7 = (s.81. x s) (1)
s:0.714 ) Specific Gravity

2-.2
(a) Open manometer (b) Differential rnanometer = 1.03 for salt water.
le the pressure 12.5 m below the ocean? Use sp. Cr-.

,P*Yh ' :

p * (9.81. x1.03)(12.5)
P=1:26.3kPa

(c) Piezometer
CHAPTER T\vO FLUID MECHANICS FLUID MECHANICS GHAPTER T\vO 2a
36 Principles of Hydrostatics & HYDRAULICS & HYDRAULICS Principles of Hydrostatics J t
Problem 2 - 3 Problem 2 - 5
lf the pressure 23 meter below a liquid is 338.445 kPa, deterrnine its unit If the pressure in the air space above an oil
(s = 0.75) surface in a closed tank is
weight y, mass densilv p , and specific gravity s 115 kPa absolute, what is the gage pressure 2 m below the surface?

Sotution Solution
(n) Unit weight, y p:pturtacelfll
p=vh Psurtace=115-101.325 Note: patm.= 101.325 kPa
338.445 = y (23) = 73.675 kPa gage
Psurrace
y = 14.715 kN/m3
p =13.675 + (9.81x0.7s)(2)
P = 28.39 kPa
) Mass density, p

p=l
I 1-6
x'l'03 the absolute pressure in kPa at a deplh of 10 m belovy the free surface of
P= 4,7'15
1

9s1 of sp. gr. 0.75 1f the'barornetric reading is 752 mmHg


p = 1,500 kg/m3

(c) Specific gravrty,


Prtria
s
'
pnbs=pntn+pgagr
/* = l,)i,,'
P water = (9.81 x 13.6)(0.752)
1,500 P"t. = 100.329 kPa
.5-
1,000 = 100.329+ (9.81 x 0.75X10)
Pnu,
,s=1.5
- puts = 173.9 kPa

Problem 2 - 4 7.-7
tf the pressure at a point in the ocean is 50 kPa, what is lhe pressure 27 meters gage 6 m above the bottom of the tank containing a liquid rea{s 90
below this point?
?q*"
{Another gage height 4 rri reads 103 kPa. Deterrriine the speeific'weight of

Solution
lhe difference in pressure belween any two point€ in a
liquid is pz - pt = t h

Pz-n=Yh.
" =?L..'|nr 1x1 .03) (27) 103-90=y(2)
p2 = 332.82kPa y.6.5 kN/6r
CHAPTER T\vO FLUID MECHANICS FLUID MECHANICS CHAPTER T\vO
3B Principles of HYdrostatics & HYDRAULICS & HYDRAULICS Principles of Hydrostatics 39
Problem 2 - 8 Solution
Arr open tank contains 5.8 m of water covered with 3.2 m of kerosene (f = 3 Since the density of the mud varies with depth, the pressure
kN/m3). Ilirrd the pressure at the interface and at the bottom of the tank. should be solved by integration

Solution dp=ydh
dp = (L0 + 0.5 h)dh
(u\ Pressure at the interface p5
aa
= fxht^
Pa
lao = |rc*o.snran
= (8X3.2)
pa = 25.6kPa
J'
00
J'
Kerosene
v =8 kN/m3 15
(b) Pressure at the bottom p = 10h + 0.25h2 |

Pa =2 Ylt lo
Water
= Y'' h'' + Ys Iq = 9.81 kN/m3 = [10(s) + [./$($)z] -[
= e.81(s.8) + 8(3.2) P = 56.25kPa
pn = 82.498 kPa

Problem 2 - 11
Problem 2 - 9 ln the figure shown, if the atmospheric
lf atmospheric pressure is 95.7 kPa ancl the gage attached to the tank reads 188
pressure is 101.03 kPa and the absolute I .5m
mmHg rru.,r.t*, find the absolute pressure within the tank pressure at the bottom of the tank is 'i#'flno;tr**
231.3 kPa, what is the specific gravity
Solution uf olive oil? Water 2.5 m
Ptus -- Ptnt
+ PgrgL

Pgty = f mercu'Y fut""u'Y


= (9.81 x 13.6)(0'188)
= 25.08 kPa vacuum
', oiuoJ-t .
2.9 m
"'"' . 'ri
Pg"g,' = -25 08 kPa
Mercury, s = 13.6 0.4 m
pots=95.7 + (-25.08)
P^u, = 70'62 kPa abs

tolutlon
Problem 2 - 10
(iagepressureatthebottomof thetank, p=23'l .3 -101 .03
The weight density of a mud is given by y = 10 + 0'5k, where y is in kN/m3
and
t inge' pressure at the bottom of the tank, p = 130.27 kPa
h is in mlters. Determine the pressure, in kPa, at a depth of 5 m
l;r - )-Y[]
P= T, hr, + Yo ho + ln, htu + lot l].,t
130.27 = (9.81 x 13.5)(0.4) + (9.81 . s)(2.e) + e.81(2.5) + (9.81 x 0.89X1.5)
.r - 1.38
40 CHAPTER TWO
Prlnclples of Hydrostatics
FLUID MECHANICS
& HYDRAULICS
.FLUID MECHANICS
& HYDRAULICS
CHAPTER TN/O
Principles of Hydrostaticl
4l
Problem 2 - 12 Probtem 2- 14
lf air had
a constant specific weight of 72.2 N/mr and were incompressible, 'Compute the barometric pressure in kPa at an altitude of 1,200-_m if the
what would be the height of the atmosphere if the atmospheric pressure (sea .,pressure at sea level is 101.3 kPa. Assume isothermal conditions a 21oC. Use
level) is 102 kPa? R = 287loule /kg-"K.

Solutlon

Height of atmosphe re, h =L


v

_ 102 x 103
n
72.2
Height of atmosphere, h = = 8,350.55 m
.RT
=U
287(21 + 273)
Problem 2 i 13 (CE Board May 1994) p = 0.00001185 p
Assuming specific weight of air to be constant at 12 N/m3, what is' the
approximate height of Mount Banahaw if a mercury barometer at the base of dp = -(0.00001185 p)(e.s-t) dtr
the mountain reads 654 mm and at the same instant, another barometer at the adn = 0.00011,63 dlt
top of the mountain reads 480 mm. p
p 7200
Solution
=.0.0001163 p,
,rrr!fr 0

jp 11200
lnp I . =-0.0001163/r'lI o
J 101.3x10'

lnp - ln (101.3 x 103) = - 0.0001163(1200 - 0)


lnp = 11.tt'
p = e77.386
p = 8$080 Pa

pmt-p.op=I,h
h,,)uotto^ - (y,, hr)rop = (l h)""
(Yn,

(9,810 x 13.6'X0.654) - (9,s10 x 13.6)Q.a4 ='t2h


h = 1,934.53 m
CHAPTER T!/O FLUID MECHANICS FLUID MECHANICS^
4Z Principles of Hydrostatics & HYDRAULICS 6. HYDRAUTICS
CHAPTER T\YO
Principles of Hydrostatics 43
Problem 2 - 15 Problem 2 - t8 (CE November 199g)
Convert 760 rnrn of mercury to (n) oil of sp. gr. 0.82 and (b) water. Piston A has a cross-section of 7,200 sq. cm while
that of piston B is 950 sq. cm.
with the latter higher than piston A by 1.7s m. If the intervening
passages are
Solution filled with oil whose specific gravity is 0.g, what is the differer.,iu
i^ pressure
between A and B.
s--
(rr) /rn,; = It^.rrur--!!!!!-
S oil
Solution
13.6
= 0.76 pa-ps=ypho
OU
= (9,810 x 0.8)(1.25)
hnir= 12.605 m of oil
Pa-Pa=13,734Pa
(b) /r*,u1",= h,,,"r.ury smercury
= 0.76(13.6)
ft*ater = 10.34 m of water

1200 cm2 950 cm2


Problem 2 - 16 (CE Board May 1994)
A barometer reads 760 mmHg and a pressure gage attached to a tank reads
850 cm of oil (sp. gr. 0.80). What is the absolute pressure in the tank in kPa? Problem 2 - L9

Solution
ln the figure shown, 300 mm O
llctermine the weight W
pabs=parn+pgage
lhnt can be carried bv the
= (e.81 x"13.6)(0.76) + (e.81 x 0.8)(8.5) 1,5 kN force acting on tt
Pnr', = 168.1 kPa abs phton. "

Problem 2 - 17
A hydraulic press is used to raise an 80-kN cargo 'truck. If oil of sp. gr. 0.82
acts on the piston under a pressure of 10 MPa, what diameter of piston is
required?
lolutlon
Solution Since points 1 and 2lie on the
Since the pressure under the piston is uniform: rrrnre elevatiorr, pr = ?z 300 mm Z
Force=pressurexArea
80,000 = (10 x 10!) LD2 1.5
-_-_____= = w
D=0.1 m=100mm [ (().03)z iQ.3)'
W* 150 kN
CHAPTER T\x/O FLUID MECHANICS FLUID MECHANTCS CHAPTER T\vO
44 Principles of Hydrostatics & HYDRAULICS & HYDRAULICS Principles of
AE
Hydrostatics 'f J
Problem 2 - 20 Solution
A clrunr 700 mm in diameter and filled with water has a vertical pipe, 20 mm
in cliamctc.r, attached to the top. How many Newtons of water must be
pourcd into the pipe to exert a force of 6500 N on the top of the drum?

Solution
Force on the top:
f=pxArea
6500=px f,(7002-202)
p = 0.016904 MPa
p :1.6,904 Pa

,s=0.78
lp: v hl
1.6,904 = 9810 h - p'' = y,, ttl
tpz

@
h=1.723n trF
lt1-
n
-
0.00323
Weight=yxVolume
P=309.6F (kPa)
= 9810 x ! (0.02)2(1.723) Area on top
0)= _
w44
Weight = 5.31N
A 0.323
700 mm A
Pz = 136.22 kPa

Problem 2 - 2L 136.22 - 309.6 F= (9.81 x 0 7ti)(4.6)


The figure shown shows a setup with a vessel containing a plunger and a l'= 0.326 kN = 325 N
cylinder. What force F is required to balance the weight of the cylinder if the
weight of the plunger is negligible?

Cylinder
W=44kN F= lhs hydraulic press shown rs filled with oil with sp. gr. 0.82. Neglecting the
A = 0.323 m2 gFlght of the two pistons, wlrat force F on the handle is required to support
tlte lllkN weight2

.4.6 m

Oil, 5 = 6.73 Oil, s = 0.78


CHAPTER T\vO
46 Principles of Hydrostatics
FLUID MECHANICS
& HYDRAULICS
fLUID MECHANICS
6. HYDRAULICS
47
$olution since the gage reads "FULL'trren the reading is equivarent to 30 cfl of gasoJine
Sinct' points I and 2 lie on the same
t'lt'vation, then; Reading (pressure heaci) when the tank contair.r
l)t = Itz water : * 2#
(y ) cm of gasoline
f,
=F,
A1 A2 fher.r; y+2'fu =30
10 F2
t1 = 27.06 cm
tQoTrz i (0.025)2
F, =.1 .11 kN
Problem 2-24 (cE B@
[IMo=0] F'or the tarrk shown the value
in the Figure, llr = 3m and /ir = 4 m Deterrine
F(0.42s) = Fr(0.02s) <tf 14
F(0.425) = 1.11(0.025)
F = 0.0654 kN
r=65.4N

FBD of the lever arm

Problem 2 - 23
The fuel gage for a gasoline (sp. gr. = 0.68) tank in a car reads proportional to Solution
its bottom gage. If the tank is 30 cm cleep an accidentally contaminated with 2 Summing-up pressure head
cm of water, how many centimeters of gasoline does the tank actually contain from 1 to 3 in meters of water
when the gage erroneously reads "FULL"?
tL
D"
+hr(0.84)-t= 41
Solution
Yy
0+0.84h2_g_r=o
hz = 1|1.9 m

r T_
I

30 cm

2.rL I
J

"Full"
CHAPTER I\vO
48 FLUID MECHANICS FLUID MECHANICS
Principles of Hydrostatics & HYDRAULICS & HYDRAULICS e.i,,.ipr.,l?ffi*rY,.: 49
Problem 2 - 25 (CE Board May 1992) Problem 2 - 26
ln the figure shown, what is the static pressure in kpa in the air chamber?
For the manometer showry
determine the pressure at the
center of the pipe,

t--
= 13.55

L
m

Solution
The pressure in the air space Solution
equals the pressure on the surface
9um-up pressure head from
of orl, pt.
1 to 3 in meters of water:

Pr=0
Yt
D.
+1(13.55) + 1.5(0.8)= Iq
fy
Pz = fu, hu 0+14.75= la
= e.81(2) f
pz = 19.62kPa
tl = u.zsm of water ,s = 13.55

- pt t yu lt,,
v
pz
19.62- pt= (9.81' 0.S0X4) Pt= M.75(9.81)
pt = 144.7 kPa
pt = -1'1,.77 kPa

Another solution.
Sum-up pressure head fronr l to 3 in meters of water
. Pt *2-4(0.80)= ta
Yy
o+z-3.2= Pt
9.81
p. = _1.1.77 kpa
CHAPTER T\I/O
50 CHAPTER TUTO
Principles of Hydrostatics
FLUID MECHANICS
& HYDRAULICS
FLUID MECHANICS
& HYDRAULICS principles of Hydrostatics 5l
Problem 2 - 27 (CE Board November 2001) Problem 2 - 2A (CE May 1993)
L)c.tcnrrirre the value of y in the manometer shown in the Figure. ln the figure shown, when the
funnel is empty the water surface
-T- is at point A and the mercury of
1m sp. gr. 13.55 shows a deflection of
.,
T- 15 cm. Determine the new
I
deflection of mercury when the
3m funnel is filled with water to B

+-
1m
J

Solution

Solution
-T-
Summing-up pressure head from 1m
A to B in meters of water:
14+ + ru-
l-
'l,

yy e(0.8) 1.s - y(13.6) =


I

3m
5 * g.9-13.6r,=!-E-
9.8't " y t-
1m
I

where pa = 0 .'

11= 0.324m

Figure (a):'Level at A Figure (b): Level at B

Solve for y in Figure (a):


sum-up pressure head from A to 2 rn nleters ot water
o^ * la
Y"y r-
0.15(r3.5s) =

0+y - 2.03 = 0
7=2.03m
CHAPTER T\VO FLUID MECHANICS FLUID MECHANICS
5Z Principles of Hydrostatics & HYDRAULICS & HYDRAULICS
CHAPTER TWO
Principles of Hydrostatics 53
In Figure (b): Sum-up pressure head from Z to m jn ineters of water:
When the funnel is filled with water to B, point 1 will move d.own to 1,
with the same value as point 2 moving up-to Z' . P'*rt0.3.6\-y=P*
Y "' y
13.5y 70
Sum-up pressure head from B to 2': -x= 9.81 Eq (1)
In Figure (b):
P-P +
o.s * y + *- (r+ 0.t5 + x)(Ig.ss) = 2
Y-y Sum-up pressure head from 2, to m,in meters of water:
0 + 0.80 + 2.03+ x -27.'tx -
26.1 x = 0.80
2.03 = 0
2 +Q.2sino+ y+0.2)(1g.6)-(r+0.2) - Pnt'

x: 0.031 m = 3.L cm
Ty
0 + 2.7Zsin 0 + 12.6y + 2.72 _ x_ 0.2 = ffi
New reading, R = 15 + ?t = 1,5 + 2(3.1) 13.6y - x = 8.183 - 2.72 sin 0 Eq.(2)
New reading,R=X.2cm
[13.6y-x=L3.5y-x]
8.183-2.72sine=#
Problem 2- 29 sin 0 = A3852
The pressure at point rz in the figure 0 = 22.660
shown was increased. from 70 kPa to
105 kPa, This causes the top level of
mercury to move 20 mm in the sloping 2-30
tube. What is the inclination, 0? closed cylindrical tank contains 2 m of water, 3 m of oil (s 0.g2) and
= the air
ove oil has a pressure of 30 kpa. If an open mercury manorneter at the
of the tank has 1 m of water, determine the deflection of mercury.

Solution
Sum-up pressure head from
1 to 4 in meters of water:
n
!!tL a3(0.82) +z+L-y(13.0y= ll
Yy
# *2.+o+3-13.6y=0
y = 0.626m.

Figure (a) Figure (b)

In Figure (a):
CHAPTER T\vO FLUID MECHANICS FLUID MECHANICS
54 Principles of Hydrostatics & HYDRAULICS & HYDRAULICS
CHAPTER T\x/O
Frinciples of Hydrostatics 55
Problem 2 - 31 In Figure (&):
llrt' Lj-tubc shown is 10 mm in diameter Summing-up pressure head from 1 to 3 in mm of water:
,rrrtl contains mercury. If 12 ml of water is lL +752.8_R(13.6): u
pourccl into the right-hand leg, what are Yy
lhc ultirlate heights in the two legs? R = 11.24 mm

--1
I
In Eq. (2):
E 11.24+2x=240
E r:1'14.38 mm
N

I
I
Ulhimate heights in each leg:
*
Right-handleg,hn=h+x
Solution
L ,ro * --'l
= 1.52.8 + 't14.38
Right-han d leg, hn = 267 ;1,8 rnm
5olvrng for /r. (sar ftgttrr'h1
Lefrhand teg' ht =
Volume of water = + (#F ir = l2 cml Note: 1ml =1cm3
fr.rX . 114.38
h = 15.28 crn = 152.8 mm
Left-hand Leg, ht = 1%.62 rnm
Since the quantity of mercury before and after water is poured
remain the same, then;
Problem 2 - 32
120(3)=R+-r+120+r Fnr a gage reading of -17.7kpa,
R+2x=240 )Eq.(1) dE'termine the (a) elevations of
the liquids in the open
plezometer columns E, F, and
G and (b) the deflection of the
tnercury in the U-tube
manometer neglecting the
welght of air.

I -l
1

E E
E E
N N

I
I I

J J

t_ 120 mm
-t
-J
Figure (b)
CHAPTER I\vO
56 Princaples of Hydrostat,cr
FLUID MECHANICS
FLUID MECHANICS CHAPTER T\VO FA
& HYDRAULICS & HYDRAULICS Principtes of Hydrostatics , I
Solution
Column G
Sum-up pressure head from 1 to g in meters of water;
lL + se.z)+ a(1) - tr{1.6) = !-E-
vy
1-
Jm
E!. _15
E!. _r)_m_
# *2.1+4-7.6fu=s
ht= 2.'U^
El
E_l
_12-*_ Surface elevation = 8 + /rs
Surface elevation = 8 + 212 = 7O.72 m
1
4m
Deflection of mercury
EI
El._8 ln _
Sum-up pressure head from 1 to 5 in meters of water;

- h4(1s.6) : !-9-
Pr + zp.4 +4+4
1 v
4m
# *'t0.1.-13.6tu
ha= 0.6'L4m
t ---v
El.4m

Problem 2 - 33
An open manometer attached to a pipe shows a deflection of 150 mmHg with
Column E
the lower level of mercury 450 mm below the centerline of the pipe carrying
Sum-up pressure head from 1 to c in metes of water;
water. Calculate the pressure at the centerline of the pipe.
P' *trr(0.7)= P,
yy Solution
# *ftr(07)=o
/rr = 2.5 m
Surface elevation = 15 - /rr
Surface elevation = 15 - 2.5 = 1Z5 m

Column F
Sum-up pressure head from I tp /in meters of water; 8um-up pressure head from 1

Dt Pr 9ln meters of water;


t- +3(07)-hr(1\=''
yy b- + 0.45- 0.15(13.6) = l
v
i# *7'1 -h'=o
P,
ltz = g'3t' *0.45-2.04=o
Surface ^ = 12 + lu
elevation
9.81
= L5.6 kPa
' Surface elevation = 12 + 0.357 = 1'2.3ST m ;r1
CHAPTER T\VO FLUID MECHANICS FLUID MECHANICS
58 Principles of Hydrostatics & HYDRAULICS & HYDRAUTICS
CHAPTER T\vO
Principles of Hydrostatics 59
Problem 2 - 34 Solution
lior llre configuration shown, calculate the (a) Gage liquid = mercury, /e = 0.1 m
wt.ililrt of the piston if the pressure gage
Sum-up pressure head from
rt,irrling is 70 kPa.
1 to 4 in meters of water;

Pr + +
x h - tx(13.6) - x - 1.5 = U
v v

-'a-
Pt f),
1.5-0.1 +0.1(13.6)
v v'
Pt - Pq =2.76mof water
v v
Solution
Surn-up pressure head from (1,) Gage liquid = carbon tetrachloride
A to B in meters of water; Ps = 70 kPa
reading, h = ?

P o -t (0.86) : r-t B
Sum-up pressure head from 1 to 4 in meters of water;
vv
tl P"t + x + lL
h - tr('1.5g\ - x - 1.5 =
Pe _o.se 7o
= 9.81 Yy
9.81 Pr -P+
= 78.44kPa =1.5+0.59/r
:rt,t
yy
FA=pAxArea
Weight = Fn where Pt - P+ = 2.76 m) from
(a)
yv
= PA x Area II

7v.aa x e)2 2.76=1.5+0.591t


i Ir= 2.136m
Weight = 61.61kN

Problem 2 - 35 ln thr. figure shown, determine Air, p = 175 lPu u6t


Two vessels are connected to a differential manorneter using mercury, tht' the height /r of water and the
connecting tubing being filled with water. The higher pressure vessel is 1.5 m
lower in elevation than the other. (a) If the mercury reading is 100 mm, what
lrge reading at A when the
lbnolutc pressure at B is 290
T-
I o
oo
h Water
is the pressure head difference in meters of water? (b) If carbon tetrachloriclc lPr,
(.s = 1.59) were.used instead of mercury, what would be the manomett't'
re.rding for the same pressure difference?
fI

700 mm
J,c
CHAPTER T\YO T\vO , t
60 Principles of Hydrostatics
FLUID MECHANICS
& HYDRAULICS
FLUID MECHANICS
& HYDRAULICS
CHAPTER
Principles of Hydrostatics O I
Solutlon Sum-up pressure (gage) head from L to 4 in meters of water;
Sum-up absolute pressure head
.tL lL
from B to 2 in rneters of water;
Air, p = 175 ;,Pu u5t
yy+ xe.s)+ 1.3(0.e) -1.3(12.6)=
lL -o.z1s.e)-h= lz f-- 4o *0.9x-16.51=o

f-
Yy h € watei 9.81.

ffi -s.sz-t'=
h = 2.2O3 m
# wt
tl
o
r = 13.81m

700 mm M;r*ry/ o
Then,x+!/=28.42m
Jc
B

Sum-up absolute pressure head from B to A in meters of water; Problem 2 - 38


0o
!!L !-t ,For the manometer sefup shown,
- o.z(tZ.o) + o.z =
Yy determine the difference in pressure
between A and B.
ffi -s.sz+ s.7 = 9.81
Pa

pa = 203.5 kPa abs

Problem 2 - 37
In the figure shown, the atmospheric Solution
pressure is 101 kPa, the gage x+0.68=y+1..7
reading at A is 40 kPa, and the vapor x-y=1.02m )Eq.(1)
pressure of alcohol is 72 kpa
absolute. Computer+y. Sum-up pressure head from A to B
in meters of water;
-.- 0.68(0.85) + y = a_
? v

Solution
? + =x-v+0.s78 ) Eq.(2)

Sum-up absolute pressure head from Substitute x - y = L.02 in Eq. (1) to Eq. (2):
l" to 2 in meters of water; Pn
yv-Ps =1..02+o.sr|
lL _ yp.t1= ?L Pa-Pn
Y.r =1.SSS
9.81
40 + 101_u.9a= _12 kPa
Pa - Ps = 15.58
9.81 " 9.81
y = 74.67 m
CHAPTER T\VO FLUID MECHANICS CHAPTER T\)(/O
62 Principles of Hydrostatacs & HYDRAULICS
FLUID MECHANICS
6. HYDRAULICS Principles of Hydrostatics 63
Problem 2 - 39 Problem 2 - 40
A differential manometer rs In thg figure shown, the
attached to a pipe as shown deflection of mercury is initially
Calcu late the pressure 250 mm. If the pressure at A is
.lifference between points ,4 increased by 40 kPa, while
and ll maintaining the pressure at B
constant, what will be the new
mercury deflection?

Solution

t00 mm
l
T- Solution

+ffi-rr 1

Sum-up pressure head from A to B in rneters of water


n"
''^ -r(Tn\ 01(136) +01(09) +y(09) = {g
VV

Pa U = 01(13 6) () l(0 e)
vy
Pa - Pa
= t.L/ trt
9.81
PA - PB = 12.46 kPa

Figure (a) Figure (b)

In Figure d, sum-up pressure head from A to B in meters of water;


!t !j-
yv-s.6-0.25(13.6)+0.25+ 2.1=
Pa Pn
= 1.65 m of water
v v
CHAPTER T\VO FLUID MECFIANICS CHAPTER T\x/O ,, r
64 Principles of Hydrostatics & HYDRAULICS
FLUID MECHANICS
& HYDRAULICS Principles of Hydrostatics (,)
ln Figure b, po = pt + 40 Solution
Sum-up pressure head from A lo B tn meters ot water
ft- - (0.6- r) (0.25 + 2xlt36 + (2.35 * ,) = Pt
Dd

YV
o" t40 250 mm

Vv

Pa * 4o -r 65 - 25.2:r = Pa
v .9.81
Pa U =zs.zt-2.423 But PA U = t.ts
VV

t65=25.2t-2.423
r=0162m=162mm
Sum-up pressure head from A to B in meters of water;
New mercury deflection = 250 + 2r = 250 * 2(162\ !-4- + 0.2(0.8s)- 0.09(13.6)- 0.31(0.82) + 0.25 - 0.1(0.0012) =
Ps
New mercury deflection = 574 mm
P e - Pn = Losi23m of water
fl
Problem 2 - 4t pA - pB = 9.81 (1.0523) = 10.32 kPa
ln the figure shown, determrne the difference in pressure between points A
and B
2 - 42 (CE Board)
Assuming normal barometric pressure, how deep in the ocean is the point
here an air bubble, upon reaching the surface, has six Limes its volume than
had at the bottom?

Applying Boyle's Law


(4ssuming isothermal condition)

lp Vt = pzVzl t-
h
pt = 101..3 + 9.81(1.03)lr
pt = 101.3 + 10.104 h
l,
v'l -- 1,
v
pz=101.3 + 0 = 101.3
Vz- 6V
(101.3 + 10.104/r)14 = 101.3 (6 t/)
10.104 h=10t.3(6) - 101.3
h = 50.13 m
CHAPTER T\vO FLUID MECHANICS FLUID MECHANICS CHAPTER TWO r a
66 Principles of Hydrostatics & HYDRAULICS & HYDRAULICS Principles of Hydrostatics O t

Problem 2 - 43 Since the pressure in air insicle the tube is uniform,


A vertical tube, 3 m long, with one end closed is inserted vertically, with the then pc - p6 = 20.0124 kPa
open end down, into a tank of water to such a depth that an open manometer Pr=T*h
connected to the upper end of the tube reads 150 mm of mercury. Neglecting 20.0124 = 9.81.h; h = 2.04 m
vapor pressure and assuming normal conditions, how far is the lower end of
the tube below the water surface in the tank? Then; x=h+ y =2.04+ 0.495
x = 2.535 m
Solution Area = A

Probfem 2 - 44
0.15 m
A bottle consisting of a cylinder 15 cm in diameter and 25 cm high, has a neck
which is 5 cm diameter and 25 cm long. The bottle is inserted vertically in
Mercury
?: Y; water, with the open end down, such that the neck is completely filled with
water. Find the depth to which the open end is submerged. Assume normal
barometric pressure and neglect vapor pressure.
,:':,,:,

x,. Solution 15cmO

Applying Boyle's Law


ptVt = pzVz
Applying Boyle's Law:
prVt= pzVz Before the bottle was inserted:
Volume of air:
Before the hrbe was inserted; + (5)r(25)
V, = i(1s), (2s) f
Absolute pressure of air inside, pt = 101.3
Volume of air inside, Vt = 3A Vt = 4,908.V4 cm\

Absolute pressure in air:


When the tube was inserted; 7 ='10'l'325
Absolute pressure of air inside, pz -- \01,.3 + 9.81 (13.6X0.15)
Absolute pressure of air inside, pz -- 721.31 kPa When the bottle is inserted:
Volume of air inside the tube, Vz= (3 - y)A Volume of air:

lptVr= pzVzl
vz= (15), (25)i
Vz= 4,417.9 cml
101.3 (3 A) = 121.3t t (3 - y) aI
Pressure in air:
3-Y=2'sos
pz='10"1.325 + 9.8-l h
A=0495m
From the mahometer shown; lp Vr = pzVzl
101.325(4,908,7a) = $01.32s + 9.81 h)(4,417 9\
pt = y,,, h,,,
101.325 + 9.8t h = 112.58
= (9.81 x 13.6)(0.15)
ft = 1.15 cm
pn = 20.0l24kPa
x=ltr25=26.15cm
CHAPTER T\vO
e,i,,.ipru,1?fi;it*#,.: 69
FLUID MECHANICS FLUID MECHANICS
68 Principles of Hydrostatics & HYDRAULICS & HYDRAULICS

Problem 2 - 45
Supplementary Problems
A bicycle tire is inflated at sea level, where the atmosfiheric pressure is 101.3
kPaa and the temperature is 27 "C, to 445 kPa. Assuming the tire does not
Problem 2 - 46
expand, what is the gage pressure within the tire on the top of a mountain
where the altitude is 6,000 m, atmospheric pressure is 47.22 kPaa, and the A weather report indicates the barometric pressure is 28.54 inches of mercury'
temperature is 5'C. What is the atmospheric pressure in pounds per square inch?
Ans: t+.02 ps;

Solution
Ptvt = Pzvz Problem 2 - 47
T1 T2
The tube showrr is fillect with oil. Determine the pressure heads at B and C irr
meters of water.
At sea level:
Pn
Absolute pressure of afu, pt ='107.3 + 445 Ans: = -2.38ln
v
Absolute pressure, p1= = 546.3kPaa
Volume of air, Vt = V Pc
= -0.51m
Absolute temperature of air. Tr = 21 + 273 = 294 "K v

On the top of the mountain:


Absolute pressure of air, pz= 47.22 + p
Since the tire did not expand, volume of ait, Vz= V
Absolute temperature of air ; Tz = 5 + 273 = 278 "K

tT=hr
,PtVt - PzVz ,
s = 0.85

s46.3(v) $7.22+P)V
'294 - 278
2- 48
47'22+ P = 5L6'57 For thb tank shown in the figure, compute the pressure at points B, C, D, and E
= 469'35kPa kPa. Neglect the unit weight of air
P
Arts pu = 4.9; pc = po = 4.9; Pt = 21.64
CHAPTER T\VO FLUID MECHANICS CHAPTER T\VO
70 Principtes of Hydrostatics & HYDRAULICS
FLUID MECHANICS
& HYDRAUTICS Principles of Hydrostatics

Problem 2 - 49 2-52
A'gtass U-tube open to the ahnosphere at both ends is shoWn. lf the U-tube cylindrical tank contains water at a height of 55 mm, as shown' Inside is a
contains oil and water, determine the specific gravity of the oil open cylindrical tank containing cleaning fluid (s.g. = 0.8) at a height /r-
Ans: 0.86 e pressure pe = 13.4 kPa gage and pc = -13.42 kPa gage. Assume the cleaning
id is prevented from moving to the top of the tank. Use unit'weight of
= 9.79 kN/m3. (a) Deterrnine the pressure pe in kPa, (b) the value of h in
and (c) the value of y in millimeters.
Ans: (a) 12.88; (b) 1,0.2; (c) 101

Problem 2 - 50
A glass 12 cm tall fillecl with water is inverted. The bottom is open. What is
the pressure at the closed end? Barometric pressure is 101.325lclL
rOO.rU OO",

Problem 2 - 51
ln Figure'13, in which fluid will a pressure of 700 kPa first be
".nt?,ilt rr*rr"
2-53
Po = 90 kPa
tial manometer shown is.measuring the differen'ce in pressure two
pipes. The indicating liquid is mercury (specific gravity + 13.6),ln is 675
Itrn is 225 mm, and h,,,2 is 300 mrn. What is the pressure differential
ethyl alcohol
p = 773.3 kglm3
60m the two pipes.
Ans:89.32kPa

oil
p
10m
= 899.6 k9/m3
water
p = 979 kglmr 5m
glycerin
p= 1236 kglm3
5m
CHAPTER T\vO
72 Prlnclples of Hydrostatics
FLUID MECHANICS FLUID MECHANICS CHAPTER THREE
& HYDRAULICS & HYDRAULICS Total Hydrostatic Force on Surfaces 73
Problem 2 - 54
A force of 460 N is exerted on lever AB as shown. The end B is connected to a
piston which fits into a cylinder having a diameter of 50 mm. what force Fo
dcts on the larger piston, if the volume between c and D is filled with water?
Chapter 3
Ans: 15.83 kN
Total Hydrostatic Force
on Surfaces

TOTAL HYDROSTATIC FORCE ON PLANE SURFACES


.lf tlre pressure over a plane area is uniform, as in the case of a horizontar
;Nurface submerged in.a liquid or a plane surface i"rla. u-g*';;;;^;;
total hydrostatic force (or total pressure) is by:
liven

Problem 2 - 55 where p is the uniform pressure and A is the area.


An open tube open tube rs attached to a tank as shown. lf water rises to a
height of 800 mm in the tube, what are the pressures pa ancl pB of the air above In the case of an incrined. or-vertical plane submerged in a
liquid, the total
water? Neglect capillary effects in the tube. Pressure can be found by the followingior*rtu,
Ans: pe = 3.92kPa; pz = 4.9Q kPa

of gravity, cg

of pressure, cp
CHAPTER THREE FLUID MECHANICS FLUID MECHANICS CHAPTERTHREE qF
74 Total Hydrostatic Force on Surfaces & HYDRAULICS & HYDRAULICS Total Hydrostatic Force on Surfaces L
Consider the plane surface shown inclined at an angle 0 with the horizontal. LOCATION OF F (yp):
-[o In Figure 3 - l, taking moment of force about s, (the intersection of the
get thc' total force F, consider a differential element of area dA. Since this
element is horizontal the pressure is uniform over this area, then; prolongation of the plane area and the liquid surface),
f
dL=pdA ry,=
Jyar
where p = yh
where dF=yysin0dA
P=YYsin0 F=ysin0 Ay
dF = .r y sinO dA
y sin 0 AT yp - v aa)
!v0 "i^e
[n,=rsi,,e Jvan
ysin0Ayyr=ysin0
I
I
From calculus, ly dn = Ay JV'aa
J
P=ysin0 Al From catcutu dA = Is (moment of inertia about
F=y(tsin0)A ", Ir2
S)

AYYp=ls
From the figure, y sin 0 = ir
Then,

P=yhA Eq.3-2
By {ansfer formula of moment of inertia:
Since y lr is the unit pressure at the centroid of the plane area, pcs, the formula Is=\+ n!2
may also be expressed as:
I"+AY2
6
lfo--
F- A Eo.3-3 AY

Eq. 3 - 2 is convenient to use if the plane is submerged in a single liquid and


without gage pressure at the surface of the liquid. However, if the plane is
submerged under layers of different iiquids or if the gage pressure at tht'
liquid surface is not zero, Eq.3 - 3 is easier to apply See Problem 3 - 15 Snce yo = 7 + e, fromFigure 3 -'L, then
I

Table 3 - 1 in Page 76 for the properties of commbn plane sections.


CHAPTER THREE THREE .1
76 Total Hydrostatic Force on Surfaces
FLUID MECHANICS
& HYDRAULICS
FLUID MECHANICS
& HYDRAULICS
CHAPTER
Total Hydrostatic Force on Surfaces I I
TABLE 3 - 1: Properties of Common plane sections
Half ellipse Quarter ellipse
Triangle Rectangle

Area = h nnb
4b Area = ln ntb
v.= 4n 4b
", -3t 1_=-tt=-
k-ttz-$-6124 l-- r *-l r ab3
3n r' 3n
l,= : let=0."11 nb| - ntb3 nbn.t
r = i+b y,=tt/3 Area = bd 6' I=-l=-
a t
"16"'16
r',_bd3
db' bn3
Area=r/zblt - --;- Iy = I =-
'sY 8
,lr. = 0.055nb3 Iru= 0.055bn3
.f 3
. blt3 bhl . =- bd3 I Sector of a circle Parabolic seqment
' !8r 'st
db3
72 '&i
I
72 36
=-
1,2
Circle Quarter circle

h
Area = f (20) = 12 0
7/z

2 rsin9 Area = ?ar,


r
-; "30
2,
3
3,
Area = l/t z 12; x, = yr l
4r -4 '58-il
x"= 1t"=
-D
Ir = -f,' (0 - % sin 2e)
Area = n2 = y4 jr D2 .1fr )J
' ltbt
'15"7
I_= 1..= 'bh3
oro nDa ,14 t4
tvr - lq, - ,t - lu -
'76 Irj (e+hsn20)
464 n
-
/r,=1r=0.055/ Spandrel Segment of arc
Semicircle Ellipse

Y=kxn

l*o---*l
+t' Area= rab Length of arc = r(20) = 2tfi
Area=%nrr; y,= ;-
.tn
'i
oa3 '
= ,1 +7
1 rsn9
tr 4 'g, - -T- Area b/r = ----*
,1.
0
lr-l{r-
'8
i
1 1t.= ,r+1 \ /hen 0 = 90' (semicircle)
n ba1 ' tr+2o: "' tr
-. d
ls, = 0.11 'w- --i- 4rt+2 2r
- E
16 CHAPTERTHREE FLUID MECHANICS FLUID MECHANICS CHAPTER THREE aO
t O Totat Hydrostatic Force on Surfaces & HYDRAULICS & HYDRAULICS Total Hydrostatic Force on Surfaces t t
TOTAL HYDROSTATIC FORCE ON CURVED SURFACES A
Fu = p,c Eq.3 -7

CASE I: FLUID IS ABOVE THE CURVED SURFACE. orFs=yiA Eq.3-8


Fv=IV Eq.3-9
tan}=Fy/Fy Eq.3-70-

where:
A = vertical projection of submerged curve (plane area)
pcg = pressure at the cenhoid of A

Note: The procedure used in solving Fn is the same are that presented in Page 73.

Vertical projection FLUID BELOW AND ABOVE THE CURVED SURFACE


of the curved surface

CASE II: FLUID IS BELOW THE CURVED SURFACE

Fx

Curved surfacb

1,"
CHAPTER THREE
80 Total Hydrostatic Force on Surfaces
FLUID MECHANICS
& HYDRAULICS
FLUID MECHANICS
CHAPTER THREE
& HYDRAUTICS Total Hydrostatic Force on Surfaces BI
DAMS
l)ams are strucfures that block the flow of a river, stream, or other waterway.
' up strranl
some dams divert the flow of river water into a pipeline, canal, or channel. lates 0p en
t'
Eoat
others raise the level of inland waterways to make them navigable by ships
Ocwnttre am
and barges. Many dams harness the energy of falling water to ge'nerate electric qates 5tr enn
power. Dams also hold water for drinking a'd crop irrigatiJn, and provide |latiJ
cl!!e :
flood control.

PURPOSE OF A DAM
Dams are built for the following purposes:
1. Irrigation and drinking water
2. Power supply (hydroelectric)
oprn
3. Navigation sJuire!

4. Flood control
5. Multi purposes llgure 3 - 3: Boat passing through canar Lock.
Canar rocks are a series of gates designed
to allow a boat or shio to pass from one rever
of water to another. Here, afier a boat has
rntered the rock and ail gates ur. t".ula,-il.rJoo*nrtr"am
through them' when the water tever is equat-o; sruices open and water flows

the stuices; the do;"ro..,


;;.,. side of the downstream gate, water
il:tt#il:gJhrou,eh sr* opens, and tnu ooui.ontinues on at

PorterhDuse
To transmiiSion line!

TYPrS oF DAMs
t the force of
grarziry ro resist water pressure_
?^r:r::, _(::.,1]1r,":"tI
cereralot, :11'.i: Ji":,1:li-1"*'l: y"'". b; ;."
o.ynwald ro do this, gru,rity d^;;;;;;#;;;;:r:
"
;;-;;;; ;;in:;?:;;
l:rll'lg
so hea'uy tr-rat the water in a reser"voir "
cannot push the darn
downstream or tip it over. 'Ihey afe
much,hi.Lu. at the base than the
top-a shape that reflecrs the distribution of the
forces oiin" -ui",
against the dam. As water becomes
deeper, it exerts more horizontal
pressure on the dam. Gravity <lams
Drait tube PenrtocL Eedrock
are relativery thi. the surface
t",::.voir,.where the warer p.";;;;;-;;
:j"*:
enables the dam to 44withstand
^ear
il.i. ;il; ;il:
the bottom of the reservoir
the more i;;;;-;;,* pressure at
Figure 3 - 2: Section of a dam used for hydroelectric
CHAPTER THREE FLUID MECHANICS |LUID MECHANICS
82 Total Hydrostatic Force on Surfaces & HYDRAUTICS & HYDRAULICS '"^:J5:#'j.:: 83
Total Hydrostatic Forc€

4. A btttttess dan consists of a wall, or face, suPPorted by several


buttresses on the downstream side. The vast majority of buttress
,dams are macle of concrete that is reilforcecl with steel. Buttresses are
typically spaced across the dam site every 5 to 30 m (20 to 100 ft),
depending upon the size and clesign of the dam. Buttress dams are
sometimes called hollow dams beciuse the buttl:esses do not form a
solid wall stretcl-ring across a river valley.

Figure 3 - 4: Gravity dam

Nn entbankment dam is a gravity dam formed out of loose rock,


earth, or a combination of these materials. The upstream and
downstream slopes of ernbankment dams are flatter than those of
concrete gravity dams. In essence, they more closely match the
natural slope of a pile of rocks or earth

Arch danrs are concrete or lnasonry structures that curve upstream into
a reservoir, stretching from one wall of a river czrnyon to the other. This
design, based on tl're same principles as the architectural arch and vault,
Figure 3 - 5: Buttress dam
transfers some water pressure onto the walls of thg canyon. Arch dams
require a relatively narrow river canyon with solid rock walls capable
of withstanding a significant amount of horizontal thrust. These dams
do not need to be as massive as gravity dams because the canyon walls
carry parfof the pressure exerted by the reservoir

Figure 3 - 7: Multiple arch darn


Figure 3 - 5: Arch dam
CHAPTER THREE
84 Total Hydrostatic Force on Surfaces
FLUID MECHANICS FLUID MECHANICS CHAPTER THREE
& HYDRAULICS & HYDRAULICS Totat Hydrostatic Force on Surfaces 85
ANALYSIS OF GRAVITY DAM
A. Vertical forces
A dam is subjected to hydrostatic forces due to water which is raised
upstream side. These forces cause the dam to sride horizontaily
on its 1. Weight of the dam
on its W't:l,Vt Wz=f,Vz; Wz=l,Vz
foundation and overfurn it about its downstream edge or toe.
tendencies are resisted by friction on the base of the
These 2. Weight of water in the upstream side (if any)
dari and gravitational Ws = yVs
forces which causes a moment opposite to the overturning
moment. The dam 3. Weight or permanent structures on the dam
rnay also be prevented from sliding by keying its base
4. Hydrostatic Uplift
Ut = f Vu't
Upstream Sjde Downstream Side Uz=fV,z
(Tailwater)
B. Horizontal Force
Headwater
I
1. Total Hydrostatic Force acting at the vertical projection
t' of the submerged portion of the dam,
I
I -t
p=yE,l
I
I 2. Wind pressure
I
I 3. Wave Action
I
I 4. Floating Bodies
I
I
I
5. Earthquake Load
I
I

IIl. Solve for the Reaction


A. Vertical Reaction, R,,
P =sE
t\y -
Lr t,
Rv = W, + Wz + Wz + Wq - L)t - l)z
Uplift Pressure
Diagram
B. Horizontal Reaction, R.
R* = IFr,
R'=P
IV. Moment about the Toe
A. Righting Moment, RM (rotntibn toroards tlrc ttpstrcnm side)
Figure 3 - 8: Tvpicar section of a gravity dam showing ,n"Torlo,"
forces acting
RM = Wt xt + Wz xz + Wt xj + Wa x1

Steps of Solution B. Overturning Moment, OM (rolatton totoards the downstreant sid.e)


with r.eference to Figure 3 - 8,'for purposes of illustration,
an assumptiori was OM=PU+Lftzt+l-Izzz
made in the shape of the uplift pressure diagram.
L Consider 1 unit (1 m) length of dam (perpendicu.lar to the sketch)
V Location of R, ( t )

ll Determine all the forces acting:


CHAPTER THREE FLUID MECHANICS
86 Total Hydrostatic Force on Surfaces & HYDRAULICS
FLUID MECHANICS
& HY,DRAULICS
CHAPTER THREE
Total Hydrostatic Force on Surfaces 87,
where: R,r 6R,e
y = unit weight of water = 9.81 kN/m3 (or 1000 kg/m.) a=-:
,B -82
y, = unit weight of concrete
y, = 2.4y (usually taken as 23.5 kN/m)

Factors of Safety ,= +(t-#),where e<8/6 Eq.3-14


Factor of safety against sliding, FS_s:

.Note: Use (+) to get the stress at point where R, is nearest. In the diagram
uR.. thown above, use (+) to get qr and (-) to get qH. A negative stress indicates
=
FS.-R, Y >'l Eq.3 -1.2 pressive stress and a positive stress indicates tensile stress

soil cannot carry any tensile stress, the result of Eq. 3 - 14 is invalid if the
Factor of safety against overturning, Fso: is positive. This will happen if e > 8/6. Should this happen, Eq. 3 - 15
be used.

,to= Eq.3-13
Hr,
where:
p = coefficient of friction between the base of the dam and the foundation e>B/6

Foundation Pressure Third


i =a/3
r Middle I

a=3V
For e 38/6 Bl3 lBt3i Blt
From combined axial and bending R, = 1/z(a)(q,)(1)
stress formula: R, = 1/z(37-)q,
n=_!r!!
'AI
Heel
P=R, 2R,v
A=B(L)=B QH qe= 4 Eq.3-15
M= R,,e '" 3i
1(B)3
1 =
12
c =.82
1m cg{ oR,
Ru $,, e)(B /2)
B ' 83/12
O=-r+
'
CHAPTER THREE
88 Total Hydrostatic Force on Surfaces
FLUID MECHANICS FLUID MEcHANtcs CHA1flE\THREE 89
& HYDRAULICS

BUOYANCY where:
y_ = unit weight of the
fluid
ARCHIMEDES' PRINCIPLE vp = volumJ displacecl. Volume of the bocly belor,v fhe Iiquid surface
A principle discovered by the Greek scientist Archimedes that states that "any
h<tdy immersedin a fluid is acted upon by an upward force (buoynnt force) equal to the solae problenrs in hrcynncy, tderirfu tlrc forces n,.lirtg ortr! +ypl!/ conditions of stntic
weight of the displaced ilibium:
fluid".
I,Fs = I
This principle, also known as the law of hydrostatics, applies to both floating lFy=6
and submerged bodies, and to all fluids. LM= 0

Consider the body shown in Figure 3 - 9 immersed in a fluid of unit weight homogeneous solid body of volume V "floa$ng,,6 homogeneous fluid at
y. u
The horizontal components of the force acting on the body are all in
equilibrium, since the vertical projection of the body in opposite sides is the
same. The upper face of the body is subject to a vertical downward force sP 8r olbody-
y, = luoay Eq.3 -17
which is equal to the weight of the fluid above it, and the lower face is subject
liquid = Yrquirr U
V
sp. gr. of
to an upward force equal to the weight of real or: imaginary liquid above it.
The net upward force acting on the body is the buoyant force.

octional area such as


the body of height.H has a constant horizontal cross-9e
cylinders, blocks, etc.:

Vo=Volz-Volr

sP'gr'olbod.y_
,=
sp.gr.of liquid
H= lboay ,
--
Eq.3-18
yhquid
Figure 3 - 9: Forces acting on a submerged body

BF = Fvz- Fvt
the body is of uniform vertical cross-sectional area A, area submerged A.
= yffolz) - y(Voh) rhe
BF=y(Vol2-Volr)

:' , BF=yVo , gq.3-te a - sP.gr.ofbody -n^


,-^s---n== Tbody Eq.3 - 19
A
sp. gr. of liquid l uquid
CHAPTER THREE
90 Total Hydrostatic Force on Surfaces
FTUID MECHANICS
& HYDRAULICS
FTUID MECHANICS CHAPTER THREE
& HYDRAULICS Total Hydrostatic Force on Surfaces V I
STATICAL STABILIW OF FLOATING BODIES

A floating body is acted upon by two equal opposing forces. These are, the
body's weight w (acting at its center of gravity) and its buoyant force BF
(acting at the center of buoyancy that is located at the ce$pr of gravity of the
displaced liquid)

when these forces are collinear as shown in Figure 3 - 10 (a), it floats rr.r an
upright position. However, when the body tilts due to wind or wave action,
the center of buoyancy shifts to its new position as shown in Figure 3 - 10 (b) Bo.{
Y
and the two forces, which are no longer collinear, produces a couple equal to
w(r). The body will not overturn if this couple makes the body rotate towards
its original position as shown in Figure 3 - 10 fb), ancl will overturn if the
situation is as shown in Figure 3 - 10 (c).

rhe point of intersection between the axis of the body and the line of action of Figure 3 - 10 (c): Unstable position
the buoyant force is called the metacenter. The distance from the metacenter
(1w)_to the center of gravity (G) of the body is called the metacentric height Figure 3 - 1O: Forces on a floating body
(MG). It can be seen that a body is stable if M is above G as shown in Figure 3
10 (b), and unstable if M is below G as shown in Figure 3 - 10 (c) tf M
MOMENT AND OVERTURNING MOMENT
coiniides with G, the body is said to be iust stnble
Wedge of
Immersion

OF A FLOATING BODY:
Wedge of
emersion
W = weight of the body
BF = buoyant force (always equal to W for a floating bocly)
G = center of gravity of the body
Bo = center of buoyancy in the upright position
(centroid of the displaced liquid)
,
!o' = center of buoyancy in the tilted position
YD = volume dispkiced
M = metacenter, lhe point of intersection between the line of action
of the buoyant force and the axis of the body
c = center of gravity of the wedges (imrnersion and emersion)
s = horizontal distance between the cg,s of the wedges
u = volume of the wedge of immersion
Figure 3 - 10 (a): Upright position - 10 (b): Stable pbsition
Figure 3 ,0 = angle of tilting
MBo = distance from M to Bo
GBo = distance from G to Bo
MG = metacentric height, distance from M to G
i!

CHAPTER THREE FLUID MECHANICS


92 Total Hydrostatic Force on Surfaces & HYDRAULICS
FLUID MECHANICS
E HYDRAUTICS
CHAPTER THREE
fotal Hydrostatic Force on Surfaces 93
Metacentricheight, MG: MB"+ GB" Eq.3-2'l Moment due to shifting of BF = moment
due to shifting of wedge
BF(z)=F(s)
Use (-) rf G is above Bo
Use (+1 if G is below Bo
BF=yVo
F=lz)
Notet M ts alwavs above 8" z=MBosin0

fVoMBosin0=yas

VALUE OF MB"
l-he stability of the body depencis on the amount of the rrghtrng momenl
which in turn ls dependent on the metacentric height MG. when the'body tilts,
the center of buoyancy shifts to a new position (Bo'). This shifting also causes
the weclge o' to shift to a new position o The moment due to the shifting of
the buoyant force BF(z) is must equal to moment due to wedge shift F(s) vAtuE oF MBo
small values of 0, (0 . 0 or 0 = 0):

Volume of

Pitchtng

Wedge, volume = v

Figure 3 - 1l: Rectangular body


Wirterline Section
a body in the shape of a rectangular parallelepiped
inFigure3-11; length L as

Volume of wedge,'u = Vz(B 2)t@


/ /2) tan OJt
Volume of wedge, o =
+LB2tan e

For small values of 0, s. I B


CHAPTER THREE FLUID MECHANICS
94 Total Hydrostatic Force on Surfaces & HYDRAULICS
FLUID MECHANICS
& HYDRAULICS
CHAPTER THREE
Total Hydrostatic Force on Surfaces 95
US
MB,=
V, sin 0

MBn= But for small values of 0, sin 0 * tan 0


Vp sin 0

MB" = #rs'
'vn
But $ tar t the moment of inertiaof the waterline section, 1

MB"= Eq,,3-23
*
Note: This formula can be applied to any section.

since the metacentric height MG is dependent with MB", the stability of a Centroid of wedge
floating body therefore depends on the moment of inertia of the waterline.
section. It can also be seen that the body is more stable in pitching than in MB.= ot
rolling because the moment of inertia in pitching is greater than that in rolling. Vp sin0
Vo = BDL where L is the length perpendicular to the figure
a= Vz(B / z)t(B / 2) tan }lL
a= f LBztan0
MOMENT
The righting or overturning moment on a floating body is:
Centroid of hiangle, i
From geome try,i = xl + x2 + x3
J
R.Azf or OM = W x = W (MG sin9 3 *24
-: 9* (B/2)sec0+(B/Z)cos}
3

* = B ( 1 *.ore)= 4fr+tos2e)
6\cos0 / dl cosO )
s - s(t+.or2e)
2
-=Y=_tl
6 | coso )
n(
s=_t_l t+cor2 e)
3[ coso )
CHAPTER THREE
96 Total Hydrostatic Force on Surfaces
FLUID MECHANICS FTUID MECHAN'CS
CHAPTER THREE
& HYDRAULICS & HYDRAUIICS Total Hydrostatic Force on Surfaces 97

MB,=
(BDL) sin 0
LB'3 sin0 1+ cos2 e Projection of
: 24 cosO cosO curved
^/R BDL sin e internal pressure and this is to be
sted by T which is the total skess of
MB-= B' l+cos2o pipe wall.
24D e
".r.2
MB,,--
B' l,-a-.r) Applying equilibrium condition;
'J [>Fs = o]
24D l..or2 e F=27
,U"= !-(sec20 p= pA=pDS
24D'
+ 1; butbec2e=1+tan2e
. T = SrA*arr

,r" = !-[ "(1 + tan2 e) + U I=Sr(sxf)


24D pDs=2x[Sr(sxf)]
MB, =
B' (2 + Bt * tu"t e )
12(2\D tan2e) = .t2D l?\2 2 |

MB^=
B' [, * tut" e) Eq'3-25
12Dt 2 ) determine the longitudinal stress, let us
the cylinder across its length as shown.
lEFs = ol
F=T
F=pA
STRESS ON THIN.WALLED PRESSURE VESSELS F=p f,Dz
T= S1 A*";1
THIN-WAILED CYLINDRICAL TAN K
A tank or pipe carrying a fluid or gas under a pressure is subjected to tensile , Anut= fiDt
forces, which resist bursting, developed across longifudinal and transverse
T = StrDt
sections.
p +Dr=SrrDt

lst
Sr *-fit-t'
\ s.
CHAPTER THREE FLUID MECHANICS FIUID MECHANrcS
98 Total Hydrostatic Force on Surfaces & HYDRAULICS ! HYDRAULICS
CHAPTER THREE
Total Hydrostatic Force on Surfaces

SPHERICAL SHELL
lf a spherical tank of diameter D ancl thickness I contains gas under a pressure
of p, the stress at the wall can be expressed as:
3-1
vertical rectangular plane of height d and base b is submerged,in a liquid
its top edge at the liquid surface. Determine the total force F acting on
side and its location from the liquid surface.

p=yhA
walsrress, s=+4t Eq.z-29 i=a1z
A=bd
F = y(d/z)(bd)
F =lz"y b iP
SPACING OF HOOPS OF A WOOD STAVE PIPE
I.
Ai
' v =i =a1z
*ua3
LZ

(bd)(d / 2)
e=d/6
Pressure diagram

Yr= i *,
(triangular prism)
yP= d/2+ d/6
Vp= 2iU3

Using the pressure diagram:

f ?t#
spacing, Eq.3- 3o
F = Volume of pressure diagram
F =1/z(Yd)(d)(b) =1/2yb dz

where: The location of F is at the centroid of the pressure diagram.


Si : allowable tensile stress of the hoop
For rectangular surface (inclined or vertical) submerged in a fluid with top edge
A1, = cross-s€ctional area of the hoop
flushed on the liquid surface,,the center of pressure from the bottom is I/3 of its
p = internal pressure in the pipe height.
D = diameter of the pipe
I oo ftril;l.H[.'[ FLUID MECHANICS FLUID MECHANICS
Force on surraces & HYDRAUTICS 6. HYDTIAULICS
CHAPTER THREE
Total Hydrostatic Force on Surfaces tot
Problem 3 - 2 Solution
A vertical triangular surface of height d and horizontal base width b is
sul'rmet'ged in a liquid with its vertex at the liquid surface. Determine the total
r=via
F = y(r)(rc rz)
forc* f acting on one side a'd its locafion from the liquid surface.
F=nyF
Solution
t.
F=yh A
6

i = +a AV
1 _-.4
A = l/zbd l p= !!__ =y14
(nr')(r\
F=y*3dx1/zbd
yp=r+e
F -- | ybdz yr= r + rf4
yn = 5r/4
I
5
a=
Ai Using the pressure diagram for this case
is quiet complicated, with the
showrl its volume
n= i = za/s m is easy to use only if"u.,
b" computed by in'tegration. Hence, pressure
the area is rectangular, with one side horizontal.
!ua3
('-- -1b

Ghd)(zd / 3)
e = d/12
3-4
Pressure diagram
vertical rectangurar gate 1.5 m wide and 3
(pyramid) m high is submerged ih water
ry,=l-r TJ'",T: :,..,n
one side of :i"l::."q:
the igate .,,,,ruce. .
-y;r:u, from
and its location
Fini u," t.iJ
the bottom
;;:;," u. ti., g
vr= 1d+d/12=3d/4
Using the pressure diagram.
f = Volume of pressure diagram r=yia
F= +Ar,","^ height i =t.S+2=3.5m
f'= {tt'"yd\et\= tryua, F = e.81(3.5)[(1.bX3)]
F = 154.51 kN
f rs located at the centroid of the diagram, which is % of the altitude
from tlre base
t"6
e-
AV
Problem 3 - 3
e= = 0.2L4 m
A vertical circular gate or radius r is submerged in a liquid with itd top edgerl (1.5 x 3)(3.s)
flushed on the liquid sru'face. Determine the magnitude and. Iocation of tlrt, 1.5 m
Y=1.5-e
total force acting on one side of the gate y=1.5-0.214
y =1.286m
t 02 ;i*i;;H:'i::l: Force on surraces FLUID MECHANICS
& HYDRAULICS
FTUID MECHANICS
& HYDRAULICS
CHAPTER THREE
Totat Hydrostatic Force #";;.; I 03
Using the pressure diagram: Solution
F = Volume of pressure diagram
r=yiA
F = (5Y !2Y ,. e) 1r.sy h =2+ *fsl
l\-/
F = 1,5.75y
'' i =zm= g
F = 1s.7s(e.81)
: [e.81(0.82)] (3)t%(1.5X3) J

F = 154.51 kN F = 54.3 kN

l" *(i.sxs)3
Ay t+(r.sxs)l(a)
e = 0.1,67 m
yr=i +e
y,= 3JL57 m from the oil surface

3 - 6 (CE Board May 1994)


Pressure diagram
(trapezoidbl prism)
Location of F:
h=2y(3)=6y
Az=lz(3y)(3): a.5y
A = At + Az= 1,0.5y

[Ay = zay]
1,0.5y y = 6y(1.5) + a.5y(1) Fo.ce on upper half:
y =7'286m (much complicated to get than using the formula) Fs=y,i A
Fo = (y,, x 0.8)(d/ +)[b(d / z))
Fo=0.1y,,bd3
Problem 3 - 5 Force on lower half:
A vertical triangular gate with top base horizontal and 1.5 wide is 3 m high. It Fw= pcgzx A
is submerged in oil having sp. gr. of 0.82 with its top base submerged to a Pcg2=fo14* Yruh,,,
depth of 2 m. Determine the magnitude and location of the total hydrostatic
P,e2 = (f ,u x 0.8)(d/2) + y",(d/ 4)
pressure acting on one side of the gate.
P,sz = 0.65 Y, d
F6, = (0.65 y,,, d)[b(d/ 2)]
Fw = 0.325 Tu b d2

Fw
Ratio =
Fo

o'225v,b72
Ratio = = s.zs
0.1,y,ubd2
FLUID MECHANICS
l04 CHAPTER THREE
Total Hydrostatic Force on Surfaces
FLUID MECHANICS
& HYDRAULICS q HYDRAUL'CS
CHAPTER THREE
Total Hydrostatic Force on Surfaces r05
Problem 3 - 7 (CE Board May 1994) Problem 3 - 8 (CE Board May 1992)
A vertical circular gate in a tunn€l 8 m in diameter has oil (sp. g. 0.8) on one A closed cylindricar tank 2 m in diameter and g
m deep with axis verticar
side and air on the other side. If oil is 12 m above the invert and the air contains 6 m deep of o'(sp gr.= 0.g) The
air above theiiquid surface has a
pressure is 40 kPa, where will a single support be located (above the invert of pressure of 0.8 kg/cm2. Determine the total
norrirar force in tg on the
the tunnel) to hold the gate in position? wall at its location from the bottom of the tank. ".u"g

Solution Solution

tt
^^
oir;s=o'8
I I
I'i
12m Air;P=46gPu ,l"l
tlI .i -,- *
I

* -'*qt -
F.i,
-.-.-.-.-8m@
4-y
4m

Fe1 =yq1; hA
Ft=Pu*A
F"rr = (9.81 x 0.80)(8) *f (8),
Pair = 0.8 kg/ cmz = 8,000 kg/ m,
F.l = 3,156 kN Fr = 8,000(2n x 2) = 32,000n kg
I- h= 6 +'1, =7 m
e= E

AV Fz= prcA
+(8)4 p., = (1000 x 0.8)(3) + 8,000
e= b4\_, =0.5m p,e= 10,400kg/m,
f (8)'(8) Fz ='1,0,400(2r x 6) ='124,800n kg
z=4-e=3.5m
Solve for e:
Fair=pauA,=40t t(8), F2=y.i A
F"i' = 2,011 kN 724,800n = (1000 x 0.8) /i (2n
" 61

The support must be located at point O where the moment due to Fuo h = v =13m
and Foir is zero. Since Fou ) Fai,, O must be below F"l.
-_ I, _ $12n11013
[>Mo = o] AV (2n x 6)(13)
Foir(z-!)=F^t(4-y) b = 0.23077 m
(3,1s6X3.5 - y) = 2,011.(4 - y)
! yz=3-e=217m
1.s6e(3.s -y)=4-V
5.493-1'.569Y=4-Y F = Fr + Fz = 155,8002r
=2'52m kg ) Total normal force
V
u
I

t 06 ;i.:il;:.T*:l: Force on surraces FLUID MECHANICS FLUID MECHANICS . CHAPTER THREE


& HYDRAULICS & HYDRAUTICS Total Hydrostatic Force on Surfaces t07
Fy=Fryt+Fzy) Solution
(156,800n) y = (32,000n) (n + 924,800n) (2.77)
[XMr,i"e"=0]
3t = 3.63 m ) Location of F from the bottom Fz=a}e)
Using the pressure diagram: r=yhA=9.8th (1)(1.5)
F =M.7tI5h
Ir
(--- ---:= Wnefe l/ =i
Ay

e= #(r.s)(r)3 1

(1.5 x 1)/r 12h


6

z=0.5+e=0.5+ 1

12h
800(6) = 4800 8000
1 \
Pressure Diagram -/
't4.7'L5h lo.s+_]:l=+o
\ ) 1.2h
Pl = 8000(8)(2n) = 128,000n kg 0.5i :2.718
+ 0.08333
P2= 1/2,(4,800)(6)(2n) = 28,800n kg i =S.ZZm= l, +0.5= 5.77m ) critical water depth
P = Pr + P2= 156,800nkg ) Total normal force

IP y: Pt yt + Pzyzf 3-10
(156,800n) y = (128,000n)( ) + (28,800n)(2) vertical circular gate is submerged in a liquid
so that its top edge is flushed
1/ = 3.63 m ) Location of P from the bottom th the liquid surface. Find thJ rafio of the total
force acting on the lower
to that acting on the upper half.

Problem 3 - 9
In the figure shown, stop B will
break if the force on it reaches
Rauo = I
F1 0.5756r
40 kN. Find the critical water ("\
depth. The length of the gate Ratio = x-.
perpendicular to the sketch is "34a
yht A,t x
1.5 m At=Az x=4r/
i
Ratio = I
h1
; ..
r(ano= 1..424r
=2.42s
LS7S6,
I oB ;Hi;;:#[.'i: Force on surraces FLUID MECHANICS
& HYDRAULICS
FLUID MECHANICS
& HYDRAULICS CHAPTER THREE
I otal Hydrostatic Force on Surfaces t09
Problem 3 - 11 Solution
A 30 m long dam retains 9 m of L=30m
water as shown in the figure. Find p=yi A
the total resultant force acting on the
dam and the location of the clnter of i =z.s+21s
pressure from the bottom. h = 4.767 m
A = %(1)(2.61)
A = 1.305 m2
| = (?s_t!" 0.ss)(4 ftn (t .305)
L = M,277N
SoluUon F =44.277kN
p=yhA
F = e.81 (4.s) [(30)(10.3e2)]
F = L3,753 kN
L=30m 3-13
I^ inclined, circular
e= " with water on one
Ay
is shown in the
Determine the
(30- x 10.392)(4.5 / sin 60')
resultant force 2m
I
on the gate.
e =1.732m

!=lz(10.392)-1.j32 o
o
y = 3.464ln o
o

y= +(10.3e2)=3.464m

Problem 3 - 12
The isosceles hiangle gate shown
in the figure is hinged at A and
.yEA
weighs 1500 N. What is the total h =2+ 0.5sin60.
hydrostatic forceacting on one side h = 2.433
of fhe gate in kiloNewton? .9.81,(2.433)t
$),
18.746 kN
Oi 1s,=rg.8a),;
I to CHAPTER THREE
Total Hydrostatic Force on Surfaces
FLUID MECHANICS
& HYDRAULICS
FLUID MECHANICS
& HYDRAULICS
CHAPTER THREE
fotal Hydrostatic Force on Surfaces ltl
Problem 3 - 14 ts #(1.5X3.6)3
'llrr. (b)
in tlre figure shown is 1.5 m wide, hinged at point A, and rests
1',ate A, (1.5x3.6)(7.21)
ag,ainst a smooth wall at B. Compute (a) the total force on the gate due to
e=0.15m
scawater, (b) the reaction at B, and (c) the reaction at hinge A. Neglect tl-re
x=1.8-0.15
wcight of the gate.
x=1.65m
[>Ma - 0]
F(r)-Rr(2)=0
218.25(1.65) = 2 RB
Ra = 180 kN
Seawater
s = 1.03
[:Fs = 0]
l

I
o Rar,+Fsin0-Rs=Q
I
I o
1

J5m
rrr 9 Rar, = L80 - 278.25 sin 33'69'
I
tgp" ) Rar, = 58.94 kN
.
[I F,, = 0]
Rau-Pcos0:0
Ra, = 218.25 cos 33.69"
Ra" = 181.6 kN

t*Ro, (1S1.6)2 + (58.94)2

Solution

Determine the magnitude i4fiiqr:i?:kle:


d2=32+22
d=3.6rn lnd location of the total m
hydrostatic force acting on
tan0 = 2/3 the2mx4mgateshown 1.5 m Water
0 = 33.69" ln the figure.

1l =-
i
" sin0 m

*4
l/ =
sin 33.69'
J = 7.21 rn

-
r=yi n
W3%/
F: (e.81 x 1.03)(a)[(1.sX3.6)]
F = 218.25 kN
\

Ia tI G
a CHAPTERTHREE FLUID MECHANICS FTUID MECHANICS
CHAPTER THREE
Total Hydrostatic Force on Surfaces & HYDRAULICS & HYDRAULICS Total Hydrostatic Force on Surfaces I t3
Solution Problem 3 - 16 (CE Novembe1t997)
Determine the magnitude of
Oil, s
the force on the inclined gate
m = 0.80
1,5 m by 0.5 m shown in the

1.5 m Wdter 001. The tank of


lrrater is completely closed
fnd the pressure gage at the Figure 001
m
of the tank reads
90,000 N/mz. Use 9,300
/cu. m. for water.

F= p,tA
Pcg= IYh + P
p,s= (9.8LxL 26)(3) + (e 81)(1.5) + (9.81x0.80)(1) + 32
P,g:9-l'645 kPa
f=91.645(2x4)
F = 733.16 kN F=prrA
Pz-p,s=ylt
Solving for e:
90000 - p,g= 9,800(2.G5)
Solve for i and y : p"g= 64030Pa
r=yi a F = 64030 (0.5 x 1.5)
F = 48,022.5 N
733;1.6 = (9.81x1.26)h Qx a)
i =7.414m
v =n /sin60"=7.4'14f sin60"
7 = 8'561m
e I #Q)$)u
Av (2 x a)(8.561)
e = 0.156 m -L7
z=2-e=1.844m
9l,T.tT*" in the figure is hinged at A and rests on a smooth floor at B.
g,,or 0.s2,;;;;o
Therefore, F is located 1.844 from the bottom of the gate. ,t"1t:r;:#"'-T":*fl1*lt thelii1fJ;f*";;;r#;;;r#'"'ii'uo"
?t_f"i.:"r
re the hinge A. rhe air auove "i"ish, "r, s
If the gate weighs 5 krrr, determin" .h"';;;;;"rir." .
,1T":l*l:
to open it.
CHAPTER THREE FLUID MECHANICS FLUID MECHANICS CHAPTER THREE
I I+ Total Hydrostatic Force on Surfaces & HYDRAULICS & HYDTTAULICS Total Hydrostatic Force on Surfaces I t5
Prcblem 3 - 18 (CE Board)
n pinp 20 mm in diameter are used for supporting flashboards at the crest
masdnry dams. Tests show that the yield point of iron to be 310 Mpa
-:- fiber st'ess). Neglecting the dynamic effect of water on flashboards
cl assuming static conditions, what is the proper spacing, S, of the iron
pins,
that the flashboards 600 mm high will yield when wateiflows 150 mm deep
the top of the flashboards.

acl
F
""ffi
Floor' B

Solution
P = p,rA h = 0.45

Pts= Pau + t}Jtu


p,s : 7 + 9.81(0.S2)(2.56)
p,r = 27.59 kPa
P = 27.ss t(3X3)l
P = 248.34 kN
1.5 sin 45'
p=yhA = 1.06 m

248.34= (9.81x0.82)[ (a. S)


l, =l+g.t't
h 3.43
sin 45'
V
sin 45" rment capacity of one iron pin (20 mn A\
i =485m
lFa= Mc/Il
I
E_ # (3X3)'' 310 =
At (3 3)(4.85)
" &Qq4
r = 0.155 m M=243,473.43 N-mm
r='l .5+a M= 0.24347kN-m
r = 1.655 nr
Itrtent caused by F (considering S m width of flashboard):
I'MA = 0l Mra'Fxy
P(r) + W(1.06)- f (2.12) = 0
2.72F = 248.34(1 655) + 511 s61
P=yhA whereA=0.6S
F = 9.81(0.45)[0.6 S]
r = 195.37 kN
I I.
I IO
CHAPTERTHREE
Total Hydrostatic Force on surfaces
FLUID MECHANICS
& HYDRAULICS
FLUID MECHANICS
& HYDRAULICS
CHAPTER THREE
Total Hydrostatic Force on Surfaces tl7
| -- 2.6495 Problem 3 - 20
v=03-,' ,At 20 'C, gage A in the figure reads 290 kPa absolute. The tank is 2 m wide
' perpendicular to the figure. Assume atmospheric pressure to be 1 bar. Sp. gr.
,,=', - #(sxo'6)j
Av (0 s)(0 4s) 6 ' of mercury = 13.6. Determine the total pressure acting on side CD.

Mt=f ,y=M
e=0067m.
y=03 -0067 =0233m r-
1m
.l,

2.6495"0.233=0.24347
1
S=0394m=394mm h

Problem 3 - 19 70 cm
l-he semi-circular gate shown
rn Figure 28 ts hinged at B Gage A

Determtne the iorce F required


to hold the gate rn Position
for h:
Solution 4ft
pt=2yhtptop
i =f
i =,t
= lo-l6elt L 290 = (e.81 x 13.6)(0.70) + (9.81)h
h=2.2m
+ 17s

=B3o2fl
force on side CD: (Note: 1 bar = 100 kPa)
1:=yh A
2l
P = 62.4(8.302)lt/znl4l r 175 - 100
I'] = 13,01 9 89 lbs
r 75 kPa l-
,=L
nt n \.
r 9.81(2.9)
:28.449kPa
1m
l
t

lI
lx=01098'I IE
2.2

l_
ldt
ir = 0 1098(4)' 'Pr(3.e)(2)
.75(3.e)(2\ E r"i
/' = 28 11 fto
4ft r 585 kN
qr
NI
28.11
=-
\ "(4\' (8 302) l_ tr/zpz(2.9)(2) 0.7 m
tVz(28.aa\(2.e)(2)
,'= itzsz t,
h -- 1 698 - O 1347 = 1 5633 fl r E2.5 kN 2m
Fr+Fz
[IMr, = 0l
P(b) = r(4) 667.5 kN
13,019.89(i 5633) = t(4)
t = 5088.5|bs
r 18
CHAPTER THREE
Total Hydrostatic Force on Surfaces
FLUID MECHANICS
& HYDRAULICS
TLUID MECHANIES
HYDRAULICS
CHAPTER THREE
Total Hydrosiatic Force on Surfaces ll9
Problem 3'21
I'he funnel shown in the figure is F=yhA
full of water. The volume of the F=9.81(2.6X1.6x1.2)
upper part is 90 liters and the F = 48.97 kN
lower part is 74 liters. What is the I"
force tending to Push the Plug
e= --L
AY
t'lu t? 1.2(1..6)3

g= '12 '

(1.6x1.2)(2.6)
1.6 m
a = 0.082 m
7=0.8-e
z = 0.718 m
= 460 cm2
T=Fxz
totlrl.lll T*48.97x0.718
T = 35.16 kN-m
rn" plug area in contact with water is horizontal, the pressure all over
tht'
it is uniform. The shape of the container does not affect the pressure on
Plug 3- 23 (CE Board)
Force=p^A
Force = e,810(3XiS )
box, 1.5 m on each edge, has its base horizontal and is half-filled
water. The remainder of the box is filled with air under a gage pressure
Force = L353.78 N kPa. One of he vertical sides is hinged at the top and is free to swing
To what depth can the top of this box be submerged in an open body
water without allowing any wate-r to enter?
Problem 3'22
ln the figure shown, the gate AB
rotates about an axis through B
The gate width is 1.2 meters. A
torque T is applied to the shaft
through B. Determine the torque T
to keep the gate closed

-T I

1.5'm

0.25
9.81(0.75) 82 kPa
= 7.36 kPa
1.5mx1.5m
,l

. -A CHAPTER THREE FLUID MECHANICS FLUID MECHAN'CS


I ZU rotal Hydrostatic Force on surfaces CHAPTER THRFF
& HYDRAULICS & HYDRAULICS rotar Hydrostatic rorclPJf:#fi: I Zl
[I Mh'^g" = 0l Ftoblem 3 - 24
Fr (r)- Fr (0.75)- Fz(1.2S; = s ) Eq. (1) the magnitude and location of
Ft = Pn,A force exerted by water on one
F, = 82[(1.5)(1.s)l = 184.5 kN of the vertical annular disk
n. 4m
Fz = Vz(7 .361 (0. 75) (1 .5)
F: = 4.14 kN
- =1.5m

Pr=Yh n l_
Fr = e.81 ii ttr.sxr.s)l
Ft = 22.07i

t = 075 + c
l'5(1'5'l p=yi.A
t"o_ 1.2 F = e.81,$)[n(1.5)'- n(l)z]
F = 154.1 kN
I
AV t(15X15)1l?
0.'t875
P=: Location of F:
h

r=0.75+ --:'--
0.1875
- Av
,= 'r - f(1.s)4 -f(1)4
h d(Gf11)r(4)
tn Equation (1):
e = 0.203 m
0'187s = 4 + 0.203 = 4.203 m below the
22.07i (OZS+
'Ir ) - 184.s(0.75) - 4 14(1.25) = 0 Ap w.s.

16.55t + 4.138 - 138.375 - 5.'t75 = 0


16.55 t =:.3.s412
i =s.szm ,gate in the figure shown
5 kN for each meter
h= h -0.75 : to the paper. Its center of
h=7.67m is 0.5 m from the left face
0,6 m above the lower face.
h for the gate just to come
the vertical position.
| ,1 CHAPTER THREE FTUID MECHANICS FLUID MECHANICS
I zz CHAPTER THREE
Total Hydrostatic Force on Surfaces 6. HYDRAULICS & HYDRAULICS Total Hydrostatic Force on Surfaces t23
Solution ,Solution .

Considering 1 m length dF=pdA


F'r=Vz (9.81/,XhX1) p=vy
Fr = 4.905 h2 kN 0.6 I
el dA* 2x dy
F, :9.S1h(1.sX1)
Fz = 14.715h kN h fw=srr,t By squared property of parabola:

[tMo = o]
cg *'=2'
F{h / 3) + w (0.6) - Fz(1.5 / 2) = 0 vq
4.e05t* (h/3) + s(0.6) - 14.71sh (0.75) = 0
1..6351f -17.04h + 3 = 0
9.81h
9.81 h.
x2= ty
x - 2Jin
Solve h by trial and error
h= 0.2748m dF = yy 12 eJi /3 ) dvj
dF = 2.31yy3/2 dy
F3
Problem 3 - 26
In Problem 3 - 25, find h when the force against the ,'stop,, is a maximum.
p, =221y [r',,n,
00

'liV,!l'|
Solution
l
F = 2.31y = 2.31(9.81)t lzsrz - osrzl
[:Mo = 0]
F{h/ 3) + w (0.6) + p(1.5)- pz(1.5 / 2) = 0 L" Jo
4.e05t* (h/3) + s(0.6) + p(1.s) F = 141.3 kN
-14.775h (0.7s) = 0
P = L.09h3 -7.358h + 3 Location:
3
= 3.27 rp- 7.3s8 = o
# F yp=
IP= 2.25 [,rn,
/ =1.5m 0
3

1.41.3 y,= rttzav)


Problem 3 - 27 _0
[rbtt
Determine the force due to
water .1

acting on one side and its location on yr= 0."1604 lvu/'a,


the parabolic gate shown using J-
0
integration.
y,=0.1504lQtnrr,,
l*--,---- o *" I y, = 0.1604 (217) 137/2 - 07/2 |
)'o

yo= 2.14 m below the w.s.


t 24 ;liil;:.:H:,? ,o,.. on surfaces
FLUID MECHANICS
6( HYDRAULICS
FLUID MECHANICS
& HYDRAUI.ICS
CHAPTER THREE
Total Hydrostatic Force on Surfaces 125
Problem 3 - 28 Problem 3 - 29 (CE Board)
ln the figure shown, find the I d."t", tt triangular in cross-section with
the upstream face vertical. water is
width b of the concrete dam the top. dam is 8 * hth and 6 m wide at the base and
necessary to prevent the dam $f,:O ln1th -The
'weighs 2.4 tons per cubic meter.The coefHcient of friction between the base
from sliding. The specific gravity gmd the foundation is.0s. Determine (a) the maximum
and minimum unit
of concrete is 2.4 and the Pressure on the foundation, and the. (b) iactors of safety against overturning
coefficient of friction between the lnd against sliding.
base of the dam and the
founilation is 0.4. Use 1.5 as the
factor of safety against sliding. Is
the dam also safe from sp.s' of conc, s6on. = Js9!l
overturning ? fw
sp.st of conc, ,.on. = 33.IJ!oo = 2.4
Solution 1000
Consider 1 m length of dam
der L m length of dam
W,--1, V,
w=Yrv
w, = yQ.a)l(bx6)(1)l
W,=1'4.44v = (yx2.4) [+fultrlAlJ
W = 57.6 y where y = unit wt. of water
r=yiA
F: y(2.25)t(a.5)(1)l p-yiA w.s.
F = 10.125y . l.e.- B = 6m
y(4)(8 x 1)
R'=f:10.125Y - 3Zy
Rv=W, -P=321
-14.4
Rv= W -W=57.6.1

-R,
I-sc=
frR, - W(4)
-
- 57,6y(4)
0.4(14.aby\ - 230.4y
' 10.125y - P(8/3)
b=2.637 m - 32y(8/3)
.85.33y
: 444
r.'oM
.,RM-oM B/6 ='l m
Rv
Dc _ wc(b /2)
'
I J^ -
F(i.s) , tr30.4y - 85.33y
=2.519m<B/2
M.aQ.637\y(2.637 /2) .:3.3>1(Safe) 57.6y
FS" =
- 10.12sy(1.s)
t/2- i
l'2,5t9=0.481 m<B16
CHAPTER THREE FLUID MECHANICS FLUID MECHANICS CHAPTER THREE
t26 Total Hydrostatic Force oh Surfaces & HYDRAULICS & HYDRAULICS Total Hydrostatic Force on Surfaces 127
Rr(- 6e\
Figure:
rl= _ jl
, l+-1
B \ B/
Ll = -
57.6(e.87)
I r r
[. o1o.+s11
------:-- I
6L6J I

Using (+). -- - 139.47 kPa


t:1t ) soil pressure at the toe
Using (-)
-
r7H=-48.88kPa ) lsoil .pressure at the heel

ir,lt,, 0.8(57.6y)
F-S,, = " =
R. ?2y
F5,, = 1.44

RM 230.4t
FS'= ona =
85.33y
l'-5. = 2.?

Problem 3 - 30 (CE Board May 1992)


A gravity dam oftraprezoidal cross-section with one face ,vertical and Neglecting hydrostatic uplift
horizontal base is 22 m high and has a thickness of 4 m at the top. Water L Consider 1 m length of dam
upstream stands 2 m below the crest of the dam The specific gravity o(
masonry is 2.4 ll. Forces
A Neglecting hydrostatic uplift: W.r =t, r, = (, x 2.4)I(4)(22)(1)l
1 Fincl the base width B of the ctam so that the resultant force will cut Wt = 211.2y
the extreme edge of the middle third near the toe.
Wz= (y x 2.4)Iv, (B-4)(20)(1)l
Compute the factors of safety against slicling and overturning
Wz= 24By - 96y
Use pt = 0.5.
Consiclering uplift pressure to vary uniformly from full hydrostatrt F=yhA=y(ro)t(20)(r)l
pressure at the heel to zero at the toe: , F = 200to
1 Find the base width B of the clam so that the resultant force will act al lll. Reaction
the extremity of the rniddle third near the toe. &=If,=P
Compute the maximum and.minimum compressive stresses achng ' R, =20Q
against the base of the dam
. Ry=IFv=Wr+Wz
:21"1.2Y+248Y-96y
Rr=248y +115.2y
t28 CHAPTER THREE
Total Hydrostatic Force on Surfaces
TTUID MECHANICS
I HYDRAULICS rotar Hydrost"ti. rc#fJ,E:J,?*:: I Zg
,4m
M.8t (M.q2 - 4$)(-1,4ss.ry
B= -
2(8)
B=r't..'r,7sm
Factors of Safety:
Factor ofsafety against slicling -
FS.=uR.y
R,
(0.s)[24(1 1.1 75)1 + t15.Zy
)
. 200y
F',S. = 0.958s

Factor of
Uplift
lafety against overturnlng
RM
pressure f 5. = -__
diagram
OM
_ t6(71.175)2 y + s3.2(tt.t75)y _ r66.4y

' FS"
-
= 2.A7
1333.33v

lV Monrent about the toe


RM = Wt(B - 2) * W.,t+ (B - 4)l
Considering hydrostatic uplift:

= 211.2y(B - 2) + (248y - e6y) [ 4 tS s))


Uplift force, U = th (Z0y)(B)(1) = t0Bzl

= 277.28y - 422.4y + 168) - 1288y + /56y Rr=Wt+Wz-l)


RM = 1682y + 83.28y - 166.4y = 24BY + 715.2y - 10By
Rr=148y+115.2y
oM = F(20/3)
= 200y(20/3\ RM=w@-2)+wzl4ts-sil
t)M = 133333v RM = 768) + 83.28y - 166.4y

V l.ocatron t-rf R .OM=FQT/s)+U(28/3)


R.,i=RM_OM = 200y(20/3) + 10By (28/3)
OM=6.6782y+1333.33y
Since the resultant force will pass through the extreme edge ol Rrf=RM-OM
the middle thirds near the toe, i B/3 Then. (1.48y + "t't'.zy)(B
= / 3) = 1 6B2y 166.4y - (6 678) + 1333.33y)
4.6682 + 44.88 - 1499.73 =0
l24BY + 11s.2y)(B/3) = l6Bzy + 83.28y - 166 4y 1333.33r
j - 44.8t l(M.q2 _ 49.66)(_14ss.73\
8B2y + 33.49r ='l6B?t + 832h 14997?y 8=
882+44.88-149973 =0 2(4.66\
B = 1.3.766 m
I 30 ;i.:ili:.:H:[ Force on Surfaces
FLUID MECHANICS
& HYDRAULICS
MECHANICS
HVDRAULICS rotarHydrostuti.FTXPJ"EIJ.?*:: I 3I
lioundation stress:
v =B/3
r = 13.766/3 = 4.59 m F =.y,"i A
r=B/2-i =2.2943nt * 9.81(3)(6 x 11
- 2m
F 176.58 kN
,,=
, Ir(rr99l x- {(o)=zm
B\ B] -
Rv = t4$s.206x9.81 ) + I 15.2(9.81 ) W=Y'v''
= 23.s[2(8)(1)l

li
R,r = 3020.73 kN
Wl=376kN
3,020.731 Wz=y,Vt
- ' 1!766 " - 11766
O1Z.ZS+Z1I
'' wl
l'' ) = 23.5[vr(2)(B)(7)] i

qr=-438.87kPa = 188 kN
r7H : 0 kPa
t4-Yz(Z)=i ;

'(2/3)(2\ = 1.333 m
Problem 3 - 31 (CE Board May 2002) ;r=rzo.sstN
Ihe section of a concrete gravity - W-r + Wz = 376 + lgta
dam shown in the figure. The
depth of water at the.upstream side
* 564 kN --*t
4m

is 6 m. Neglect hydrostatic uplift trIi,


and use unit weight of concrete =
R,
equal to 23.5 kN/mt. Coefficient of
0.6(50qt
friction between the base of the =ffi'l.elb
dam and the foundation is 0.6.
Determine the following: (a) factor 'Wtrt+Wzxz
of safety against sliding, (b) the 5 = 376(3) + 188(1.333)
factor of safety against = 1378.604 kN-m
bverturning, and (c) the
overturning moment acting against =F^y
the dam in kN-m = 1.76.58(2)
= 353.16 kN-m ) overturnlng moment
RM
OM
1378.604
" 35316 = 3.eo4
CHAPTER THREE
132 Total Hydrostatic Force on Surfaces
FLUID MECHANICS
6. HYDRAULICS
FIUID MECHANICS
CHAPTER IHREE
& HYDRAUTICS Total Hydrostatic Force on Surfaces t33
Problem 3 - 32 (CE Board November 2OO1) Problem 3 - 33 (CE Board May 1986)
I'lre sectiorr of a gravity dam is
duT
as shown in the figure. Assume I|| n:*:lT:"T::*y
9.81 : fown rhe specific weight of water is
kN/mr and that of concrete is l23.54kN/#';5r:;;;"i,1;:,;rffi;
1s
hydrostatic uplift to vary varies rino,,h, r.^*
linearly from
urriformly from full hydrostatic maximum hydrostatic 7m fl.52m
uplift from the heel to zero at re at the heel to zero al
the toe Determine the total the location of the drain,
reaction per unit length at the ine the (a) location of
base of the dam Use sp. gr of resultant force, (b) factor
concrete = 2 4 safety against sliding if
:fficient of friction is 0.75,
Solution factor of safety against
rning, (d) the stress at
heel and at the toe, and (e)
unit horizontal shearing
Consicler 1-foot length of dam at the base
B,=P=y/rA . 30' ,|. t0' 6,

I
1
= 52.4(30)(60 , l)
I
R, = 112,320 tbs I
I 10
i

Ru= LV1+Wz+Wt-U I
i
\
I
Wt=y,Vt I
I

wt -- (62.4 ^ 2 4) ]q'je 60' I


:;l1lr. i t '
l0
(bo)(t )
I ,,60
I
Wz
wt = 224,640]bs I
-'
,1,
" Wz=l,Vt

4
I
,l, Wg
w.= gz.4 " 2.ay 99'i91oy1r1
W'z = 44,928lbs 60'

Wz = t,Vt = (62,4 x Z.Q vzQal(40\(7\


Wq = 71,884.8|bs
Wq = tu, Vq = (62.4\ vz(30)(60)
Wq = 56,160lbs

U = y^, Vu = (62.4) Yz(60)(70)(1) = 131,040lbs


R,,= 224,640 + 44,928 +71,8848 + 56160 - 131,040
R,,=266,5728Ibs

R = !/ (1 12,320\' , (266,5728\' = 289,269 tbs per foot


i -I ' CHAPTER THREE FLUID MECHANICS FIUID MECHANICS
I 5+ Total Hydrostatic Force on Surfaces & HYDRAULICS
CHAPTER THREE
O HVDRAULICS Total Hydrostatic Force on Surfaces I 5 5
I Consider 1 m length of dam
(c) Fs" = RM - 683,900'"12

I I Forces
oM 377,758
FS,, = 1.81
Wt=y, Vt
Wt = 23.54 [vz(5.2)(52)(1)] - 3,183 tN (d) Foundalion pressure:
w2= /t.$4 t(7X52)(1) = 8,56e kN e=B/2-i
e = 38.2/2 - 13.2 = 5.9 m < B / 6
wt = 23.541v,(26)(52)(1\l = 1s,e]3 kN
Wa = e.81[rz(5xs0x1)] = 1,226.3 kN
u = vz(490.5)(%.2)(1\ = s,6e0 kN
,t=
, Yh
B\
tlel
B/
-]
23,201.3f ots.gl
r = yi A = e.8i (2s)t5o(1)l = 12,263 kN ,.t -- ,*
i, 38.2 L 3s.2 l
lll Reaction
R, = F= 12263 kN Stress at the toe, (use ,,+,,;.
qr = -1;1.70.2't kpa
Rv=Wt+Wz+Wt+Wq-L)
= 3,183 + 8,569 + 15,913 + 7,226.3 - s,690 Stress at the heel, (use ,,-,,)
R'l = 23,201.3 kN qn =.-44.52kPa

lV Moment (e) Unit horizontal shearing stress, S.


RM = W t(34.73) +'vt1r129.5) + Ws(17 .33) + W4(36.53)
= 3,183 (34.7 3) + 8,569 (29.5) +'.r5,91 3 ('17 .33\ + 1,226.3(36.53) S.= R' -12,263 = 321 kPa
Aho"" 3g 2(1)
RA4 = 683,900.12 kN-m
oM=F(50/3)+U(30.47)
= "12,263(50/3) + 5,690(30.47) 3-34
OM = 377,758 kN-m
cubmerged curve AB is one
of a circle of radius 2 m
V Location of R,
ls Iocated on the lower
11,,f=RM-OM
of a tank as shown. The
T = 583,900jt2 - 377,758
23,201.3
of the tank perpendicular
T =13.2m
sketch is 4 m. Find the
and location of the
lnl The resultant force is 13.2 nr from the toe
and vertical
ts of the total force
uR
(h\ FS. = v on AB
R-,

L\
0.75Qg.207.3\
\ /
= -1An
12,263
I A, CHAPTER THREE
fLUID MECHANICS
I 5() Total Hydrostatic Force on surfaces HYDRAUTICS

Solutlon way of solving t r

tn=yi A
unit pressure is always normal to the
FH = e.81(s)[(4)(2)] and a normal to the circle passes
Fp = 392.4 kN
gh its. center, then the total force F
t1 =l+p also pass through the center of the
t" O, hence the moment about O clue to
6
Or due to Fa and Fy is zero.
Av
4e)3 /D 4o=01
t(4x2)l(5) Fyi-Fsy=Q
e = 0.067 m 437.13i =392.4(7.06n
7 = 0.9578 m
v=1+0.067
y = 1.067 m Note: This is true to all cylindrical or spherical sudaces.

Therefore; Fp is acting 1.067 m below B 35 (CE Board)


3-
.
crest gate shown L=10m
Fy = Wcrglrtasslt
Fv = yVMsD,
of a cylindrical
of which AB is the
V as(-D = 4(A)
A=At+Az supported by a
tural frame hinged at
A'r=(4X2)=8m2
r7n
The length of the gate
A2 = T(2)2 = 3.14 m2
10 m. Compute the
A=8+3.1,4 and location of
A ='l'1.'14 m2 horizontal and vertical
nents of the total
V aa(D = 4("1"1 .14\ = 44 56 mr
ure on AB.
Fv = 9.81(44.56)
Fr = 437.13 kN
Locahon if Fv
Ax =Arrt+Azx.t
rr=1m
41 4(21 E
t7 - (o
-3n 3n q
€O

12 = 0.849 m il
o
io
11.14 i =8(1) +3.14(0.849) c
i = 0.957 m o

Therefore; Fv is acting 0.957 to the right of .4


fI J2o(, CHAPTERTHREE FLUID MECHANICS FIUID MECHANICS CHAPTER THREE . -A
Total Hydrostatic Force on Surfaces & HYDRAULICS & HYDRAULICS rotarHydrostaticForce;"1"##; 139
l'11=yh A lolution
I: r r = 9.81(4.33) [10(8.66)] Considering 1 meter length:
tir = 3679 kN Fs=yh A
Fu=9.81(3)(6x1)
y = \ p.oo1= 2.887 m
FH = L76.58 KN

Therefore; Fs is acting 2.887 m above O


Fv=^lV,
As=Asetor-Atnangre.
Fv = I Vasc
Vasg=Vn1sg-V16s o,:
" ^tu]]{9')
360.
-vz(elzsin 6oo
vasc= *(8.65) x t0 -y2(70)2 [00"6] " to A'=J./$12
Vasg = 125.9 m3 ' fu= 9.81(3.26x1)
Fv -- 9.81.('].25.90) Fv = 31.98 KN
Fv = 1235 kN
F= Jir, *pr,
Moment about O due to Fa and fy = Q
Fv (x) = Fu (y) F = 179.45 KN
7235 x = 3679(2.877)
:
.t 8.57 m

Therefore; Fv is acting 8.57 m to from O 3-37


the magnitude of the
Itant pressure on a 1-ft-wide strip
Problem 3 - 36 (CE May 1999) I semicircular taintor gate shown
Flgure-12.
Calculate the magnitude of the
resultant force per meter length due
to water acting on the radial tainter
gate shown in Figure 021.

Ftr = p,r A
Q
Pr, = (62.4x 2.5)(5 x 1) = 736 16t
Fv - f VaBc
Fv - 62.4 x If (s)r(1)] = 1225tbs

fr (Fr)2 +(Fy)z
fr (780)2 + (7225\2 = 1452lbs
I 40 ;if,';;:,X[ff Force on surraces
FLUID MECHANICS
& HYDRAULICS
FLUID MECHANICS
& HYDRAULICS
CHAPTER THREE
Total Hydrostatic Force on Surfaces l4l
Problem 3'38 Problem 3 - 39 (CE Board November 1993)
l)etermine the magmtude of In the figure shown, the 1.20 m
the horizontal and vertical diameter Qylinder, 1.20 m long is
components of the total force acted upon by water on the left and
per meter length acting on the oil having sp. gr. of 0.80 on the right.
three-quarter cylinder gate Determine the components of the
s'hown reaction at B if the cylinder iveighs
19.62 kN.

6olution
fm=yh A
Fm =9.81(1..2)(1.2 x 1.2)
Solution Fnr = 16.95 kN

Fvr= TVt
' Fw, = 9.81.V/z n (0.6)2(1,.2)l
Fvr = 5.557 kN

Fuz=yh A= (9.81x 0.8)(0.6)(1.2 x 1.2)


Fsz = 6.78 kN
Fvz= yVz= (9.81 x 0.8)11/zn (0.6)2(1.2)l
Fvz = 5.32 kN

Fu=yh A r+=01
FH = e.81(3)[(1X2)] Fm - Fuz- RsH = 0
Fs = 58.85 kN RsH=16.95-6j8
RsH = 10.17kN

l,,Trl,i:'-t1
[tFv = 0]
-^'ffi^ Rsv+Fy1 +Fvz-W=0
Rsv=19.62-6.657 -5.32
Rrv = 7.64 kN

Fv=fVol
F v = e.8U4(2) (1) + 0.7sln(2),(1)l
Fv = 170.94 kN
. ,^ CHAPTERTHREE FLUID MECHANICS FLUTD MECHANICS CHAPTER THREE
143
I+Z rotal Hydrostatic Force on Surfaces & HYDRAULICS & HYDRAULICS Total Hydrostatic Force on Surfaces

Problem 3 - 40 P' = (9.81 x 0.82)(0.00628)


Fz = 0.0505 kN
An inverted conical plug 400 mm diameter and 300 mm long closes a 200 mm
diameter circular hole at the bottom of a tank containing 600 mm of oil having F N
rD- -F ,-7- f1
sp. gr. of 0.82. Determine the total vertical force acting on the plug. F"=0.11,4-0.0505
F" = 0.0635 kN - 63.5 N downward
Solution o+m
--11
r-- 3-4L
2 m diameter horizontal cylinder 2 m long plugs a 1m by 2m rectangular
ll at the bottom of a tank. With what force is the cylinder pressed against
bottom of the tank due to the 4-m depth of water?

ution

hr=2x(1cos30')
h=1.732ll:.
hz=4-hr
hz=2.268m

F.t = yVr
Vt=Atx2
Area, Ar = Area of rectangle DEFG - A4

Area of segment, o,= -y,(I)(l)sin 60o


^2;#0",
Fr = TVt Area of segm ent, Aq= 0.09059 m2
n = (e.81 x 0.82)[n(0.1)'z(0.45)] Area, A1= 1'(2.268) - 0.09059
Fr = 0.114 kN Area,Al =2:1,774m2
Vr= 2.1774(2) = 4.355 mg
Fz-- YVz Fr = 9.81(4.355)
V2 = Vrrrrtu- - Vcyltnd.. Fr - 42.72 kN
% = '(0u15) ft0.21' * (0.2)(0.1) + (0.1)'] - n(0.1)'z(o.r5j Fz= Fs=yVz
Vz = 0.00628 m3
Vz= Azx 2
I 44 fi*i,];H:*:i:ForceonSurraces
FLUID MECHANICS FLUID MECHANICS
CHAPTER THREE
& HYDRAULICS & HYDRAULICS Total Hydrostatic Force on Surfaces t45
r(1)2 F,1=yh A
Area of segment, A2 - !l?0") - %e)g)sin
'-r 120o
360" Fu = e.81 (6.12)t9.24) (11
Area of segment, A2= 0.614nf Fr = 254.56 kN
Vz= 0.674(2) = 1..228 m3
Fz= h= 9.81(1..228) Fv = Y %roa"a
Fz = Fs = 12.05 kN V"nua"a = (Asemtcircle 1- A1r.rp"ro14) x 1

Net force = Fr - Fz - Fl %nua"a : Ltn(s)t * uy 1+..i+lg4


Net force = 42.72 - 12.05 %hu,i"a = 40'1 m3
- 12..05
Net force = 18.62 kN Fv = 9.81(40.7)
Fv: 393.38 kN
Problem 3 - 42
In the figure shown, Problem 3 - 43
determine the horizontal and gate shown is a
vertical components of thq guarter circle 2.5 m
total force acting on the wide. Find the force F
cylinder per m of ib length. sufficient to prevent
n about hinge B.
the weight of
gate.

Solution

c ''!
fp=yE,A
all Fn=9.S1(1)(2.5x2)
Fu = 49.05 kN
Fv = f Vasc 2.5m
F v = 9.81[(2 x 2) - 0.25n(2)z](2..s)
Fv = 21.05 kN
CHAPTER THREE FLUID MECHANICS
146 Total Hydrostatic Force on Surfaces & HYDRAULICS
FLUID MECHANICS
HYDRAUTICS
CHAPTER THREE
6( Total Hydrostatic Force on Surfaces 147
Solve for z and x. Forces due to oil:
Since the surface is circular, LMo -- 0 due to Fri and Fr Fuo = preo A
Fv (z) = Fu (2/3) Fno = (9.81 x 0.80)(7
- 1.273) * Vrn(3)2
2-1.05(z) = 4e.05(2/3) Fso : 635.4 kN
z=1.55m
x=2-z=2-1.55 Fvo : Tn V,,
r=0.45m V, = Volume of imaginary oil above
the surface
[tMa = o] Z, = Volume of halfiylincler _ Volume of
Fu (2/3) + Fv (x) - F(2) = 0 % spher.e
V" = Vzn(3)z(7) _ 1/<
n(z)z
2F = ae.05(2/ 3) + 21.05(0.45) u!
F = 21.09 kN V" = 70.686 m3
Fvo = (9.81 x 0.80)(70.636)
Fvo = 554.74 kN
Problem 3 - 44
Forces due to water:
he cylindrical tank showr-r has a
flt=p,e*A
hemispherical end cap.
Compute the horizontal and jw= \(!,st
x 0 8)(7) + s.8r.(1..273)l xrzn(3)z
\-
vertical components of the total
Fr*v = 953.19 kN
force due to oil and water acting
= Weight of real and imaginary oil above
Fvr.ri
on the hemisphere the surface
+ weight of real water above
the surface
Fvw= (9.87 x 0.8)x Vzn(3)r(7) + 9.81 x
%! ne)3
Fvw = 1,054.07

Total horizontal force, FH = FH() + FHry


lotal horizontal force, Fu = 635.4+ SSS.f
S
Solution Total horizontal force, Fs = 1,5gg.59
kN ,
'l'otal vertical force,
Fv = Fvw - lvo
Total vertical force, Fv = 1,054.01 _ 554.74
Total'vertical force, Fv = 499.27 kN
.

Another wray to solve for the total ,r".ti"ufi..", Fu,


Fy = weight of fluid within the hemisphere
Fv=ToVnty*V.
fy = (e.81x0.4)tix n (3)u)l + e.81[ix
f ln (3),)]
I:v = 499.27 kN
I +8 ;tril;il:fiff Force on surraces
FLUID MECHANICS
& HYDRAULICS
FLUID MECHANICS
6. HYDRAULICS
CHAPTER THREE
fotal Hydrostatic Force on Surfaces 149
Problem 3 - 45 Problem 3 - 46
Pressurizecl water fills the tank shown in the figure Compute the net Determine the force required to
hydrostatic force acting on the hemispherical surface. open the quarter-cyri.der gate shown
weight of the gate is so [N u.ti"g The
i;-_ ioi# ,r*n, o, o

Hemispherical
suface

-T
I

l=
l")
Solution l-
Convert 100 kPa to its
equivalent pressure _t
head, fi.,,
p
h"u= t the gate has circular surface,
v to-tal water pressure passes
,
nP,' =
100 gh point O which is alio the
' 9.81 of the hinge therefore the
-
h.q = 10 194 m due to water pressure
the hinge is zero
h=10.794-5
h -- 5."194 m -01
F(Z.S;=s0(1.2)+Fr(g)
F-24kN
F = Weight of imaginary water.above the hemispherrcal surface
F = yu, Vu, 3-47
V,,, = Volume of cylincler + Volume of hemisphere
nispherical dome shown is filled with
= n(2)2(5.'194\ + L
oil G = 0.9) and is attached to the
V,,,
"t n(2)3 by eight diametrically opposed bolts.
What f in each bolt is required
V^ = 82.025 m3 a ,h"?.-"l.-",?in" dome weigh" 50 kN?ot""
/. = 9.81(82.025)
,
F = 804.7 kN
I 50 ;ifi;;I,.:H:[ Force on surraces
FLUID MECHANICS
& HYDRAULICS
FLUID MECHANICS
& HYDRAULICS
CHAPTER THREE
fotal Hydrostatic Force on Surfaces l5r
Solution
[Erv=o] -20 kPa
F + Foir - F.,r ="0
F=Foir-Fo,,
Foil = Y Vorl at ove rhe curre '
I F.,r
F"ir = (9.81x0.S)[r(0.805)z(5)
+ n(0.s0s)r(3)l i,t
F"ir = 63.91 kN
Fou = Pu* A

F"i' = 20 I+ Q.61), J = 40.72 kN


il
F=63.9't-40.72 r.or mJ
Solution
F = 23.19 kN
Fy = Y Vi.ug,na11, orl above rhe rtome

Fy = (e.81x0. 9)ln(2)2(8) - I n1zf1


Fr. : 739.66 kN 3-49
300 mm diameter steel pipe 12 mm thick carries water
under a head of 50 m
8F6n11 + W=Fv water. Determine the stress in the steel.
r_
Th,|ll -
739.66 - 50
8
Fr.rr = 86.2 kN DD
l5r='
' I
2t'
Problem 3 - 48 - 9.81(50)(300)
)/= ' " / =6131.25kpa
2(12)
Determine the force F s,= 6'13 MPa
required 'to hold the cone ,
shown. Neglect the weight ot
the cone
3-50
the required thickness of a 450 mm diameter steel
pipe to carry a
pressure of 5500 kPa if the allowable working
stress of steer is 124 Mpa.

[5,= PD t
2!'
x.tooo _ p(qso)
124
2t
t = 9.98 mm say 10 mm
I F-t CHAPTER THREE FLUID MECHANICS
I )Z Total Hydrostatic Force on Surfaces FTUID MECHANICS CHAPTER THREE
6. HYDRAULICS
& HYDRAULICS Total Hydrostatic Force on Surfaces r53
Problem 3 - 51
Pipe diametet, p =6 m = 6000 mm
l)etermine the stress at the walls of a 200 mrn diameter pipe, l0 mm thick.
Maximum pressure the tank (at bottom), p
uncler a pressure of .150 m of water and submerged to a aepir, or 20 m in salr = ^lnith
p = 9.81(0.8)(7) = 54.936 kPa
water

Solution .- 2(110x103)(300)
s4.936(6000)
,,.=
rv'
PD,
I S = 200.23 run say 200 mm
2r
p=prnsrrte-/oursrrle
p=e.81(1s0)-e.81(1.03)(20)
" p = 7269.4 kPa = 1.269 Mpa 3-54
,
1.2o9(200') , thin-walled hallo.ry sphere 3.5 m in diameter holds
\,= ="12.69MPa the minimum wall thickness of the sphere if
helium gas at 1700 kpa.
2(10) its allowable stress is
MPa.

Problem 3 - 52
A 100-mm-lD steel pipe has a 6 mm wall thickness. For an allowable tensile Wall srress. S, = !2
stress of 80 MPa, what maximum pressure can the pipe withstand? 4T

60,000 _
1,700(3.5 x 1000)
Solution 4t
t=24.79mm
rs,= 49r
' l/'
p(100)
80= 3-55
2(6)
vertical cylindrical tank is 2 meters in diameter
p = 9.6 MPa = 9,600 kPa and 3 meters high. Its sides
i*tl:^ryrtg.o: rI *."i":-.?j two steel hoops, one at the top,and the other
the bottom' If the tank is filled with water t;;
d"p;h';f ii i.,,*d"*r*""
t tensile stress in each hoop.
Problem 3 - 53
A wooden storage vat is 6 m in diameter hnd is filled with 7 m of oil, s = 0.g
Ihe wood staves are bound by flat steel bands, 50 mm wide by 6 mm thick;
whose.allowable tensile stress is 110 Mpa. what is the required spacing of the
bands near the bottom of the vat, neglecting any initial stress?

Solution

Spacing ot hoops, 5 = 25,Ar,


pD,j
Allowable tensile stress of hobps, Sr = 110 Mpa
Cross-sectional area of hoops. At, = 50(6) = 300 mm2
2Tz
CHAPTER THREE FLUID MECHANICS FLUID MECHANICS CHAPTERTHREE . FF
154 Total Hydrostatic Force on Surfaces & HYDRAULICS & HYDRAULICS Total Hydrostatic Force on Surfaces I 55
lI M6u = 0l 3-57
2T2(3) = F(2.3)
'Ir = 0.3833F cylindrical container 8 m high and 3 m in diameter is rein-forced with two
) Eq. (1) 1 meter from each end. when it is filled with water, what is the tension
F=yiA each hoop due to water?

F= e.s1(# tttzltz rtl : 43.26 kN


ln Eq. (1)
T2 = 0.3833(43.26)
l"z = 16.58 kN (tension in the bottom hoop)
-T-----T-
[tFH=o]
ZT2+27,=P
ZTt=P'",
7Tt = 43.26 - 2(16.58)
ttI rt

8m 6m 6m 8m
t.
Ir = 5.05 kN (tension in the top hoop)

Problem 3 - 56
H t'"-l I

A vertical cylindrical tank, open at the top, is filled with a liquid. lts sides are
held in position by means of two steel hoops, one at the top and the other at
p=yhA
the bottom. Determine the raho of the stress in the upper hoop,to that in the F = e.81(8/2X8(3)l
lower hoop F = 941.76 kN

[IMtop noop = 0]
Solution 272(6): F (13/3)
Tz = 73F /36
Raho=Tr/ Tz T2= 13(94L.76)/36

[IM1.,o = 0l
T-
h
Tz = 340.08 kN
[EMro,.o. r,oop = 0]
2T2(h) -- F(zh/3)
2T{6) = F(5/3)
Tz= F/3

[EMuouuu' = 0l
l_ Tr = 5F / 36 = 5(e47.76) / 36
Tr = 130.8 kN

zrlh) = F(h/3)
Tt = Fl6
3- 58 (CE Board November t9B2)
Raho=
F/u =o.u
drical tank with its axis vertical is 1 rneter in diameter and 6 m high. It
together by two steel hoops, one at the top and the other at the bottom.
F /3
r liquids A, B, and C having sp. gr. of 1,.0, 2.0, and 3.0, respectively fill this

each having a depth of t.iO m.


-on
the surface of A there is adospheric
Find the tensile stress in each hoop if each has a cross-sectional area
mm2.
2rz
t56 CHAPTER THREE
Total Hydrostatic Force on Surfaces
FLUID MECHANICS FLUID MECHANICS
CHAI'TER THREE
& HYDRAULICS & HYDRAULICS Total Hydrostatic Force on Surfaces t57
Solution
Tz= 3.6(9870) = 35316 N
2T, pr=0 'f^
Stress, g, = 2- 35316

Liquid A
A2 7250

1.6 m
+
Liquid B
1.2 m Stress, Sz = 28.25

lrFn = ol
Mpa )

271 + 272= p', + Fz+ Ft+ F+ + Fs


stress in bottom hoop

s=2.0
2T1= 9.7r, + 1.44y +'t.44y + 4.32y + 2.L6y _
2(3.6y)
Tt = 7.44y
Tt ='1.44(9870)
iiquid C Tr='1.4126.4N
1.2 m
s=3.0

stress,s,=A-74,126.4
A1 1.,250
Pt=0 Stress, Sr =L1.3 Mpa ) stress in top hoop
p2= pt + yAh^
Pz=0+ (y'1)(1 2\=1.2y
3-s9
pt=p?+y|lt!<
pt = 1 2y + (i'2)(1 2\ -- 3.6y

p4=p\+y(ltl
pq = 3.6y + (yx3)(1 2\ = 7 2y
f1 = t/2(p2l(1 2\("1)
Ft = t/z(12y1{d, 2)(11= 0.72t

F1 = pr( I 2)(1 )

Fz=12y(12)(1)=1.aay
F\ =t/2(p, pz)(12)(1)
1.86 m2
Fr= V:(3rrv t2y\(1 2\(l)= l44Y
0.93 m2
f t = pt("| .2t(1 t

F, = 3 6y(1 2'10) = a32t

Fs = Ltz(h p1)(1 2)(1)


Fq = t/z(7 2y . 3 6y'10 2)q't1 = 21Ur,,

[lM.p=0] t ,

3.6(2T2)= F'(0.8) + Fz(].8) * F{2)+ Fa(3) + Fs(3.2)


7.272= 072y(0.8\ + 1 44y(1.8\;1 44y(2) + a32v(3) + 2.16ip.2)
Tz=36y
€ After lowering
CHAPTER THREE FLUID MECHANICS FLUID MECHANICS
t58 Total Hydrostatic Force on Surfaces & HYDRAULICS & HYDRAULICS
CHAPTER THREE
fotal Hydrostatic Force on Surfaces t59
In Figure @ Problem 3 - 60 (CE Board November Lg77)
2,831,
Volume of water = (1.86 - 0.93)(b + h) + 0.93b = x000 having specific graviry of 0.92 is floating on salt water
't.86b+0.93h=2.831,
1l^a:l"lg
1.03. If the volume of ice above the water surface is
of sp. gr.
1000 cu. m., what is the
2h+h=3.044 total volume of the ice?
b - 7.522- 0.5h ) Eq. (r)
Solution
lP Vt = Pz vz) Let V = total volume of ice
p1= 101.325 kPa (atmospheric pressure) Vo = volurne displaced
Vr = 0.93(3.05) = 2.8365 m3 Vo=V-7000
pz= 101.325 + 9.81,h
Vz= 0.93(3'05 - b\ 7V." = yr." V = (9.87x0.92)(1,)
Wir":9.0252 V
101.32s (2.836s) = (101.32s + 9.81h) [0.93(3.0s - b)]
BF = yr"u*o1". Vp
309.04 = 309.04 - 1,01,.325b + 29.92h - 9.81bh BF = (e.8111.0.3)(y _ 1000)
29.92h - 1,0"L325b - 9.81bh = 0 BF=10.1043(y_1000)
29.92h - 101.32s(1.522 - 0.5h) - 9.8L(L.s22 - 0.5rt)/1 = 0 [IFy = 0]
29.9Ax - 154.22 + 50.651t - L4.931r + 4.9051* = 0 Wi,"= BF
4.905h2 + 65.65h - 154.22 = 0 9.0252V = 10.1043(V _ 1000)
1.0791= 10704.3
. -6s.65t@
h= " 2(4.e05) =2.039 V :9,364 cu. m.

b = 0.5027 m Solution:
H=b+h=2.542m For hornogeneous solid L-,,cly floating on a homoge.eous
liclr-iicl
yo
t"" t ',.,
The maximum tensile stress occurs at the bottom of the tank. = vr,uay = '', y,,,,.,,.
s liquirl I liqutd
p=yH=e.81.(2.s42)
p = A.937 kPa= 0.024937 MPa then; V-1000 =ffiv
T
0.106796V = 1000
Tension, T: V:9,364cu.m.
2r = pD(l)
+Dr:1.86 m2

D=1.539m=1,539mm 3- 61 (.t to"rd


:k o{ wood 0.60 m x 0.60 m x /l meters in dimension
was thrown into the
2T = 0.024937 (t,53s) (1) and floats with 0.18 m projecting above the water
surfac". fn" ,"^. Lfo.t
T=19.2N thrown into a container of a liquid having a specific gr.avity
of 0.90 and it
above the surface.-Determine i.,e following,
1Tl|.lli
(a) the value":projecting
of ir,
. , A CHAPTERTHREE FLUID MECHANICS FLUID MECHANICS CHAPTER THREE
I OU rotal Hydrostatic Force on surfaces & HYDRAULICS & HYDRAULICS Total Hydrostatic Force on Surfaces I b I
Solution Wrro."
'Ystone --
.In Water: %ro."
S*ood 460
Draft = 7, Ysrone = - = 28,204N/ms
Sn ater 18 0.0153

S*ood h - 0.18
h-0;Ig- 7,
L 3- 63 (CE Board May 1993)
S*,oah=h-0.18)Eq.(f) body having a sp. gr. of 0.7 floats on a liquid of sp. gr. 0.g. The volume
of
l body above the liquid surface is what percent of iis total volume?
ln another liquid iir'-.
'iri;:::r
S*ood l..rf:r
Draft = 1,
Suquia 0.14 ,,YD _ sbo.iy ,,l/bodv
-
S*nod sliquid
It _ 0."14_ ;, - 0.14
0.9 Vo= #Vuoay
- = 0.B75Vb..ru
S*oo4 /r = 0.9h - 0.126 ) Eq. (2)
In other liquid (S = 0.9) Since the volurne of the body displaced (below the liquid surface)
[S.ooa /r = S*ooa /r]
is 0.875
87.5% of its total volume, then the volume of the body above the liquid
h-0.18 =0.91t-0.126
ft=0.54m ) height of the block .is 72.501' of its total volume.

Substitute /r to Eq. (1):


S*'"a(0.54) = 0.54 - 0.18 3.- 64 (CE November 1997)
S-ooa : 0.667 ) Specific gravity of wood block of wood 0.20 m thick is floating in sea water. The specific
gravif, of
is 0.65 while that of seawater is 1.03 Find the minimum area
Weight of block = |*oocl Vbtock of a block
Weight of block = (9.S1 x 0.667)l(0.6 x 0.6)(0.54)l will support a man weighing 80 kg.
Weight of block = 1.272 kN

Wvlr'r = B0 kg
Problem 3 - 62
,l
A stone weighs 460 N in air. When submerged in water, it weighs 300 N, Wwooo
Find the volume and specific gravity of the stone.
{
Solution
Weight of stone = 450 N
Weight of stone in water = 300 N [>Fv: 0]
BF= W'"n *W-ood
Buoyantforce, BF = 460 - 300 - 160 N
. fsu,Vwood= ?Vnr"n + ]*ood V.ood
IBF = y*",", %,on"] (L000 x 1.03) V.oo6 = 80 + (1000 x 0.65) V*oo<,1

160 = 9810 (%"*) V*ood= 0.2105 m3 - Area x 0.2


Vrtun" = 0.0163 cu. m.
Area = 1.05 square meter
CHAPTER THREE FTUID MECHANICS FLUID MECHANICS
162 Total Hydrostatic Force on Surfaces & HYDRAULICS & HYDRAULICS
CHAPTER THREE
Total Hydrostatic Force on Surfaces t63
Problem 3 - 65 (CE November 1997) Problem 3 - 67
A cube of wood (s.g. = 0.60) has 9-in sides. Compute the magnitude and A unifonn block of steel (s = 7.g5) k
direction of the force required to hold the wood completely submerged in will float at a mercury_water Water
water. interface as shown in Figure 2/.
What is the ratio of the distances
t'a" and a'b"
Solution for this cohdition?

Weight of wood = (62.4 x 0.60) gI = 15.795 lbs


Buoyant force when completely submerged in water:
BF=62.4(+I :26.325tbs
Required force = 23.325 - 15.795
Required force = 10.53 lbs downward

Problem 3 - 66 (CE Nov 2000) L2' x 12'


The block shown in Figure 04
weighs 35,000 lbs. Find the
value of ft. A be the horizontal cross-sectional
of the block
B\+ BPr=17y
lt,Von,*f,VD,u=f, V
9.81.(A x a) + (9.81 x 73.56)(A x D) (e.81
= x 7.85)[A(a + b)]
Figure O4 :
a + '1.3.56b 7.85;a + 7.85b
Solution 5.71.b=6.85a
a/b = 0.$a
From the figure shown:

lIFv = 0l
BFr+BFz=35,000 3 - 68 (CE May 199E
BF1 = ysil Vp 5.kg steel plaie is attached to one end of a 0.1
m x 0.3 m x 1.20 m wooden
3P, = (62.4x0.8) (12x12x3) t, what is the length of the pole above
water? Use s.g. ;;;l of 0.50.
Bh= 21,5rU.nnrbt lect buoyant force on steel "f
+ BF: = 35,000
21,,565.44
BFz= 13,ntn.U'tO"

BFz= h,Vo
73,434.56 = 62.410.2)(1,2) hl
h = 1.495 ft
CHAPTER THREE FTUID MECHANICS
164 ,FTUID MECHANICS CHAPTER THREE
Total Hydrostatic Force on Surfaces & HYDRAULICS & HYqRAULICS Total Hydrostatic Force on Surfaces l6s
Solution Problem 3 - 70
Neglecting the buoyant force on steel: buoy (s.g. =:::? is
LVsteet * |V*ood
BF*ooa =
I wooden
i1 se-a water (s.g. = 1.025). _50
mm by 50 mm by 3 m long is made to float
1000(0.1x0'3xV)=S+ .toy Tu", N of steel tr g li.slJ should be
make the buoy noat with exaJny 450;m
y=0'77m
1000(0.5)[0.1 x 0.3 x 1.2] :ffi51*ff;::ff"1j. exposed

Solution 0.05 m
h=1.2_!/

ll
h=1.2-0.77
/r = 0.43 m
iEFv = 0
BF"t*r + BF*ooa -W*oo6- tr/Vrt""l =0
sffi 0.45 m
BFrt*r = 7r* %r"ur
BF,t*r = (9810 x 1.025)%t*r
BF,tu"r = 10,055.25 %t""r N

1,, BF*ooa = ls* Vp


= (9810 x 1.025)[(0,05)r(2.5s)]
Problem 3 - 69
ll--o
BF*ooa = 64.1 N
2.55 m
.
If a 5-kg steel plate is attacl'red to one end of a 0.1 mx0.3mxl,.20mwooden W*ood = |wood Vwood
pole, what is the length of the pole above water? Use sp. gr. of wood of 0.50 W*ood= (9810 x 0.62)[0.05)r(3)]
and that of steel 7.85. W.ood = 45.62 N

Solution W"t*f = Tsteel Vsteel


LV*ood= (1000 x 0.5)(0.1 x 0.3 x 1.2) TVsteer = (9810 x 7.85) V,t""r
W*ooa = 18 Kg LVsteer = 77008.5 V,ua

Wrro"r = 5 kg 10055.25 V"t""r * 64.1 - 45.62 - 7700g.5


%t*r = 0
66953.25 %tu"r = 18.48
BFw- 1000(0.1x 0.3 x d) l/:t""r = 0.000276 m3
BFw=30d
lV,tu"r
= 9810(7.85) (0. 0027 6)
BF. = 1000 Vs 1.2 m
Wrt.r = 21.255 N
Ws = (1000 x 7.85) Vr = 5
V5 = Q.[QQ$]/ 6t
BFs= 0.637 Kg
3 -71
I/V*ood t WrL*t = BF5 + gP* ce of lead (sp'gr' 11'3) is tied to a 130 cc of cork whose specific
18+5=0.637+30d They float just submerged in water. what
gravity is
d=0.745m is the weight of the lead?

x=1,.2-d
r = 0.455 m
t*'
CHAPTER THREE FLUID MECHANICS FLUID MECHANICS
t66 Total Hydrostatic Force on Surfaces & HYDRAULICS & HYDRAULICS
CHAPTER THREE
Total Hydrostatic Force on Surfaces t67
Solution Solution
lIFy = ol
Wc+Wt=BFc+ BFt
Wc=kVc
w6= (1x 0.25)(130)
Wc = 32.5 granls

BFc = Yu, Vc
Brc = (1X130)
BF6 = 130 grm
Wr= IrVr
Wr = (1 x 11.3) Vr
Wt= 11'3 Vr
BFr= Yu' Vr
BFr = (1) Vt=Yr
32.5 + 11.3 Vt.= L30 + Vt,
Vt= 9.47 cc 1

(a) Lead is fastened outside the cylinder (b)


Wt= 11'.3(9.47) Lead is placed inside the cylinder
Wr=106.97 grams
(e) Lead is fastened outside
BFc= y* Vo
Probfem 3 - 72 (CE November 1993) Brc = e.81[f (1)r(1.5)]

A hallow cylinder 1 m in diarneter and 2 m high weighs 3825 N. (a) How BFc = 11'5U U*
many kN of lead weighing 110 kN/mr must be fastened to the outside bottont BFt= Y, Vt
of the cylinder to make it float with 1.5 m submerged in water? (b) How many BFt = 9.8'l'Vr
kN of lead if it is placed inside the cylinder?
Wr=ytVt = 110Vr

[XFv=0]
BFs+ gPr=Wc+Wt
11.55 + 9.8'J.Vt= 3.825 + 1,70Vt
Vt= A.0772xn3
Wr = 110(0.07721= g.4gr*

$) Lead is inside the cylinder


[EFv = o]
Wt+V\/r= gP,
Wt+ 3.825 = 11.55
Wr = 7.735 kN
l6g ;ifil;H:lff Force on surfaces
FLUID MECHANICS
& HYDRAULICS
FLUID MECHANICS
& HYDIIAULICS
CHAPTER THREE
Total Hydrostatic Force on Surfaces t69
Problem 3 - 73 Solution
A stone cube 280 rlm on each side ancl weighing 425 N is lowered into a tank
containing a layer of water 1..50 m thick over a layer of mercury. Determine
the position of the block when it has reached equilibrium.

Solution
W= 425 N
BFu = !uVou
BF,ra = (9,810 x 13.6)[0.28'?(x)]
BFm:19,45n.U' *
BFw = yw Votv
BFyy= 9,810[(0.28)z(0.28 - r)]
BFw=769.1(0.28-.r)

x 7,\[(2.2)z(2.2 - n)] + (62.4 x


2:= !9?n 0.8)[(2.2)z(h)]
/r\-
[IFy = 0]
BFpl+ BP*:ryn
BF = [62.4 x 2.22] (3.08 _ Lah i g.SpS
10,459.81 7 + 769.1(0.28 nx 1. 6)l(2.2)z (1.1)l + (62.4 x
- x) = 425 Y,= !r? 0.4[e.z), (1.1)]
:
9690.71,x 209.652 W=162.4x2.221(2.53)
r = 0.0216 m IBF =w1
r = 21.6 mm
\6?.^a "
2.211 (3.08 - 1
3.08 -1.4h + 0.8/z = 1..76 + O.Zi
y * O.s/r) = [62.4 x z.z?](2.ss)
Therefore; the block will float with 21.6 mm below the mercury surface.
h = o.917 ft

Problem 3 - 74
A cube 2.2 feet on an 3- 7s (CE May 1e97)
100-mm diameter solid cylinder is
. edge has its lower half of 95 mm high and weighing 3.7F
s.g. = 1.6 and upper half mersed in a Iiquid (y = g.17S N is
IN/*) contained in a tall meiui.yti.,au.
of s.g. = 0.7. lt rests in a ling a diameter of 125 mm. Before i*-"rri*,
the liquid _u, ii-^L a""p.
two-layer fluid, with what level will the solid cylinder floati----'--'
lower s.g. = L.4 and
upper s.g. = 0.8.
Determine the height ft
of the top of the cube
above the interface. See
Figure 33.
CHAPTER THREE FLUID MECHANICS FLUID MECHANICS
t70 Total Hydrostatic Force on Surfaces & HYDRAULICS & HYDRAULICS
CHAPTER THREE
Iotal Hydrostatic Force on Surfaces l7t
Solution Probfem 3 - 76
L25mmA
'A wooden beam of sp. gr. 0.64 is 150 nm
by 150 mm ancl is hinged at A, as
;chown in the Figure. At what angle 0 will the beam float i. water?

Solution

x--
LIt
E T
I
C 1m
y1
N

l25mmb

- (a) Before immersion (b) After immersion

Solve for the draft D in figure (b):


TBF=W
ItVo=W Weight of beam, LV= yr,"u^ Vt".,^
(8,175) Vo = 3.75 Weightof beam, W= (9,870 x 0.64)[(0.15)z(5)]
. Vo = 0.0004587 m3 = 458/716, mrn3 Weight of beam, W = 706.32kN
+(100)'?xD=458,7't6 Buoyant force, BF = f *ater Vp
Draft,D=58.4mm Buoyantforce, BF = 9,g10[(0.15), /]
Buoyant force, BF = 220.725x
When the solid cylinder is immersed, the liquid in the tall cylinder riser
due to volume of liquid displaced. Therefore, the volume of liquid lE tvto = g1

displaced equals the total volume of real and imaginary liquid above the BF(5 - 0.5x) cos 0 = W(2.5 cos 0)
original level 2_20.725 x(5 - 0.5r) = 706.32(Z.s)
Vaboue orig. level = VD
5x-0.5x2=8 s.
0.512-5r+8=0
ft (1?5)2(x) = 458,716
r = 37.38 mm x=
2(0.s)
From Figure b: x=2m
/$+a=P+y
!=75+37.38-58.4
sin0= J-
J-x
=rru
V = 53'98 mm Q = 19147"

Therefore; the solid cylinder will float with its bottom 53.976 mm above
bottom of the hallow cylinder.
CHAPTER THREE
li t72 Total Hydrostatic Force on Surfaces
FLUID MECHANICS
& HYDRAULICS
FTUID MECHANICS CHAPTER THREE
& HYDRAULICS Total Hydrostatic Force on Surfaces 173
Probfem 3 - 77 (CE Board May 2003)
Problem 3 - 78 (CE Board November 1993)
F'rom the figure below, it is shown that the gate is 1.0 m wide and is hinged at
A boat going from salt water (sp gr. = 1.03) to fresh water
the bottorn of the gate. Compute the following: (sp. gr. = 1.0) sinks
7,82 cm and after burning Z2,ZZO-kg of .out rises
, (a) the hydrostatic force in kN acting on the gate, up by 15.24 cm. Find the
original displacement of the boat in sea water in kN.
(b) the location of the center of pressure of the gate from the hinge,
(c) the minimum volume of concrete (unit weight = 23.6 kN/m3) needed to
Dolution
keep the gate in closed position.

D+
Jlii: r)'.',i ii, "
llll r, rilil :irl::l :,

I;:

Figure (a)
Figure (b)

Solution
.dS
F=yhA=9.s1(1)(2x1)
F = 19.62 kN f-sF6+r
;r D+0.0762-0.1524
V
IT = D - 0.0762
v= +Q)=0.667r..
[IM,a = 0]
F*y=Tx2.5
19.62(0.66n = 2.57
T = 5.232 kN
Figure (c)
From the FBD of the
concrete block: have to assume that the boat have a constant cross-sectional
area A below
Water surface and use |water = 1000 kg/ma
lxFy = 0l
T+BF=W
Figure (a):
BF = y,uVron, = 9.81 V.on Bh=W
W = Yron, V.on. = 23.6 Vror*
lryVo=W
5.232 + g.g1 y.o,,. = 23.6 V,o,," W= (1000 x 1.03)[A(D)]
V.on. = 0.3796 m3 W=1030AD ) Eq. (1)
CHAPTER THREE FLUID MECHANICS FTUID MECHANICS
I I + CHAPTER THREE
Total Hydrostatic Force on Surfaces & HYDRAULICS & HYDRAULICS Total Hydrostatic Force on Surfaces t75
In Figure (b): 6olution
BF2 - 1V'1
For any floating body; Buoyant force Weight
=
lr*VD=W
w= 1000[,4(D+0.0762)] Solving for displacement in sea water:
W=1000A(D+0.0762) ) Eq. (2) |sea water Vot = W
(&) Vr, = 24,A00 x 2,240
ln Figure (c): Vor = 840,000 ftr
Bh= W -72730
i000[A(D - 0.0762)l = w - 72730 ) Eq. (3) Solving for displacement in fresh water:
"L
|freshwater Voz=W
From Eq. (1) and Eq. (2): (62.2)(V D2) = 24,000 x 2240
Voz= 864,308.68 ft3
Iw=w
1030AD = 1000A(D + 0.0762) Figure (a)
1030D=1.000D+76.2 Let /l be the difference in trre drafts in fresrr
D = 2.54 m (draft in sea water) Voz- Vu = Area x /r
& seawater:
864,308.68_940,000
From Eq. (1) fr_
32,000
w=1,o3oA(2.s4) h = 0.76 ft
w = 2516.2A
Draft in fresh water, D = 34 + 0.76 = 34.7G ft
From Eq. (3)
7000A(2.54 - 0.07 62) = 261,6.2A - 72730
2463.8A = 26L6.2A - 72730
A = 477.23m2
3-80 lcrao@
rr an arbitrary shaped body with a submerged
volurne Vs and a
Therefore: p', submerged in a fluid of density p1.
What is the net vertical force on
w=261.6.2@n.n) body due to hydrostatic forces?
W = 1,248,529 kg (9.81/1000)
W= 12248 kN
Fnet= Tf Vs
yf=ilxg
Problem 3 - 79 Fn* = plg Ve
A ship having a displacement of 24,000 tons and a draft of 34 feet in
enters a harbor of fresh water. If the horizontal section of the ship at
waterline is 32,000 sq. ft, what depth of fresh water is required to float 3-81
ship? Assume that marine ton is 2,240 lb and that sea water and fresh w
)herical balloon, 9 m in diameter is fined with herium
weight 64 pcf and 62.2pcf, respectively. gas pressurized to
u.,a unchor"d by a rope to the ground.
H::,,L^,1Tli'_1*.':
Frting the dead weieht?fof?2"c:
the bailoon, derermini ,r.." iJ*r", iri,fr.
,oo"
R-212mloK for helium gas and ,",, = tilJ
N/-..
.q, CHAPTERTHREE FLUID MECHANICS FIUI!i MECHANIC5
II O Total Hydrostatic Force on surfaces & HYDRAULICS CHAPTERTHREE
& HyDRAUUCS -rotat Hyctrostatic Force
"" surracll t77
Solution Wu,=45.52 N (fronr Figure3_ l)
P "177
x 103 BF = Y,n Vn
nerrutrr BF = 9810(1.025)[(0.05),Ll
' RT 272(273 + 2o)
BF = 25,138L
Yhelrun, = 1'787 N/mr
Vbulonn = \n(9/2)3
[IMo = 0J

Vboloon = 381.7 m3 =u

[IFv = 0]
[r!,i;:* ?,;z{ltit'),3;'r
12.57 Lz 58.43
BF-W-T=0 L*2,33m
BF = y"1, V5o1lnon
sin 0 * 2/l
BF ='t1..76(381.f = a43.u * sin 0 * 2/2.93
h/- yh.riunr Vboiloon
0 *59o
W=1..787 (381..7) - 682.1 N
4488.8-682."t=T
T = 3806.7 N
t.
mnr in diameter and 200
Problem 3 - 82 If:ir:t';*il;:ffiij:r mm hisrr and weighs r.6
o a body of tiquid h":".:.:-Tquired to-push..'h" :on-u d;;:;nward)
t
*l ;ffi
The buoy in Figure 3 - N of steel weight attached. The buoy
has 80
lodged against a rock 2 m deep. Compute the angle 0 with
has
:ffilffi,111i j 3,,1?,,::":",::i f;"T:i:*,jT;
the horizontal at
which the buoy will lean, assuming the rock exerts no mornent on the buoy.
:r:1,
Solution
The reqrrirecl downward vertical
force is:
F=BF-W
BF * yriqud %uuo
BF - (e,810 x 0.S)
[(r/3)( 0.1/212(0.21!
BF*4.11 N
F*4,11-1.6
F-2.51N

This force F. L5l N becomes


no matter how deep further
tt
cone is submerged
FLUID MECHANICS FLUID MECHANICS
t7 8 ;i*il;:#[:,? ,o,." on surraces & HYDRAULICS ,& HYDRAUTICS _
Total Hydrostatic
CHAPTERTHREE
Force j";;:;; t79
""
Problem 3 - 84 Solution
'Io what depth a 2-m-diameter log, 4 nr long and of sp gr 0.425 sink in F=40N
fresh water.

Solution
For a homogeneous solid body floating on a
homogeneous liquid:
yo =
j!!9L vu.o,
sliquid

ast=ffiar
As = 0.425nr2 (shaded area) Let 7 = volume of woocl

From geornetery: ln water:


As = Ar".,or - At ,ongl* IXFY = 91

0.425nra = lz ra Q" - lz 12 sin 0 Bh-W-F=o


0, - sin e -- 2.67 9810V_W =40
= 49+w
es10 )
Y Eq' (1)
Solve 0 by trial and error:
Try 0 = 170' ln glycerin:
170" (n / 1"80") - sin170" = 2.7 6 (*2.67)
lxFv = 0l
Try Q --'1,66" BFz-W-F=0
1,66'(n/'180") - sin166" =2.655 (+2.67) (9,810x
1.3)V_W=t00
Try 0 = L66.M" (e,810 xr.sr14*wl -w=loo
1.66.M" (n / 1,80') - sin166'44" = 2.67
'L lsro J
52+1361_W=100
It=r-A W= 160 N
ft=1-(1)cos(0/2)
h = 1 - (1) cos (166.44'
From Eq. (1):
/2)
ft = 0.882 m , _ 40+160
9810
V = 0.0204 ml

Problem 3 - 85 Unitweight, r=
A block of wood requires a force of 40 N to keep it immersed in water and a
Unit weight, f = Zg43
+=#
force of 100 N to keep it immersed in glycerin (sp. gr. = 1.3). Find the weighl N/pr
and sp. gr. of the wood. SP.gt.,r- Iwoo'l -7843
Iwater 9910
. Sp. gr.,s = 0.8
I

CHAPTER THREE FLUID MECHANICS FLUID MECHANICS


tB0 Total Hydrostatic Force on Surfaces & HYDRAULICS & HYDRAULICS CHAPTER THREE
I otat Hydrostatic Force
on Surfaces tBt
Problem 3 - 86 Since the volume of oil
remain unchangecl;
A rectangular tar-rk of interrral width of /orr linirrar,l = voir lrnalt
5 m, as shown, contains oil of sp. gr. = 5)J5](1 2s) = (0 s)(s)(/,) _
!9 0.1274
0.8 and water. (a) Find the depth of oil, /r = 1.301 m
h. (b) If a 1000-N block of wood
if theoil-water interface crrops -
is As shown in Figure b'
floated in the oil, wllat is the rise in free rree surface of water by a distance of y, the
surface of the water in contact with air? 1i' rrge
right compartnrent is twice o;;I;,';;;the cross-sectionar
thar ;riffi;;ilrnparrment. area of the

Sum^-up pressure head


frorn oil surface to wat
vvat€rsurfaceinmof
0+.1.301(0.8) + (3_ y)-4_V/;:; '-
water:
1.0408- 1,-3ry/2=0
Solution 3v/? = o.o4o8
1000 N
y/2 = 0.0136 m or 13.6 mur
o Therefore; the free surface
of water will rise 13.6
{vtz mm.

I
oit
s=0.8
o

I
I

4m
-T-
I

4m
3-87
open cylindrical tank 350
lcallyinro a body
mnr in diameter and 1.8
m high is inserted
,0;5 rnrr,:{;,ri; ,i :;., 1m, lit,l I
ir
,ocr< : of .il,"-t "i;;";;'-I:::1T"'"r
,concrete ;. ;Jtf'!XiT,,':"^]li ll1.
d: wn and
('p g.. = 2.4j;;;p-";;il:iir'i.-n a ncr floats *i,r, a 1300
n""1,' with
"
rsoo
irilj or the cyrindei, io what
i.ll
riiil I a"pin *iri;J:;:X"":T1" Tff::;%T
Figure (b)
Figure (a)

.oJ
(a) Depth of oil: (Refer to Figure a)
Sum-up pressure head from oil surface O to water surface € in m of water
+BF.vr -W=0
BF.on. = I*ater Vcon.
) Eq. (r)

a+/r(0.8) +g-4= Pz W"on"


fv V.on. =
I.onc
0+0.8ft-1=0
/r = 1.25 m
t,TCOnC -_ 1300
e870(2.4)
(b) Rise of the water surface: (Refer to Figure b)
%on. = 0.0552 m3
BF =W tF.on. = -9310(0.0552)
lotVo=W BF.un. = 541.2 N
(9810 x 0.8) Vo = 1000
tFn,r = l*arc, Vn
V6 = Q.l)/Q sP
EF.yr = 9Si0lf (0.eS;27,;

c1t =943.83 lt
CHAPTER THREE FLUID MECHANICS
182 Total Hydrostatic Force on Surfaces & HYDRAULICS
FLUID MECHANICS
& HYDRAUI-ICS _ CHAPTERTHREE

l;ronr liq. (1)


Totat Hydrostatic Force on Surfaces t83
,Solutlon
541.7+943.83h-1300=0
/r = 0.803 m
Wr = 205 kg
^101.325 kPa)
Applying Boyles Law (taking p^nn = 6me
Wr = 205 k9
Before irrsertion:
0.8.4 -
Absolute pressure in au, pt = 101.325 kPa __{ 0.6m O
Voiume of air inside the cylinder, V, = i (0.35)'z(0.18) 1.8m -------r-
I .2.1 m Y
Volume of air insicle the cyiinder, % = 0.0173 m3 0.96

After insertion:
',
1.Bm
l
*

Absolute pressure in air, p2= 101.325 + yh H

^ Absolute pressure in air, p2= 10L.325 + 9'81(0.803)


Absolute pressure in a.-r, 1tz = 109.2 kPa
Volume of air inside the cylinder, V, = i (0'35)2 r l'', -rl
Volume of air inside the cylinder, Vz = 0.0962x l*,
[pr Vt = pz Vz] Figure a: HiSh Tide
101.325 (0.173) = L09.2(0.09 62t)
Figure b: Low Tide

x ='1..67 m ,ht of-chain = 12kg/m


x-h+U= 1.8 rty ot steel = 7,790 kg/rfi
y =1.8-1.67+0.803 of steel (chain) =1Z1ZZOO
t/ = 0.933 m of steel (chain) = 0.00154
m3 per nreter length
Therefore, the open end is submerged 0.933 m below the water surface. Figure a:
lxrv = 0l
B\+Vpr-Wt-Wz=0
Problem 3 - 88 (CE Board) BFt = y,* Vo
A cylindlical buoy 600 mrn in diameter and 1.8 m high weighs 205 kg. It is = (1000 x 1.03)[* (0.6), (0.e6)]
moored in salt water to a 12 m length of chain weighing 12 kg per m of its
Bh = 279.58kg
length. At high tide, the height of buoy protruding above water surface is
0.84. What could be the length of protrusion of the buoy if the tide dropped BFz = 7,* %noin
2.'I m? Density of steel is 7,790 kg/ mt. Use density of water : 1000 kg/m.. = (1000 x 1.03)[0.0015a(r)]
BFz= 1.586L

Wz=72L
279.58 + 1.586L _ 205 _ 12L
=0
L=7.'1,6m

Depth of wa ter, [! = L + 0.96


Depth of water, iLI = g.12 m
I 84 fifil;:#[:[ ro,.." on surfaces
FLUID MECHANICS
& HYDRAULICS
FLUID MECHANICS
& HYDRAUTICS _ CHAPTERTHREE
Total Hydrostatic Force on Surfaces r85
Irr liigrrre h:
Solution
Depth of water, H' = H - 2.1,
Depth of water, H' = 6.02 nr
Weight of ball:
Draft,D=H'-L' W = yutt Vaou
Draft,D=6.02-L' S4r = (9870 x O.aZ)
!n(0.15)3
[IFv = 0] W= 5s.2S N
BF'1+gP'r-Wt-W'r=g Buoyant Force:
3P" = (1000 x 1.03))[f (0.6)'zD] BF = y*otu, Vb"rr
BF',r: 291.rU (6.02 - L',) BF = (9810)
+r(0.1s)3
BF' t= 1753.1,8 - 29L.23L' BF = 138.69 N
BF'2 = (1000 x 1.03)[0.00154(L')) Depth of pool:
BF''= 1'56Ut' Work done by W = Work done
by BF
W'z=12L' W(4.3+h)=bpfu\
58.25(4.3+h)=138.69h
1753.1,8 - 291,.23L' + 1.586L', - 205 - 12L', = 0 h= 3.llm
L'= 5.13 rn

D=6.02-5.13=0.89
y=1'8-D 3-90
y=1.8-0.89 hydrometer weighs 0'0214N
and has a stem at the upper
y= m (Iength of protrusion) end which is 2.7g
auuf"'
0.9'1,
illHT:: rT;#*"'n t- o""t in ol^(sp.
'", s,. = ;.;;, that in

Problem 3 - 89 (CE Board)


In alcohol:
A wooden spherical ball with specific gravity of 0.42 and a diameter of 30()
BF =W
mm is dropped frorn a height of 4.3 m above the surface of water in a pool of
(9810 x 0.821)V s,
. unknown depth. The ball barely touched the bottom of the pool before it = 0.0214
began to float. Determine the depth of the pool. Vo" = 2.657 x 10-6 m3
Von= 2,657 mms

oir:
BF =W
(9810 x 0.78)VD,= 0.0214
Voo = 2.797 x 10-6 m3
Voo= 2,797 mmt

VDn Voo- Vo,


O - 2,797 - 2,657 = |40 mm3 Alcohol, s = 0.82I
Oil, s = 0.78
Yo. tQ.7e1zh=740
229 mm
CHAPTER THREE FLUID MECHANICS FLUID MECHANICS CHAPTER THREE
I UO rotal Hydrostatic Force on Surfaces & HYDRAULICS & |{YDRAUUCS Total Hydrostatic Force on Surfaces t87
Problem 3 - 93
Problem 3 - 91
A plastic cube of side L and sP gr. 0.82 is placed vertically in water. ls the A block of wood (sp. gr: = 0,64)-is_ in the shape of a rectangular parallelPpiped

m
a 10-cm square base. If the block fltats in salt w"ater *i*, it"'rqriur"
cube stable? |avin-g
LxL base horizontal, what is its maximum height for stable equilibrium in the
'
Solution upright position?
Ihe body is stable if M is above C. folution
Drarr,D=opt- i i r-+-6c I Note: The body is stable when M is

liT-l F".
Draft, D = 0.82L lbove G and unstable if M is below 10cmx10cm
I G. Mth'smaller value of H, the
I + (L)(L)3 tnetacenter M will become higher

l
lvtD^:
VD (L x L)(0.821) t than G making it much stable.

t-.l
-
MB,,:0.'1,02L When H increases, M will move

GB"= L/2- D/2


GB,, = 0.091
,lil
rit
closer to G making it less
. Hence, the maximunr
for stable equitbrium is
Seawater, s = 1,03

Since MBo > GB", M is above C.


The body is stable.
lll Mcoincides with G, or MBn=

Waterline Section
the figure:
Problem 3 - 92 GB"= H/2- D/2
A solid wood cylinder of specific gravity 0.6 is 600 mm in diameter and L2(x) Draft,D=*#H=0.62.tH
mm high. If placed vertically in oil (sp. gr. = 0'85), would it be stable? GB"-0.5H -0,621,H/2
GBo=g.1tt,
Solution
sP gr' wood
Draft, D - , wB"=
*r
Draft, D = ffi trzoo) : 847 mnr #(roxro)3
MBo=
I (10x10)D
MB"=
v, E MBn=
100 _ 73.419
,E
a 12(0.621,H\ H.
+ (300)4
MB,,= O' ;' o
N
n(300)'(8a7)
MB"-- 26.56 mn oR: MB.= B' [, * tu"2 e)
12D( , )whereo=oo
GB" = 699 -1/z(847)
GB" = 176.5 MB,, =
lo' (1 + o) 73.41.9 =
1,2(0.621H)', H
Since MB, < GBo, the metacenter is below G
Therefore, the bqdY is unstable.
CHAPTER THREE FLUID MECHANICS FLUID MECHANICS
r88 Total Hydrostatic Force on Surfaces & HYDRAULICS & HYDRAULICS rotar Hydrostut'. F."?1PJ"E:JH:: I Bg
lMB"= GB"l Initial metacentric height, MG = MBo _ GBo
73'419 Initial metacentric heighg MG = 77 .49 _ tt/.SS
= 0.189H
H Initial metacenkic height, MG = -39.96 mm
H = 8.43 cm

3-9s
Problem 3.94
A wood cone, 700 rnm diarneter and 1,000 mm high floats in water with its
vertex down. If the specific gravity of the wood is 0.60, would it be stable?
Determine also its initial metacentric height.

Solution
V-ood = { n(350)'z(1000)
V*ood = 1,28,281,,700 m3
Initial metacenkic height:
ttYD - ---
0.60 tt
Ywood ---T-- !--1, * tan2 el
ut"= 12DL
Vo = 0.6 V*ooa
E
E
2 )
Vo = 0.6 (L28,281.,700) o
o where 0 = 0"
o
Vo = 76,969,020 mm3
MB"= e)2 f, * tunt o"l
By similar solids: L2(2.4)l z _l

vruo,l _1rooo13 MBo=2.8125m


v, - l. D,/ GBo = 2., - 1.2 = 1.5 m

v
osv-* -l
u,uu,l _ r 1000
D
\'1
J
Initial metacenkic height, MG = MB"_ GB"
Initial metacentric height, MG = 2.g725 _ 1-.5
D = 843.4 mm il metacenkic height, MG = l.31^2i m
350
843.4 1000 Waterline Section e=+3
r = 295.2 mm
-'14.93
I +Q95.2\4
MBn= =-=//.+vmm
G 76,969,020

Fronr the Figure:


GB"=759-3D/4
GB"=750 -3(U3.4)/4
=#l*o+--l
GB" = 117 'nU * =2.9'l.m

Since MB,, < GB", M is below C and the cone is UNSTABLE.


t 90 ;l|3|I;:'.H[:,1,o,.." on surraces
FLUID MECHANICS
& HYDRAULICS
FLUID MECHANICS
& HYDRAULICS
CHAPTER THREE
Total Hydrostatic Force on Surfaces 19l
Metacentric height, MC = MB" - GB" Along longitudinal axis (ro[ing):
Metacentric height, MC -- 2.91 - 1.5 = 1.41 m B=10m
gince MG > MBn, the moment is righting moment.
MBo= I-lr*
12DL
tul'el
Righting moment, RM = W (MC sin 0) ;l where o = oo

1111= Bf = yVp
w= (9.8.1, x 1.03)[9(1sX24)l= 3,273.8 kN MBo= (1 + o) = 5.45 m
Rrghting n"roment, RM = 3,273.8[(1.41) sin 74.93"1
Righting nloment, RM = 1,189.3 kN-m "i*,
Metacentric height, MG = 5.45 3.2gs
-
Metacenkic heighf MG = Z.21Sm (the barge
is stable in rolling)
Along kansverse direction (pitching):
Problem 3 - 95
B=30m
A barge floating rn fresh water has the iornr of a parallelepiped having
dimensions 10 m x 30 m by 3 m lt weighs 4,500 kN when loaded with center MB"= B'| *8.2ol
of gravity along its vertical axis 4 m from the bottom. Find the metacentric
"'""- :nDl'*-T-l where0 = 0'
height about its longest and shortest centerline, and determine whether or rrol
the barge is stable *u"= o) = 4e.o2m
#ru)(l+
Sotution Metacenkic height, MG = 49.02 3.2gs
-
Metacenkic height, MG = 4S.7gSm (the barge
W = 4,500 kN is stable in pitching)

.l 4 m above the bottom


3- 97 (Ce eugust rSZS;

I--
3m
frRollinq
crane barge, 20 m long, g meters wide, and

l|r::::l
meters l:]fi
*"lqt*g 20 shortto;H;;
2 meters high roaded at its center
on fresh water with a draft of
and has its center of giavity located along its ,er'.ai
I t L'50 meters above its bottoml Compute the horizontar
side from the centerlin"
distance out", to
"_i,uut "
t(J
barge thiough which the crane could swing
?f,g:
Solve for the draft, D
:r":a i::il*l-ilj:1ti{Td ,i* q" lenter of the deck, and tip the
with the 2O-meter edge just touching the water
surface?
IBF =vq
YVo=W
9.81 [10 x 30 * Dl = 4,500
D=1.53m
GB,,= 4 - D/2
GB,,= 4 -1.53/2
GB"= 3.235 m
CHAPTER THREE FLUID MECHANICS FLUID MECHANICS
t92 Total Hydrostatic Force on Surfaces & HYDRAULICS q HYDRAULICS CHAPTER THREE
Total Hydrostatic Force on Surfaces t93
Solution z=dsin0
I short torr = 2000 lb z = 0.552sin 11.31" 0.10g m
=
= 900 kg GB"=1.4-d=0.848m
Wp= (20 x 900) 9.81
Wn = 176.58 kN Road roller MB"= B'lr* tan2el
Wp = 20 short tons 12DL 2 J

MB^=
,vLDo-
82 |-., . ,u!t 11.31'l
45 Lt*___r_J =a.saam
MG = 4.533 - 0.848.= 3.685 m
r=MGsin0
r = 3.685 sin 11.31" = 0.723 m
BF = yVe lrMc = ol
Br=9.81[8 x7.2x70] (BQr=Wn(L+z)
BF ='1,883.52 kN = Wr ].,883.52(0.723) = (12d.58)(L + 0.108)

Weight of barge, Ws = BF -Wx


L =7,604 m) Horizontal distance from the center of the
deck
Weight of barge,Ws= 1,883.52 - 176.58
Weight of barge, Wn = 1,705.94 kN
3-98
Tilted position wooden barge of rectangurar cross-section is g m- wid",
4 m high, and 16 m long.

Xjii*:Tg: ::Ta1if =
*."ieh: of.rf I i..:p l"1d
or r,ioo kN:;:r"dt"; ib ow.
25 kN (included t";;ilidilii'.rffi;
, ot z.J m to {one
.*1".fl9"; " side, it willcause the barge to go down +SO mm in "
ttre
of imrnersion and also rise 450 mm in th"
Wsl
'p .oltopona ing wedge oy iennn
bur"lu i*,*6;t
3*l*::,f*:*^
how far above the !::.T,:l:,".,k*r)
te waterline is the centerof ""i,ii"
;6;r;: tLJ;
*"igh,
ffij

tut' 0 = 9$s
0 = '11.31'

polve for the new position of C in the tilted Position:


Wr(0.5) = ws(d)
1,883.52(0.5) ='t,706.e4(d)
d = 0.552 m
CHAPTER THREE FLUID MECHANICS FLUIDMECHANICS
t94 Total Hydrostatic Force on Surfaces & HYDRAULICS & HYDRAULICS
CHAPTER THREE
Total Hydrostatic Force on Surfaces t95
Solve for the draft, D: Problem 3 - 99
BF =W The waterline section of a 1,500-kN
barge is as shown. Its center of gravity
(9.81 x 1.03)[8 x 16 x D] = 1,500
D=1.16m l;1,n*ffithe'center
or u,ovu,,.v E;;;ffi;'il; ;ilil""
is
height

ln the Tilted position:

wB"=
tan0=f *t
f= frrctangle * /triangl" t lsernirircle
e:6.42 1= #(12)(8)3+
'l +(6,X4)3 x2+ #(4)4
^r I = 676.53ma
::-lr
MB"= * tu1'0
12Dl 2 l I

IBF = vvl
MB^= 8' lr-tur'6.42'I
" 't2(1.16)l 2
=n.uu.n 9.81' Vo = 1,500
Vo ='1,52.9 m3
)

[IMs" = 0]
1,425(b)+75(q)=BF(c)
MBo=
ffi =4.4?sm

, =MBosin0 IMG= MBo-GB"l


:
c 4.63 sin 6.42" MG = 4.425 -1.5
c = 0.5'18 rn MG = L9E m) initial metacentric height
a = 3.42 sin 6.42o + 2.5 cos 6.42o
a = 2.867 m

b = (lt + 0.58) sin 6.42"


1,4251(h + 0.58) sin 6.42"1 + 75(2.867) = 1,s00(0.518)

lt -- 2.947 m) distance of G from the w.s.


CHAPTER THREE FLUID MECHANICS
l96 Total Hydrostatic Force on Surfaces & HYDRAULICS
FTUID MECHANICS
& HYDRAUTICS
CHAPTER THREE
Total Hydrostatic Force on Surfaces 197
Supplementary Problems 3-104
Water in a tank is pressurized to g0 cmHg. Determine
the total force per meter
Problem 3 - 100 width on panel AB.
A vertical rectangular gate 2 m wide. and 1.2 rn high has water on one side Ans: 482 kN
with surface 3 m above its top. Determine the magnitude of the total
hydrostatic force acting on the gate and its distarrce from the water surface
Ans: [ = E4.b kN. yy, = 3.63 nr

Problem 3 - 1O1
A vertical serni-circular area of radius r is submerged in a liquid with its
diameter in the liquid surface How far is the center of pressure from tht'
liquid surface?
Ans: 0.589r
3-10s
Problem 3 - 102
An open vat holding oil (s = 0.80) is 8 m long and 4 m deep and has .r
trapezoidal cross-section 3 m wide at the bottom and 5 m wide at the top
Determine the following: (a) the weight of oil, (b) the force on the bottom of tht'
vat, and (c) the force on the trapezoidal end panel.
Ans (a) 1002 kN; (b) 752 kN Ans: Fu =7491b )
(c) 230 kN Fv = 2,7341bs {
Problem 3 - 103
Freshly poured concrete approxirnates a fluid with
sp. gr. of 2.40. The figure shown a wall poured
'between wooden forms which are connected by six
bolts. Neglecting end effects, compute the force in
the lower bolts.
'oL
Oil, s = 0.75
Ans:19,170lbs

3. 106
tute the hydrostatic force and its location
on semi-cyrindrical indentation

ii:,H.
Consider only 1 mete. reng*, oi.yri'a".';d;;;;.;;;to the

Ans: Fs = 109.5 kN @ 1.349 m below D


Fv = 20.5 kN @ 0.531 m to the left
of B
FLUID MECHANICS FLUID MECHANICS
198 fHili:#*:,? ro.." on surfaces & HYDRAULICS & HYDRAULICS rotarHydros,ur,.rTf;J't:J#fi: 199

Problem 3'LO7 3-109


The 1-m diameter solid cylinder shown is 8 rn long perpendicular to the figure
and rests in static equilibrium against a frictionless wall at O. Determine thc'
unit weight of the cylinder'
Ans: 10.5 kN/nrl
Ans: s. = 10.5'

Water

Problem3-108 i
The section of a concrete dam is shown in the figure. Concrete weights 23'!
kN per cubic meter an<l water weighs 9,790 N per cubic meter. Coefficient of
friction between the dam and foundation is 0.55. Determine the factors of
safety against sliding and against overturning, and also the soilpressure at thc
heel and toe. Assume hydrostatic uplift varies unifornrly fro5n full hydrostatlc
head at the heel of the dam to zer6 at the toe. Consider L m length of dam.
Ans FSs= 2.20; FSs = 1.66
qL'*r = 85.2 kPa; q6= 300'2 kPt
CHAPTER THREE FLUID MECHANICS
-AA
ZUU Total Hydrostatic Force on surfaces & HYDRAULICS

Problem 3 - 11O
Re, a ti ve E s u ifi f"Tff Tu:;iT 20t
Two spheres, each 1.3 m in diameter, weigh 5 kN and 13 kN, respectively.
They are connected with a short rope and placed in water. What is the tension
in the rope and what portion of the lighter sphere prokudes from the water?
Chapter 4
Ans: T = 1".74 kN; 40.1%
Relative Equilibrium
Problem 3 - 111 of Liquids
A block weighing 1,25 pcf is 1 ft square and 9 inches deep floats on a stratified
liquid composed of a 7-in layer of water above a layer of mercury. (o)
Determine the position of the bottom of the block. (b) If a downward vertical er certain conditions,
the particles of a fluid
force of 260 lb is applied to the center of mass of this block, what is the new o' betwee' each other ,"r';;';:""::.:,tjtto mass rnay have no
,'",i,iol ',i'"',i'",,
relahve
position of the bottom of the block? rr<r $
is moving with a constanr .,,.-;;,.:;
onrrun, speed *
.14y ue
r'" tn motion.
.,
If a nrass of
Ans: (a) 0.8 " below mercury """JlilT,::T',.1Y*"t
e s.ame as in fluid ,ruri, (uniform velocity), ,f.r"-.".ai,ons
;; ;il,"*; .h;;i;;' u,li'il,n" ooo, are
(b) 4.67" below mercury
reguired, and this wilr lTfJilHTil; #;:'"".
1urscteJ
be dir.";;";';;ilffi'ililotation), special rreahnent
Problem 3 - 112
Would a wooden cylinder (rp. g.. : 0.61) 660 mm in diameter and 1.3 m long
be stable if placed vertically in oil (sp. gr. : 0.85)?
Ans: Not stable TR ANSLATToN ( ivrovrNc
VESSET)

Motion
Problem 3 - 113 rrDr(ler a lnass of fluid
A rectangular scow 7 ftby 18 ft by 32 ft long loaded with garbage has a drnft
of 5 feet in water. Its center of gravity is 2 ft above the waterline. Is the scow
*n,m*l;:.:m:T$jF.t+;*::H"i:,1.,.1nff Tj;ilT:di;
stable? What is the initial metacentric height? , i ,' t;; * i'^'j':#.,l.ili lffi"'l1;T" i;ffi i::i:i* ri
' Ans: The scow is stabla
" "
,j.$ hic i s

, MG=0.90ft
w=M9

Problem 3 - 1.t4
A cube of dimension L and sp. gr. 0.82 floats horizontally in water. Is the
stable?
Ans:

:o
lt
>

REF = Ma
CHAPTER FOUR FLUID MECHANICS FLUID MECHANICS
ZUZ Relative Equilibrium of Liguids & HYDRAULICS 6(HYDRAUTICS
Rerative Esu',fffftjrfi:ir| ZO 3
F'ronr the force polygon showtr Vertical Motion
REF
tan0= Consider a mass of fluid accelerated
w uo

tan0:
-Ma ;.':",fi :'Tili; J
;flJ;lt, :L'm
:* :,*" "''il;;;"I'q'id .b;;; il';;;ri;
n*, ;x1, #
Mg ro,,u,=tuti,"n.oli"
- ii:,:T"-,:'.t:t "i fr: the i,rerria

,a lEF,, = ol
tanO=:o
Eq.4-1 F=Ma+yV
, M=py= I y
Therefore; the surface and all planes of equal hydrostatic Pressure must I
0 'ffi"#ffi
be
o
inclined at this angle 0 with the horizontal F= LVa+yV o o
o-

ff:
\
Volume, V = Ah
lnclined Motion F=PA
Consider a nrass of fluid being accelerated upwards at arr inclinahon ov
Y
the horizontal so that al = a cos o and n,, = n sin cr., l1A= o
(AI)a+y14111
6
P=y141+ a/g)
REF' = 1Y uu
I,=,^
W=Ms

3s REFs = [t4 3n

Use 1+; for upward motion


and (-) for downward
motion
Note: a is positive.for acceleration
and negative for deceleration.

From the force polygon shown


Mou
" tan0=
M g+ May
aH
tarr e =
8+av

Use (+; sign for upward motion and (-) sign for downward motion
204 ;5#J5[;"""trium or Liquids FLUID MECHANICS
& HYDRAULICS
FLUID MECHANICS
& HYDRAUTICS
CHAPTER FOUR
Relative Equilibrium of Liquids 205
From calculus, slope = dy =tan0
dx
:dv =tane= _,2,
dxg
. ur2
clY=
-r4^
o
6
lntegrate both sides:

Hgurel-t1ay Figure4-1(b)

W=M9
For cylindrical container of radius r revolved about ils vertical
axis, the height
of paraboloid is:

W=M9

CF (W/g) o2 x
=
e

where ol is the angular speed in radians per second.


' Figurea-1(c) NOTE: 1 rpm = n/30 radlsec

Figure 4 - 1: Paraboloid of revolution 4 - 1 (b),the relationship between any two points


figule in the parabora can
grven by (squared property of parabola):

From the force polygon


tant-CF
- w Eq'4-7
- ;= v
tano= \w/g).'zx
w of P"araboloid of Revolution

tu^e=
tlt Eq.4-4
g

Where tan 0 is the slope of the paraboloid any point x from the hxis of rotation

Volume=l/znrah Eq.4_g
CHAPTER FOUR FLUID MECHANICS FLUID MECHANICS CHAPTER FOUR
206 Relative Equilibrium of Liquids & HYDRAULICS & HYDRAULICS Relative Equilibrium of. Liquids 207
For closed cylindrical containers more than half-full of liquid, rotated
':""':'::::;::'"':?:::,::ers more than harr-ur or riquid, rotated about about its vertical axis (/r > H/2):
its vertical axis (h > H/2):

€ I

Y/2<D Yl2=D
YIz<D yl2=D (liquid surface just
(No liquid spilled) Liquid surface just touching the top rim)
touching the top rim
(No liquid spilled)
-'-----------
t---_______-'1

I
ll
rl ;''l

Y/2>D Y = H2l2D
Y=H (with imaginary (vortex just touching the
Yl2>9 paraboloid above)
(Some liquid spilled) Vortex at the bottom bottom)
(Some liquid spilled)

For closecl vessels, there ca.n


be any liquid spilled, so the initial
of liquid (before rotation) is
s equal to the final volume of the
id (after rotation) or tht initinlaolunr
y>H air inside is equnl to the fnnl aolume of air
Vortex (imaginary) below y > H2l2D
the bottom The oolume of nir relation is more (Vortex below
(Some liquid spllled) ient to use in solving this type ol the bottom)

vr= Q/H\(y - K1; K= Hz/2D

"-l-
20 g Fl,#ff [;"?'Tium or Liquids
FLUID MECHANICS
& HYDRAULICS
FLUID MECHANICS CHAPTER FOUR
& HYDRAULICS Relative Equilibrium of Liquids 209
For closed cylindrical containers completely filled with liquid:
U-tube revolved about its own axis:
Note: the pressure head at any point
T-ir::-:-:l
I t\
in
the tube is the vertical distance from the

rivj
:l
Yl\. '
tube to the paraboloid. The pressure is

i', positive if the paraboloid is above the

" point and negative if it is below the


point. The limiting pressure is absolute

Without pre'ssure at top

For pipes and tubes:

t I

i
I I

i i
;
I
<: F-! I
I i
I
! i

I
!
I
I
I

'i I
I

l* *, --'l ' t'


rl-/

Without initial pressure inside With initial pressure inside


2lo CHAPTER FOUR
Relative Eguilibrium of Liquids
FLUID MECHANICS
& HYDRAULICS
FLUID MECHANICS
& HYDRAULICS
CHAPTER FOUR
Relative Equilibrium of Liquids 2tt
lS"lu"d Pr"bl.-t (c) When a=6m/sz
tan0!,a6=931
Problem 4 - 1
0 = 31.45'
An open rectangular tarrk nrouuted on a truck is 5 m long, 2 m wide and 2.5 m
high is filled with water to a depth of 2 m (a) What maximum horizontal x = 2.5 cot31.45o
acceleration can be tmposed or.r the tank without spilling any water and (b) x=4.0875<5m
deterrnine the accelerating force on the liquid mass? (c) If the acceleration ts V un = 1/z(4. 082 S) (z.s) (2)
increased to 6 m,/s2, lrow nruch water is spilled out? Vut= 10.22rrlf
Vorisinar = (2X2X5)
Solution
Voriginat = 20 m3
(al
, %piiled = Vortginar - Vleft
Vspilled = 20-1'0.22
Vspiled = 9.78 m3

Problem 4 - 2
A closed horizontal cyrindrical tank 1.5 m in diameter and
4 m long is
Ihe figure shows the water level under nlaxtmunr a when no water ts compl'tely-filled with gasoline (sp. gr. = 0.g2) and accererated horizontaly
at 3
spilled out rn/s2. Find the total force acting aithe rear'wall and at the front wall of
the
ta60= 9; ={) 2 tank. Find also the accelerating force on the fluid mass.

tano= ! =oz Solution


I
a = 0.2(9.81) tan0= 3
o
= 1,.962 m/s) 6 a=3m/s2
't
y_ 3
(1, )
4 9.81
Accelerating Force, F = Mn y =1..223 m
Mass, M = p(Voiun-re of liqurd)
Mass,M=1000[5"2"21 h =t.zSg+0j5
Mass,M-20,000k9 i : t.gzz rn
Acceierating Force, F = 20,000 x 1.962 F**=yi A
Accelerating Force, F = 39,240 N F*u, = (e.81 x 0.82)(1,.973)l.f
1r.sy1
F*u. = 28.05 kN
or;
F = F."o, rvall - Fhonr *atl ' i Fmnt= yii A
F = e.sl( ! ltz.stzll e 81(+ )tl.s(2)l Fho.t = (9.81 x 0.82)(0.75) t.f 1r.S1r1
F = 39.24 kN Fr.."t =10.66 kN
CHAPTER FOUR FLUID MECHANICS
2t2 Relative Equilibrium ot Lrqurds
FLUID MECHANICS
& HYDRAULIC5 & HYDRAULICS
CHAPTER FOUR
Relative Equitibrium of Liguids 213
Accelerahng Force Also:
[=Ma Tair (original) = Vair (finat)
Mass, M = p(Volurne) 4(0.2) (2) = (1. / 2) x z (2)
xz = 1,.6 + Eq. (2)
| = {(1000 ' 0 ri2)l+ (t s)r(4)ll (3)

I = 17,3q() N
t = 17.39 kN Substitute z and xz to Eq. (1)
aQ.6/ x) - 1.6 = 4.1x ) multiply by r
or
6.4 - 7.6x = 4.'t xz
rL_D.
-rredr-rlroilt
I)
4.Lx2+1..6x-6.4=0
F=28.05-10.66
f = 17.36 kN :ljj
X- --__-Jtt.u y2 - +1+.ry1-0.+y = 1.0695 m
2(4.1)
z = 1.6/1.0695 = 1.496 m
Problem 4 - 3
A closed i'ectangular tarrk 4 nr [ong, 2 m wide, and 2 m high is filled with
6.ylg =!8)c
=Z
water to a depth of 1.8 nr lf the allowable force at the rear wall of the tank is a 7.496
200 kN, how fast can it lre accelerated horizontally? t = 1-o6ed
a =13.72 m/s2 (horizontal acceleration)
Solution

4-4
1-
f-l open tank 1_82 m square weighs 3,425 N and contains
0.91m of water. It is
h
*j:::::?:tT::1,.*.:.of-ro,+00 N p","ua to a pair of sides. what is
force acting in the side with the ,*uffuriJ"ptnZ

P=yha
Solvefor aand,y:
F=Ma=1O400
p=yn a M= M*^t", f Munk
200=e.81 i1Z1Zy = .82)(1.82)(0.s1)l + 3,42s/s.81
Y 1,goot(t
M=3;363.42kg
lr =51rn
10,400=3,363.42x a
y= h I =4.1 nr a=3.092m/sz
By similar triangles
4-x .r , tan$= a !
= 0.91,
I
I
4.1 z
4z-xz=4.7t ) Eq. (i)
1.82mx1,82m
2l+ CHAPTER FOUR
Relative Equilibrium of Liquids
FLUID MECHANICS
& HYDRAULICS
FLUID MECHANICS
& HYDRAULICS
CHAPTEII FOUR
Relative Equilibrium of Liquids 2t5
3.092_ y AasE = 5.11 m2
9.81 0.91 VaM =5.11(1.5) = 7.665 m3
y=0.29m<0.91m(OK)
h=0.91-y
Ir= 0.62m Vrcn=12.4635 -7.665
P = 9,810 (0.62/2) 10.62 x 1'.821 V:".n= 4.7985 ms
P = 3,432 N %pined=8.6985 -4198s
%pilled = 3.9 m3

Problem 4 - 5
An open trapezoidal tank having a bottom width of 3 m is 2 m high, 1.5 m 4-6(CEBoard)
wide, and has its sides inclined 60o with the horizontal. It is filled with water A vesser 3 m in diametercontaining 2.4
to a depth of 1..5 m. If the tank is accelerated horizontally along its length at m ofwater is being raised. (a) Find the
pressure at the bottom of the vesser
in kpa when the
4.5 mf s2, how much water is spilled out?. (b) find the pressure at the bottom and
when it""I;iry;;;.1-,stant,
i, u."h"rui^ g 0.6 m/
upwards. "rirti'"*r"r sz
Solution
3 + 2(2 cot 60') = 5,399 t
3 + 2(1.5 cot 600) = 4,732 m For vertical motion:
p=yh(1xa/g)
a 4.5
tan0=t =9.81 Til
EI
It=2.4m

(a) When the velocity is constant,


0 = 24.64o .il 1.5 p=vh
4 = 0, then

p = e.81,e.a)
Vspilled=Vorig.-%eft p = 23.544 kpa (pressure at the bottom) ,

v*'r= lEZ (1.5)x 1.5


(b) When a = 0.6m/s2,(use', forupward motion)
Vo,ig = 8.6985 m3
p=e.81(2.4X1+(1**f)
\,,
V*n= Vasco - VaeE p = 24.984kpa
vAsco = 3rry92 (2x1.s)
Vasco = 12.4635 rfi
Varc: Aase(1.5)
4-7
Aale = (AB)(AE)sinl
1/z vessel containing oil is accelerated on a plane
- incrined 15. with the
cr=180o-60"-24.64' at 7.2 m/ sz. Determine the inclination
i3ontal of the oil surface when the
is (a) upwards, and (b) downwards,
= 95.35o
or
AE = sin 60o
AE = sin50' = 4.618 m
A as r = 1/z(5.309) (4.618) sin 24. 64'
CHAPTER FOUR
216 Relative Equilibrium of Liquids
FLUID MECHANICS
& HYDRAULICS
FTUID MECHANICS
& HYDRAULICS
CHAPTER FOUR
Relative Equilibrium of Liquids 2t7
Solution (c) Downward motion with a positive acceleration (use,,_,,with
a = +g m/s2)
AU p = (e.81x 0.s)(3) (t -
tan0= r*-)
Stav p =4.34kPa
All=OCOSd.
as = L.2 cos 15o (d) Downward motion with a negative accereration (use "-" with a = -grn/s2)
as = 1.159 nf sz p = (e.81. x 0.8)(3) U - Dt)
av= asincx p =42.74kPa
av = 1'.2 sin 15o
ay = 0.31, mf s2
4-9
(a) When the motion is upwards: A cylindrical water tank used in lifting water to the top of a tower is
1.759 1.5 m
tan0= high- If t{re pressure at the bottom of th*e tank is must not 16 Kpa, what
9.81+ (+0.31) ,ntaximum vertical acceleration can be imposed in the cylinder"x."ed
when it is filled
0 = 6.533' tvith water.

(b) When the motion is downwards:


1.1.59
tanH= P=yh(1+a/S)
e.81- (+0.31) 16 = 9.81(1.5)(1+ a/9.81)
0 = 6.955" a = 0.857 m/sz

Problem 4 - 8 4-to
An open tank containing oil (rp. g.. = 0.8) is accelerated vertically at 8 m/s2. open rylindrical vessel having a height equal to ib diameter is
half-filled with
Deterrnine the pressure 3 m below the surface if the motion is (a) upward with er and revolved about its own vertical uiis *ith a constant angular
speed of
a positive acceleration, (b) upward with a negative acceleratiorL (c) downward rpm. Find its minimum diameter so that there can be no riquid Jpil"a.
with a positive acceleration, and (d) downward with a negative acceleration.

Solution ttrat there's no liquid spilled, base of the


/ \ must just coincide with the
The pressure at a depth ft is given by,.p = *ltxZ) rim of the cylinder. Since the cylinder

(a) Upward motion with a positive acceleration (use "+" with


p = (e.81. x 0.s)(3) (t. #r)
a = +8 m/s2)
initially half-fuIl, the height of the
aboloid is therefore equal to the height of
cylinder. -l
ll'
P = 42'74kPa ,lt= _,2r2
2g
(b) Upward motion with a negative acceleration (use "t" with a = -8 m/s2 ) h=H=D
p = (s.81x 0.s)(3) (t . rt) or = 1.20 rpm x n/f,Q
P = 4'34kPa a = 4n rad/sec
2 t 8 [:fi,J5:;.?',Y#ium
FLUID MECHANICS FLUID MECHANICS CHAPTER FOUR
or Liquic,s & HYDRAULICS & HYDRAULICS Relative Equilibrium of Liquids 2t9
/ 2)) %pilled = Vairltinag - Vuirlinli"tl
,. (4n)2 \D
l,=
%piued = 1/ztr(0.6)2(1,63) - n(0.6)2(0.7)
2(e.81)
l) = 0.497 rn or 497 mm
Vsplled = 0.13 m3 x 1000 lit/ml'
%pired = 130 liters

-
Problem 4 - Ll Problem 4 - 13
An open cylindncal tank 1.6 nr tu dtameter and 2 m hrgh rs iull ol water An open cylindrical tank, 2 m in diameter and 4 m high contains water to a clepth
When rotated about its vertical axis at 30 rpnr wlrat would be the slope of tlrt
of 3 m. It is rotated about its own vertical axis with a c-onstant angular speed ol
water surface at the rim of the tank? .
(a) If ro = 3 rad/ sec., is there any liquid spilled?
Solution (b) what maximum value of ar (in rpm) can be imposed without spilring
any liquid?
Slope = tar.r 0
(r) If o = 8 radf s, how much water is spilled out and to what depth wilr
2
u)I the water stand when brought to rest?
SloPe = _-
I (d). what angular speed ro (in rpm) will just zero the depth of water
at the
rev 2n rad I mrtr center of the tank?
trr = -]0 (e) If or = 100 rpm, how much area at the bottom of the tank is uncovered?
min rev 60 ser

to = n rad/sec Solution
(n)2 {0.8) a212
Slone =
' 981 2g
(a) ro=3radlsec

Problem 4 - L2 (CE Board November 1978) /, =


(?): (l)'z 4
2(e.81)
An open cylindrical vessel 1.2 m in dian-reter and 2.1 m hrgh rs 2/3 full
h = 0.46
water Compute the anrount of water ilr liters that will be spilled out is
vessel is rotated about its vertical axis at a constant angular speed of90 rpm
h/2=0.23<7m
.'. no liquid is spilled out
Solution F
ir=1m
22
t\ t' ->
, The maximum <o so that there is no liquid spilled is such that
)o
-6 h/2=lmor/r=2m
ur= 90 rpnr ' n/3() .
^2r2
o = 3n rad/s 2g

,
n= 1sn.1210.0;2
" - ,'(1)'
2(9.81) 2(e.81)
/r= 1.63 m a = 6.26 rad/sec x 30
h/2 : 0 815 > 0 7 nr (sorne hqurd sprlled) to = 59.78 rpm
-
=0.6m
--A CHAPTER FOUR FLUID MECHANICS FLUID MECHANICS
ZZV Relative Equilibrium of Lrqurds & HYDRAULICS CHAPTER FOUR
& HYDRAULICS
Relative Equilibriurn of Liquids 22t
{c} r,r=8racl/ser Area,l=nyz ro = 100 rpm
Ir='
(8)2(1)2
€ y=5.59_4
=J26ur
2(e.81) .y = 1.58 m
h/2=1.63nr>lnr
By the squared property
some liquid spilled but the vortex of the of parabola;
paraboloid is inside tlre tank si4ce /r < 4nr
x?
-12
ysprllerl = Tarr (trral) - Vo,,
t,u,l,"l,
!/h
Varr (hnal) = Vparatrlor<l x, = * (1.s8)
Varr (rrnar) = L/z n(1)2 (3 2ol
x2 = 0.283
V.r (finat) = 5.I 21 nr '

Area, A = n(0.253)
l/arr{rrrrrrat) = tr(1)2(1) Area,A=0.889m2
Varr (rnrtrat) = 3.142 nt'

V;r,,re,r = 5.121 .a 142


Vspiloa = '/,,.979 m3
a:?
E5,
Another solution
When the tank rs brought to rest, the
water level will rest at /r/2 fronr top
v=h/2-1
v = 1.63 '1
r/=063rrr
V = n r) !
'r'11n'1
4piled = 7r (1)2(0.63) '

4pillecr = 7,979 m3
, a2r2
2g
(d) The vortex touches the bottonr when lr = 4nr
Solve for ft (by squared property)
" ,2 0\2
+=
2(e.8't) Q)2 (0.s)2
r,r=g.g6* # h h-4
h-4=0.25h
rrl = 84.6 rpm
0.75h = 4
lel When o = 100 rpnr /r = 5.33 m
t,r = 100(n/30) = 3.33n radlset
5.33 =
g-11'
,.: ot2r2 _ (3.33n)2(1)2 2(e.81)
2g 2(e 81 0) o = 10.23 rad/sec x !!
h=5.58>4nr a = 97.65 rpm
the vortex of the paraboloid rs already below the tank (imaginary)
CHAPTER FOUR FLUID MECHANICS
---
ZZZ Relative Equilibrium of Liquids
FLUID MECHANICS
CHAPTER FOUR
& HYDRAULICS & HYDRAULICS
Relative Equilibrium of Liquids 223
(bl Vten = Vc),linae, - Vf.urtuu, of paraboloid hz= pz/y
Vron= 7r (1)' (4) - [1/z n (1)2 (5.33) - 1/ztr (0.5)z (5.33 - 4)] lu= 74/0.0008
Vv* = 4.776 m3 hz = 17500 cm = 175 nr
h=hz-2.75-p1/ru
Problem 4 - 15 (CE Board November 1993) h=175-2.75-62.5
h ='109.75
A 1.90 m diameter closed cylinder, 215 m high is completely filled with oil
a2 (o'gs)2
having sp. gr. of 0.8 under a pressure of 5 kg/cm2 at the top. (a) What angular tog.zs =
speed can be imposed on the cylinder so that the maximum pressure at tl're 2(e.81)
bottom of the taflk is 14 kg/cm2? (b) Compute the pressure force exerted by oil co = 48.84 rad/sec x 30
ft
on the side of the tank in kg. o = 466.44 rpm

Solution (bl p=yn A


lmaginary L.S. tr=0.95 i = hz - 2.75/2 = 173.62s m
F =
800 (17 3. 62s)
[zn (O.es) (2.7 s\]
I [, l6v
F = 2.28 x 106 kg

I
I

!
I
4- 16 (CE MayigsD
F 'f
lT-
o{"" cylindrical tank having a radius
of 300 mm and a height of 1.2 m
full of water. How fast should i", U" is
o/"
,.i"i"i about irs own vertical axis so that
hz = Pzly
of its volume will be spilled out?

E
b
N
tri Oil (s! = 0.8)
, a2r2 o=7
2g
:e 75o/o of the total volume is
spilled out,
paraboloid will be formed a part
outside
vessel (i.e. with its vortex beiow the tank)
2n(0.95) = 5.969 m

%piled = V = 0.75[nrz(1.2)]
^"
V^i, = 0.9nr2
Unit weight of oil, y = 1000(0.8)
Unit weight of oil, y = 800 kg/m3 = 0.0008 kg/cm3
22 t V^ir= Tbrgparabolotd - Vsnrallparabolord
(al l, = '=' 0.9na = yz nr2 h _ 1/2 r.x2 y
z8 7.8f=r.zh_x2y >Si.(1)
Solve for lr :
Pt/r = 5/0.0008 = 6250 cm
fi/y = 62.5 rn
CHAPTER FOUR
I
224 Relative Equilibrium of Liquids
FLUID MECHANICS
& HYDRAULICS
FLUID MECHANICS
& HYDRAULICS
Rerative Ec,i,if"ryf;:?t;?il: ZZs
lly squared properiy of parabola:
a ='1.2.528 rad/sec x rea 60 sec
'J,

*2 ,2 ,2
2rrnd lmin
vhh co = 119.64 rpm
ln Eq. (1)
-x_
t2
1.8 P = ,'h - ; y(y) ) multiply both sides by h/ta
lt 4-18
1.81t=lP-t/2 An open vessel,500 mm.in diameter
and fillecl with water, is rotated
butY=1,-1., about is
1.8h=h2_(h_t.z\z fi':.TlTi;:il'-i ;,ll':y,*:ilt::$;;;..
angte of 40o with the hoiizo'tat. 100 r'm rrom the axis
C;;ilil:;".1':T;.;"il,1i: makes
1..81r = l? - (hz - 2.4h + 1.44) illl
0.6h = 1.44
h=2.4m The slope of the paraboloici
at any clistance ,,r,, fronl
the axis is given by:
Finally: tuno='2,
,210.3;2
I
2.4 = Where 0 = 40. and;r = 0.1 m
2(e.81)
a :22.87 rad/sec tan4oo=
x 30
T $fo.rl
trl = 218.4 rpm a = 9.07 rad/sec x !Q = g6.64rpm

Probfem 4 - 17 4-19
Ar"r opencylindrical tank 1 m in diameter and 3 m high is full of water. At open cylindricar tank 1.2 m in
what speed (in rpm) must it be rotated to discharge 1/3 of its content. diameter and 1.g m deep is
rotated
vs qvvqr
about rrb
its uwn filred with water
own axrs
axis at 50 rewnl,,r,^-. per
6O.revolutions --_, -
^^_ rninut,e.
and wllat is the pressure How
Solution at the center of its bottom? ".r"Jfiq.,ia
Let y be the height of the paraboloid.
ro = 60 rpm
since the volume of the paraboloid represents the volume of water spilled, then: , a2r2
2g
I st- r€:
Volume.of paraboloid = % Full volume of cylinder
co = 60 x (n/30) = 2n rad./ sec
Yz n (0.5)2 y =
t (0.s), (3)
"n r .2/2 = 0.6 m
='1
y =2m
r,= Qt-9.t=0.724m
,= #r .r=o5m
%pillea = Tparatroloid
., _ .2 10.5;2 %pilied = lz n2 It
2(e.81) V,spirhcr = Vz rc(0.6)z (0.724
)
%pilled = 0.409 m3

r=0.6m
CHAPTER FOUR FLUID MECHANICS FLUID MECHANICS CHAPTER FOUR
226 Relative Eguilibrium of Liquids & HYDRAULICS & HYDRAULICS Relative Equilibrium of Liquids 227
[)rcssure at the center: h/2=3.67m>1m
(part of the paraboloid is above the vessel)
P=vy
l/ = 1.6 - ft = 1.8' 0.724 = 7.076 r^t'r

p = 9.8"1(1.075) = 10.555 kPa Verify tl're position of the vortex (See page 207)
H2 _ $)'
2D 2(1)
Problem 4 - 20 n',2
A closed cylindrical vessel, 2 m in diameter and.4 m high is filled with water = 8nr>7.3{m the vortex is inside the vessel
2D
to a depth of 3m and rotated about its own vertical axis at a constant angular
speed, ol . The air inside tl-re vessel is under a pressure of 120 kPa. Il
Y nir (final) --
r,
Y arr (irlitidl)
(o) If io = 12 radf sec, what is the pressure at the center and circumference al lznxzy = nrz111
the bottom of the tank? x2y=2P )Eq.(1)
(b) What angular speed tu will just zero the depth of water at the center?
(c) lf a = 20 rad f sec, how much area at the bottom is uncovered? By squared property of parabola:
r-)1 r'
Solution
a= L2 radls
ylt
=
-> ^12 >Eq.(2)
I

i
It '
xtx
T--f- Substitute in Eq. (2) to Eq. (1)
I r'
12
t
( r' !t)v=2rz
u
!/2 = 2lt: 2(7.34)
|-
ml y=3.83m<4rn
| ,'. Pressure at the center, (at O)
pt=ylh+pait

I
ll ht=4-lt
lq=4-3.83=0.17rn
p=9.87(0.77)+tzO
I

Figure (a) fi= 121,.66 kPa (pressure at the center)

(al Refer to Figure (h) Pressure at circumference, (at O)


rt: -- "12 rad / s p2=ylh+ p^n
22 lh= h't + Ir
. oJr
lt = _
)o
Iu= g.t + 7.34 = 7.51 m
-d
pz = 9.81,(7 .51) + 120
(t2)2 (1)2 :7
,., -_ z,t pz= 193.67 kPa (pressure at the ci_r.curnference)
2(e.81)
CHAPTER FOUR FLUID MECHANICS FLUID MECHANICS
ZZA Relative Equilibrium of Liquicls & HYDRAULICS & HYDRAULICS
- CHAPTERFOUR
Relative Equitibrium of Liquiai ZZc)
(b) (c) or = 20 rad/ sec
' [,et us first derive the general value of ft
when the vortex of the paraboloid , ,2r2
reaches the bottorn of the vessel 2g

'Varr (final) = Varr (rntttall.


rt_ Qo).(\2
2(e.81)
L/znx2H = niD
, h=20.4m
x2H=2rzD )Eq.(b-1)
E
In Figure (d): tf
By squared property of parabola ct
V"i. N
=
xr22 liniai"ly Vai.
ni(7) = rTrn*tz yt - lznx2z y2
1finu4 lt
-c
Hh 2r2 = xt2 !/t - xz2 yz >nq.
2 1c_r;
r,='
It
H ) Eq. (b-z)
By squared property of parabola:
*t2 xr2 12

Substitute.r2 to Eq. (b-1)


atVzh
I

(Lr\H=zt2D ^1- - --h" V7 ) Eq. (c-2) Figure (d)


lh )
t2-^12
- llt ) Eq. (c-3)
Simplify: -n

Substitute rr2 and *22 to Eq. (c-7)


, = fteight of the paraboloid when it touches the bottom) 2
#
{
2
2t= + v,(yt- yr 0lz) multiply both side by h/r2
2h=y-r2-y2z
(4)2
h- -8m Butyr=4+ y
2(1)
n=@+yz)2-y2z
. ,2r2 2h=15+8yz+!22-y22
11 =
2g 8yz= 2(20.4) - 16; yz= 3.1 m
-
6 = '2(t)2 Eq. (c-3)
2(e.81)
a = 12.528 rad/sec , 30 ^12
X2'= U,
r
-hr-
or = 11.9.5 rpm
*n= =e.15,
#(3.r;
Area = nxz2= n(0.1521
Area = 0.48 m2 (area uncovered
at the bottom)
FLUID MECHANICS
23o ;i,:i'J5[;:,,Y*um or Liquids & HYDRAULICS
FLUIDMECHANICS
& HYDRAULICS
CHAPTER FOUR
Relative Eguilibrium of Liquids 231
Problem 4'21 Determine the position of the vortex:
A closed vertical cylindrical vessel, 1.5 m in cliameter and 3.6 m high is 3/a H2 rc.G\z
7.2rn
full of brine (s = 1.3) and is revolved about its vertical axis with a constant 2D 2(0.e)
=
angular speed. The vessel is made up of steel 9 mm thick with an allowablt'
tensile stress of 85 MPa and has a small opening at the center of the top cover Since /r = 13.86 > 7.2, the vortex is below the
vessel, See Figure (b)
(a) If the angular speed is 2L0 rpm, what is maximum the stress in the walls?
(b) To what maxirnum angular speed can the vessel be revolved? V"ir 1iniu"l1 = Vair 1nnat1
nr2(0.9) = Yznxf lt - l/znx2z y2
Solution 1.8 rz = xr2 yt - xz2 yz ) Eq. (1)
(a) a=210rpn"rxnl30
By squared property of parabola:
a=Trradfs
x'r2 xr2
^=_=- ,2
S, =
PD Note: The maximum pressure ls at the circumference at the bottom Ur !/z h
2t
= lt,in" I r=0.75m ^12
P h'r
xt'=
T lt ) Eq. (2)

G'

il
^12-;
x2'= y2 ) Eq. (3)
r=0.75m
Substitute rr2 and rz2 to Eq. (1)
'
't.8r?=
-.2 ,2
r rr(rr)-
|V,0) ) multiply both sid.es by h/p
1..8h=yf_y2z

But yr = 3.6 + y2
1.97=(3.6+yz)z-y22
1.8h= 12.96 + 7.2y2+ yz2 _ y22
7 .2 yz = '1,.8h - 12.96
) Eq. (a)

111 7.2y2 = 1.8(13.8d) - 12.96


lz= 1.665 m
Figure (a) Figure (b)
lu: 1g.8G - 7.66s
h=12.195rr.
p = e.81, (1,.3) ('I2.1es)
Solve for hr:
p = 155.52 kPa
h=h-Vz
-
s,=
(L55.52)(1500)
i ,\ /
, ,2r2 (7n)7 (0.75)2 2(e)
2g 2(9.8"1) St= 12,960kPa
/r = 13.86 m &=12.96 MPa (maximumwall skess)
CHAPTERFOUR FLUID MECHANICS
-t-5 Z
Z
FLUID MECHANICS CHAPTER FOUR
Relative Equilibrium of Liguids & HYDRAULICS & HYDRAUTICS Relative Equilibrium of Liquids 233
(lt) Iror nraximum value of trl, Sr = 85 MPa
e.81(1.3)h] (1500) 82x103=
85 x 103
2(e)
pz:455.5 kPa
h1 = 79.98 m
y2=h-79.98 Pz = Ylt2
455.5 = 9.81(L.6)h2
In Eq. (a) ln= 29.02n..
7.2(h - 79.98) = 7.8h - 12.96 h=lu-pt/v-2.7
5.4h = 562.896; h ="104.24 rn
22
h: 29.02 - 2.7 - 15.61

trr = 9-I-
h = 10.77 m
1
,/o
-6 - ,rtz (0.g\2
t'= =10'77
2PJ'L,
104.24 = '2(o'zs)'
2(e.81) to = 16.1 rad/sec x 39
o = 60.3 rad/sec " 30 / n co = 1.53.8 rpm (maximum allowable angular speed)
o = 576 rpm

4- 23
Problem 4 - 22
1..5 m diameter impeller of a closed centrifugal water pump is rotated at
A 1.8 m diameter closed cylinder, 2.7 m high is completely filled with glycerirr rpm. If the casing is full of water, what pressgre is developed by
lraving sp. gr. of L.6 under a pressure of 245 kPa at the top. The steel plakrr tion?
which form the cylinder are 5 rnm thick with an ultirnate tensile stress of 82
MPa. How fast can it be rotated about its vertical axis to the point of bursting?
p
Solution Pressure head,
- = il
v
,
tl = _
a2r2
, _a2r2
)o tt-
'Solve for
-6
.29 r=0.75m
/r
, o - 1500xn/30
Pt- 245
ro = 50n rad/sec
y e.81(1.6)
(sol)1(o,zs)2
rr
h-
hz = Dzlt ,, = =707.4rn
= 15.61 m 2(e.81)
Y

=707.4mof water
The maximum tensile stress occurs *n
r 5,940 kPa
at point @:
pD
From Sr =
2t
CHAPTER FOUR FLUID MECHANICS FLUID MECHANICS
234 Relative Equilibrium of Liquids & HYDRAULICS & HYDRAULICS CHAPTER FOUR
Relative Equilibrium of Liquids 235
Problem 4 - 24 (CE Board) Solution
A conical vessel with sides inclined 30' with its vertical axis is revolved about Pz=vlh t
another axis L m from its own and parallel. How many revolutions per
!

minute rnust it rnake in order that water poured into it will be entirely Solvirrg for /r2:
discharged by the rotative effect? lu=yz-yt i 0= 27.5 rad/sec
)1
,lI2= o-x^' I

Solution --3- - I
2g I

.
I
Yz

The water in the vessel will entirely be t,,= Q7.sf I

VOf. [(2.s), - (0.s),] I

discharged at a speed when the


I
I
I
paraboloid is tangent to the cone at the
!
lu= 23L.27 m ,+ 0.5 -+l
vertex, hence, tfie inclination, 0, of the
i
iol zZm
paraboloid at r = 1 m is 60o or its slope is 3Oo
i
i 30. p = (e.81, x 0.s22)(237.27) ::-r-=1:_: --:: tr_
tan 60o. p = 7,,865kpa i*-- _____t r.,." 1

From the formula:


-12 4-26
f2n$= i rr ,/i glass u-tube whose verticar stems
o
-1d f-------7-i are 300 mm apart is fited with mercury
i/:/ verrical ;,-"*"- ii
ugh the midpoint i,p
2
is rotatea about a vertical axis
tan 60o = *9.81 (r) -- t/ :*T,1,:ti,rri:: of the horizo.,,ur ,".uor,];il;i#il:";:::.:
'.
--i-" luce a pressure of absolute
zero in ;i";;;y ill;
a=4.12rad/secx 39 i at the axis?

ro = 39.36 revolutions per minute


a2 *2
"29
Problem 4 - 25 (CE November 1992)
ll=1t,,,+9.15
A 75 mm diameter pipe,2 m long is just filled with oil (sp. gr. = 0'822) and r=0.15m
then capped, and placed on a horizontal position. It is rotated at27.5 rad/str€
about a vertical axis 0.5 m from one end (outside the pipe)' What is tht btnce the pressure at the center
pressure in kPa at the far end of the pipe? is absolute zero, then the gage T
0.15
pressure at the center is _pot. or
-760 nrmHg, therefore lu,'
= [.7U
.L
f
^ h,
Y=0.75+0.75 I
y=0.91 m
g.91 <o2(0.t5)2
=
2(e.81)
a= 28.17 radlsec x 30

<o = 269 rpm


CHAPTER FOUR FLUID MECHANICS
236 Relative Equilibrium of Liquids & HYDRAULICS
FLUID MECHANICS
& HYDRAULICS
RerativeEq,i,,ff,;1::1;fl:T 237
Problem 4 - 27 $olution
A glass U-tube whose vertical stems are 600 mm apart is filled with mercury d-
to a depth of 200 mm in the vertical stems. It is rotated about a vertical axis \>u, :
lhrough its horizontal base 400 rnm from one $tem. How fast should it be I

rotatecl so that the difference in the mercury levels in the sterns is 200 rnm?

Solution

xz = 0.4m
I
_______+k_xr T
I
I
ht

Initial mercury
level

0.2 m

tce there is no liquicl spilled


out, its initial volume is equal
lu're or as show^ in the hgure, to its final
ms before and after rotatio]r
th"'.;;;;'arL nurgn, of water in the vertical
In the figure shown: ,"rrt t .""'
1/z- lr = 0.2 " "lu'uf
- ,2*2 4r+4r=5(0.5)
where 1..25
" 1n Yt + yz = ) Eq (1)

a2 *22
- ,'*"r2 = 0.2
. By squared property of parabola:
2g 2g Q)2 (o.s)z
,2 Uz Ut
[(0.q)z- (0.2)r]= 0.2
4r.81) !12= 4Yr ) Eq. (2)
a = 5.72 rad/sec x 30T
Substitute y: to Eq. (1)
ro = 54.6 rpm y1+ 4y1= 7.25
y = 0.25
(1)2)I"-
Problem 4 - 28 "29
l/i = ------:-
A glass tubing consist of 5 vertical sterns which are 500 mm apart connected to
a single horizontal tube" The tube is filled with water to a depth of 500 mm in g.2u (o.s)2
= ^2
the vertical stems. How fast should it be rotated about and axis through the 2(e.81)
middle stem to just zero the depth of water in that stem? a = 4.43 rad/sec x 30
n
or = 42.3 rpm
CHAPTER FOUR FLUID MECHANICS FLUID MECHANICS
238 Relative Equilibrium of Liguids & HYDRAULICS & HYDRAULICS
CHAPTER FOUR
Relative Equilibrium of Liquids 239
Problem 4'29 Mirrimize /r', differe'riate Eq. (1) with respect to
-i- arrd equate to zero:

A 75 rnm diameter pipe, 2 m long is filled with water and capped at both ends :dlr = 2.5n8* - tan 60o = 0
It is held on a plane inclined 60" with the horizontal and rotated about a
vertical axis through its lower end with a constant angular speed of 5 rad/sec
r = 0.68 n'r
a = .r sec60"
(a) Compute the pressure at the upper end of the pipe and (b) determrne the
n = 0.68 sec60o = I .36 nr
minimum pressure and its location in the pipe
In Eq. (1):
Solution
initial Pressure
ll = 1.27aQ.68), + t.Zg - (0.68) tan60o
Since there is no h' = 1.747 nt
in the pipe the pressure head at
p.,u = 9.81(1.141)
the lower end of the PiPe will p,ri,,=11.196 kPa Iocated 1.36 m frorn the lower.errcr
remain equal to the static (arong the pipe)
pressure head of "1.73 rn, and
therefore the vortex of the
paraboloid will be 1.73 m above
4-30
the lower end. cyli'drical bucket 150 mm in diameter and 200 mm
high contains 150 mm of
rter: A boy swings the bucket on a vertical plane so that
, ,2r', :ket describes a circle of radius 1 m. How fast
the bottom of the
)o should it be rotated so that
water will be spilled?
,
,t=
(s)2(1)',
+
2(9.81)
O.7mA
h = 1..274 m
*-
(a) Pressure at the uPPer end

= Ih /v
Pus,y*,
pupper = 9.81(1.274)
1r:
,r\ .
s
c
puppe, = 12.497 kPa | \"\. -\'
.i i\.. il
/ ''' ":'- .."i.
| -- .....

(b) Minimum pressure *


pn r, = flt' I
I
\
Solve for ft' ,\
h'--Y+2 \ l::i '-i
i
:

z='1.73-rtan60o \ rri':-]i
t...
,-' ,2 1.2 '!.2 \tf/--:!t Figure (b)
y lt L.274 \ ---l
Figure (a)
a = 1'274 xz
h' ='1.274 x2 + (1.73 - x tan60") ) Eq. (1)
CHAPTER FOUR FLUID MECHANICS
240 Relative Equilibrium of Liquids & HYDRAULICS
FLUID MEcHANIcs
& HYDRAUTICS
CHAPTER FIVE
Fundamentals of Fluid Flow 241
l he critical position for the liquid to fall is at the highest point

From Figure (b).


CF =W
CF=Mn,.
Chapter 5
w
CF=-rrr2r
Fundamentals of
I Fluid Flow
W
-f, ul) r=W
The previous chapters only with fluids at rest in
t.r2r=g which the only
tigruficant property used-dears
is the weight or *re Ruia. Thrs chapter
(0.925) = e.8l
--iit aua *itt
(02
fluids in motion which is based on ihe fo'owing-
ot = 3.26 rad/sec J9 principres: (a) the principle
" eonservation of mass, (bl of
r'r = 31.13 rpm rnergies), . feof-energy principle
and (c) the principle
(the' kinetic ani potential
-oril.,i rr.,

Problem 4 - 3l
A cubical tank is filled with 2 nr of oil having sp. gr of 0.8. Find the force oR FLOW RATE, Q
acting on one side of the tank when the acceleration is 5 m/s2 (a) vertically harge or flow rate is the amount of fluid
passing through a section per
upward, and (b) vertically downward as a max ftur,.,i,
*:::.::l:"'"q
.:,{H:; and aolume
kN/sec), rate or
floto iaten(ex.
floto
1u*"ug1,",i,,,,J,t 7o,, rate
m3/s, litls)
Solution
(al F= p,rA Volume flow rate, e = Aa -- .-- E+ 5- 1
F = fy lurQ + (g)) A Mass flow rate, M= p e
F = (e.81 x 0.8)(1)(1 + 5 / e.81)[(2)'?ti Eq.S _2
F = 47.392 kN Weightflow rate,W=y e

where:
tbt F= p,rA
F = [7lr.r(1 - a/9] a Q = discharge in m3/s or fF/ s
2m A = cross-sectional area of flow in m2 or
F = (9.81 0 8)(1)(1 - 5/e.87\(2x2) ft2
" z, = mean velocity of flow
in m/s of ft/ s
F = 15.392 kN 'p = mass density in kg/mr or
slugs/fts
y = weight density in N/mr
or lb/fF

OF TERMS
Flow.
1a1 be steady or unsteady; uniform or
' or turbulent; n on-untform ; continuou s ;
one-dimensional, ftio_dimensional
or three-dimensional; and
or urotatronal
CHAPTER FIVE FLUID MECHANICS FLUID MECHANICS
CHAPTER FIVE
Z+Z Fundamentats of Fluid Flow & HYDRAULICS & HYDRAULICS Fundamentals of Fluid Flow 243
Steady Flow Turbulent Flow
This occurs when the discharge Q passing a given cross-section is constant with The flow is said to be turbulent when the path
of individual particles are
time. If the flow Q at the crosi-section varies with time, the flow is wtsteady. irregular and continuously cross each other. Turbulent
flow normallv occurs
when the Reynolds number exceed 2,100, (although
the most .o-*on
situation is when it exceeds 4000).
Uniform Flow
Laminar flow in circular pipes can be maintained
This occurs if, with steady flow for a given length, or reaclr, of a stream, the up to values of & as high as
50'000' However, in such cases- this rype of flo* i,
average velocity of flow is the same at every cross-section. This usually occurs i"r,"r"rrirf ,r*tabre, and
the least disturbance wilr transform ii instantly
when"an incompressible fluid flows through a stream with uniform cross- into turburent flow. on the
other hand, it is practicaily impossible for t'rbul""in.*
section. In stream where the cross:sections and velocity changes, the flow is * pi;;
said to be non-uniform.
persist at values of & much below 2100, because "'r""lgn,
any turbulence that is set up
will be damped out by viscous friction.

Continuous Flow
One-Dimensional Flow
This occurs when at any time, the discharge Q at every section of the stream
This occurs when in an incompressible fluid, the direction
the same (principle of conseruation of mass). and magnitude of
he velocity at all points are identical

Flow
occurs when the fluid particles move in planes
or parallel planes and the
mline patterns are identical in each planei

Continuity Equation
or incompr essible f luids: are imaginary curves drawn through a fluid
F to indicate the direction of
r in various sections of the flow of the fluid
system.
.l Q=A{ut=Azuz=Atat=constant Eq.s-a I

tubes
F or co mp re ssible fluids:
represents elementary portions of a flowing
fluid bounded by a group
Itreamlines which confine the flbw.
PrArar - PzAzaz= Py'r:roI = constant Eq' 5 - 5
arhA;rr\= y2A:llr;z* lsAs:oe = constant n+ !: 6

Nets
are drawn to indicate flow patters in case
Laminar Flow of two-dimensional flow, or
three-dimensional flow
The flow is said to be laminar when the path of individual fllrid particles tlo
not cross or intersect. The flow is always laminar when the Reynolds nurrtlre!
R" is less than (approximately) 2,100.
CHAPTERFIVE
-,,
Z++ FLUID MECHANICS FLUID MECHANICS
Fundamentals of Fluid Flow 6. HYDRAULICS & HYDRAULICS CHAPTER FIVE
]
Fundarnentals of Fluid Flow 245
ENERGY AND HEAD Pressure Energy (potential
Energy)
'l'he energy possessed by a flowing fluid consists of the kinetic and the potential
consider a crosed tank filled with
a fluid which,has a-small opening
energy. Potential energy may in turn be subdivided into energy due to the top at
position or eleuation above a given datum, and energy due to pres;sure in the
,H:Yi.l:::'."1" "r,,1"
top,
ihe n"ia-pr".u.aily will
w'r not tlow.
flow. In ch"-r.*
rn Chapter .>
the equivalent head (pr"rr.r;1;;;; tvt o pr€ssure of p it 2,
fluid. The amount of energy per pound or Newton of fluid is called thelrcad. pt"rrrrruenergylseqivalent-a,"-**' i;;:;""uv p/y Hence the

Kinetic Energy Pressure Energy = Wl- Eq. 5 _ 12


The ability of the fluid mass to do work by virtue of its velocity: v
PressureEnergy
Pressure head = _ _
_;p Eq.5_13
K.E. =1/z M az = yrATp Eq.5 -7 W
I where:
z = position of the fluid above (+) or below (_) the datum plane
Kineticorvelocityhead"= +w29* = Eq.5-8 p= fluid pressure
r) = mean velocity of flow

For circular pipe of diameter D flowing full: Flow Energy, E


,2 _(e/A)2 = e, total energy or head in a fluid
flow is the sum of the krnehc
29 29 2gA2 rtial energies. It can be summarized
as:
arrd the

u') q_
Total Energy = Ki""fi
-=
)o
zs(iD ,Y

o' 8Q''
TotalHead, ,= Eq.5_15
=
= Eq"s-e *. f,.z
2s WF
AND EFFICIENCY
Elevation Energy (Potential Energy) lver is the rate at which work
is done. For a fluid of unit
I moving at a rate of e (m3ls) with weight r (N/m.ti
The energy possessed by the fluid by virtue of its position or elevation with a totJ eiergy of E (m), the power
respect to a datum plane. e floule/sec) or watts is: in N-

Power = ey E
Elevation Energy =Wz = Mgz Eq.5-10 Eq.5-16
Elevation-Energy Efficiency, OttPttt
Elevation head = - , Eq.5-11 ,., = x 700% Eq.5 -77
w lnput
Note; 1 Horsepower (hp) = 246 Watts
1 Horsepower (hp) = 550
ft_lblsec
1 Waft = 1 N_m/s = 1
Joule/sec
CHAPTER FIVE FLUID MECHANICS FLUID MECHANICS
Z+O Fundamentals of Ftuid Flow & HYDRAULICS & HYDRAULICS CHAPTER FIVE
Fundamentals of Fluid Flow 247
BERNOULLI'S ENERGY TH EOREM Energy Equation with Head Lost:
l'hc Bernoulli's energy theorem results from the application of the pnnuples ol Considering head lost, the values
that we can attain are called
r'ottsen,ntion of energrl. This equation may be surnmarized as follows: With reference to Figure 5 _ 4: nctual oqlttes.

Bernoulli's Principle, in physics, the concept


that as the speed of a moving fluid (liquid or
gas) increases, the pressure within that fluid
decreases. Originally formulated in 1738 by
Swiss mathematician and physicist Daniel
Bernoulli, it states that the total energy in a
steadily flowing fluid system rs a constant
along the flow path. An increase in the fluid's
speed must therefore be matched by a
decrease in its pressure.

nerr.cgdgg",ecl
\
Eat sechon I* Eadctecl - Elost or extlacted = Eat section 2 .5-18
f'*
| -r- -.
Pll
Energy .Equation without Head Lost:
If the f-luid experiences no head lost in movrng from section 1 to section 2 tht'rr
.l
the total energy at section must be equal to the total energy at section 2
Neglecting head lost ir-r fluid flow, the values that we get are called ideal ot
theoretical ualues. With reference to Figure 5 - 3:
5-4
Z1
Et: Ez Eq. 5 - 1()
a
tr'12 Pt 0o
+a= 2
+22 Eq.5 - 20 22
2g 2g v
-f
Datum
_t
Energy grade Line, EGL
v12l2g Equation with pump:
F* vz'}l29 basicany to increase the head. (usualry
.tr "::o- to raise water from
I

tlq"l.oo,u, (p,^*,) of the pump is erectricara


I

Qrlt
I
1jr1_t*::T::::1,Tn"
and its output power (p.u,p,,) is the
flo* #;gy
+
I

PzJt

Zt

Figure 5 - 3
L-
l

_D1tg
Z2

+
248 Fl#"tiii,',",for Fruid Frow
FLUID MECHANIC5
& HYDRAULICS
FLUID MECHANICS
6. HYDRAUTICS
' CHAPTER FIVE
Fundamentals of Fluid Flow 249
Et+ HA-HL|2=E, Eq.5-23 Characteristics of HGL

'. HGL slopes downward in


. P1 + 21+ HA= ?' *
+29v2gr P2 + zz+ Hh-t Eq.5-24 the direction of flow but
to changes in velocity or pressure. it may rise or falr due

Output Power of Pump = Q y IU .5-25 : X::;11l:::ttry cross_section,;;;;d;;il;""..i#1p,".,o,"h"ud.


HGLisparailel to the EGL
i:n:1':ly,*,n::.,1""if
anv two points is urro uq,rui-io;:T"1Tiil
Energy Equation with Turbine or Motor:
i:ffi*
points. ?:"ff::: ;:::
Iurbines or motors exh'act flow energy to do rnechantcal work which in turrr
converted into electrical energy for furbines Grade Line (EGL)

:lffti]l:ff"fi :i:.#.;:J,txi;:"..:.l:fix,"Hffi
+ !_ +2.
;lffJ,,""
v

cteristics of EGL
a
direction or now, and it w*r onrv
iT?{'ilini;i"i:,:::iff;1,'; 'n"
a The drop of the EGL retween I any two points is the head
those points. lost between
For uniform pipe cross_section,
EGL is parallel to the HGL.

h - HE - HLr.z= Ez Eq.S - 26 ES! * always above the \tsL


HGLbyvy an amount equal to the velocity head,
u2/2g.
!r' * Pt * r,= ,r^2
-if *
Dt
* zz.+ HLn+ HE Eq.S - 27 NeglecHng head loss, EGL
is horizontal
2g v ;
ut Power of Turbine = Qy HE
HLz
vqzl29
_lq=
HLv
i,r__
-Frcg
ENERGY AND HYDRAULIC GRADE LIN-ES T--
II ;l; --r\
i-i-r---->- HLs

HE
lrr r\
i-*1
I

Hydraulic Grade Line (HGL)


I

p/y Pa/yrrl HLn


!il
ril
Also known as pressure gradient, hydrauhc grade lme rs the graphrcal
vn2/29
gepresentation of the total potential energy of flow. It is the line that conrru,tl
the water levels in successive piezometer tubes placed at intervals along tlu
Valve
i e IURBINE h
pipe. lts distance fronr the datum plane is 4 + z I
v Illustration showing the
behavior of energy and
hydraulic grade lines.
CHAPTER FIVE FLUID MECHANICS FLUID MECHANICS CHAPTER FIVE
Z5U Fundamentals of Fluicl Flow & HYDRAULICS & HYDRAULICS Fundamentals of Fluid Flow 25t
Solution
Solved Problems W= yQ=yAa

Problem 5 - 1 ,=
,RTP
Water flows through a 75 mm diameter pipe at a velocity of 3 m/sec. Find (a)
110 x 103
the volume flow rate in m3/sec and lit/sec, (b) the mass flow rate in kg/sec, ' 1= 2%(go+273) =12'39N/mr
and (c) the weight flow rate in N/sec.
20 = 72.3e10.1 6 (0.32)lu
Solution o = 31.53 m/s (average velocity)
(a) Q=Au Q= Aa
= -i (0.075),(3) (0.16X0.32X31.s3)
= 0.013 m3/s x '1000 lit/mj a = 1*674 m3/sec (volume flux or discharge)
Q = 13lit/sec

(b) M=pQ 5-
= 1000(0.013) 4.110-.1y" diameter plunger is being pushed at 60 mm/sec
into a tank filled
M = 13 kglsec (mass flow rate)
11ltis :|,l11lq q o.f 0
::. rf tne fluia is incompressibte, how many N/s of
"f out at a 30-mm diameter hole?
being forced
(c) w=vQ
= e810(0.013)
W = 1,27 N/sec (weight flow rate)
the fluid is incompressible:
Qt= Qz
Problem 5 - 2 (CE November 1995) Qr = AtVt
What is the rate of flow of water passing through a pipe with a diameter of 20 = t (0.1)'z(0.06)
mrn and speed of 0.5 m/sec? Qr = 0.00047 rfi/ s

Solution Qz = 0.00047 m3 / s

Flow rate, Q= At, w=yQ


Q= f (0.02)'z(o's) = (9810 x 0:82)(0.00042)
W= 3.78 N/s
Q = L.57 ,, 1g_r 6:/sec

Problem 5 - 3 5-
the velocity of flow in a zS-mm diameter fire hose is 0.5
Air at 30'C and 110 kPa flows at 20 N/s through a rectangular duct tlrill m/s, what is the
locity in a 25 mm diameter iet issuing from a nozzre attached
measure 160 mm x 320 mm. Compute the average velocity and volume fltrx at the end of
pipe. Compute also the power available in the jet.
Use gas constant R = 29,3 m/'K 3
-F- CHAPTERFIVE FLUID MECHANICS FLUID MECHANICS
Z5Z Fundamentars of Fruid Frow & HYDRAULICS CHAPTER FIVE
& HYDRAULICS Fundamentats of Fluid Flow 253
Solution Solution
lly continuity equation: PE=Wz
Qnnr" = Ql", W=y * Volume
An at, = Ai at
=9.81,x f,(s)r(19)
. t (0.075)2 (0 s) = f, (0.025)2 z',

oi= 4.5 rc/s (velocity of the iet)


w- 1,925.2kN
PE='1.,926.2x7
PE = 13183.32 kN-m
Power, P = QV E

Q= Au
' Q = + (0.025)'z (4.5) = 0.002209 rn3/ s
Determine the kinetic energy flux of 0.02 m3/s of oil (rp. gr. =
,r2 L \2
E=:-
2s 2(e.81)
discharging through a So-mmtia*",*
;";;i". 0.85)

Power, P = 0.002209(9,810)(1.0321) Solution


Power, P = 22.37 walts (power available in the yet) Kinetic energy flux = Kinetic Energy per
second = power
Power,P=QyE
Problem 5 - 6 A * 0.02 m3/s
A turbine is rated at 600 hp when the flow of water through it is 0.61 m3/s ^a2
Assuming an efficiency of 87%, what is the head acting on the turbine? 2g

Solution ,= IA - o'02

Given: Power output = 600 hp


t(0.05)2
a = 1.0..1.96 m/ s
EfficiencY, 11 : g7o1'
(10.186)2
^
ts=-:____-, =5.2ggm
600 2(e.81)
Power input
' = - 589.555 ho
f P * 0.02(9810 x 0.8s)(5.288)
0.87
Power input = 514,483 watts P = 882 watts

Power input = Qy HE
514,483 = 0. 61 (9,81 0)HE
HE = 85.97 rn Iecting air resistance, d"::rTiT to what height
'-o- a vertical jet of water could
if projected with a velocity of 20 m/ s?

Problem 5 - 7
A standpipe 5 m in diameter and 10 m high is filled with water. ialculate tht'
potential energy of the water if the.elevation datum is'taken 2 m below the
base of the standpipe.
25 4 Fl#.ti:i,'"",f or Fruid Frow
FLUID MECHANICS
& HYDRAULICS
FLUIE MECHANICS
& HYDRAULICS
CHAPTER FIVE
Fundarnentals of Fluid Flow 255
Solution Problem 5 - lt
As the jet rises, its krnetic erlergy is transformed into potential energy A pip" oil of specific gravity
0.g77 changes in size from 150 mm
;arrying at
:: .l-llffi- section 1 and 450 mm at sectiori2.
Neglecting ait resistance: section 1 is 3.5 m berow section 2 and
the
KE_PE
d,
r-l l
pressures are g0 kpa and 60_tpa,respectivery. If the dischargJl,lso
determine the head lost and the direction;ifir*. [t/sec,
1/2M7'1
L/
trr'"
=Wh
$l
$n
$OlUtion a
4s0 mm O
450

;t
,,1 =yy71 = 60 kPa
8 Qr= Q, = 0.15 m3ls

It='
,,2
l* 'l "t
=
0.15
:-::--:;
(0.1s)z
=8.49m/s
)o
i f,
I

-6
','l I 0.15
.20\2 a'z= =o'e43m/s
Ir= ' ' =20.4m
TQ'45f
2(e.81)
o
' Pr=gokPa
Taking O as datum:
Problem 5 - 10 E,=++Pl +zr= g.492 + 90
Water is flowing in an open channel at a depth of 2 mand a velocity of 3 n"r/s 29v 2(9.s1) (e.81x 0.8??) +0
It flows down a chute into another channel where the depth is 1 m and tht' Er = 14.L35 m
velocity is 10 m/s. Neglecting friction, determine the difference in.elevation ol
the channel floors o"2
-
E2= Do
---:- + J-:- + z-
z8y
Solution
r-f _ 0.9432
ilrl T-r,fe
60
2(9.81) (9.81x0.877) ' "'"
Ez='1.0.62m

I v22l2g Since Er > E2, the flow is from L to 2

to
lt
.irl,
Head Lost, HL = Et - Ez = 14.135 -'t0.62
Head Lost, HL 3.515 m
=
I

Neglecting friclion ----i --li


--t+
I

(head lost): - iA J1'n


E't = Et
-2
(Jt
I
-.2
U.L
rAr-- L1
)o )o
-d -d

32 aL--I
lo2
- 11
2(e.81)
I L L_

2(e.81)
z = 3.64 rn
CHAPTERFIVE FLUID MECHANICS
^F '
Z5O Fundamentats of Ftuid Ftow
FT-UID MECHANICS
CHAPTER FIVF
& HYDRAULICS & HYDRAULICS Fundamentals of Fluid Flow 257
Problem 5 - L2 El. 30 m
M= p2A2a2
()il flows from a tank through 150 nr
oz[(0.3)(0.3)] ( 2) = 0.7s75
of 150 mm diameter pipe and then Pz = 0.875 kglmt (mass density at section 2)
discharges into air as shown in ti-re N
I.igure. If the head loss frorn point 1 E
E
to point 2 is 600 mm, determine the o
El.20 5-
pressure needed at point 1 to cause I
Iter flows at the rate of 7.5 m/ s
17 lit/ sec of oil to flow E
o
through Z5_mm diameter pipe (pipe 1) and
h
diameter
and 65lmm diamet", pipes at the rate
Solution /sl":jlTX*,U^O respectivery u, ,.o*^l'tth;;;';; nxt=.'"i'dl",op of 3
and 3'5 m/s, -l*
of the
Q:0.017 m3/5 *,ff.:rv"",: *:.:ygl .l,s9rmm_aia1".",:.r,".,.. cqlculate dtt/dt and the
of air flow through
Energy equation between O and @
the vent. Assume thu flo_=;;;;,"..*rl"Slr"
Et - HL1-r= 2,
r,r2 * Pt ('22
+ zt - HLt t: * P, ,
zgyZgy Z2

0+ "Dt +20-0.6 = 8(0.017\2


;' ' ; +0+ 30
Y n'g(0.15)'
Pt
10.65 m of oil
v v2=3m/s
P1 = 10 65(9.81
' 0.84) = 87.76kPa
vr = 3.5 m/s
Q:

Cas is flowing tluough a square conduit whose sechon gradually changes frorrr
150 mm (section 1) to 300 rnm (section 2). At section 1, the velocity of flow is 7
Assuming the flow to be incompressibie:
m/s and the density of gas is 1kg/mi wlrile at section 2 the velocity of flow is 2
m/s. Calculate the mass flow rate and the density of the gas at section 2. Q'n = Qou,
t (0.075)2(7.5) = f (0.0s),(3) *f (0.065)r(3.5) +
Solution dlt/ dt = 0.0553 m/s
! (0.6), dh/ dt

p) = I kg/m' dM '' 300 mm Considering the air above the tank:


150 mm
e
E [Qn = Q"i.]
o
f, (0.05)zu = 4L
(0.6)2 dh/ dt

f, (0.05)2 ut =
* (0.6)r(0.0553)
M=pQ
M = prA'ttr, a+ = 7.963 m/s (velocity of air flow)
= 1[(0.15X0.1s)](7)
M : 0.1575 kg/sec (mass flow rate)
FLUID MECHANICS
258 Fffi#H'Y"torFruid Frow & HYDRAULICS
FLUID MECHANICS
& HYDRAULICS
CHAPTER FIVE
Fundamentals of Fluid Flow 259
Problem 5 - 15 Sqlution
A liquid having sp. gr. of 2.0 is flowing in a 50 mm diameter pipe. The total Q, = Qz = 0.03 m3ls
head at a given point was found to be 17.5 Joule per Newton. The elevation ol u2 ge2
the pipe above the datum is 3 m and the pressure in the pipe is 65.6 kPa 29 n'gDn
compute the velocity of flow and the horsepower in the stream at that point.
ur2= g(0.03)2
Solution ,s F(rrtxrrf = o'0465 m

Totalenersv,E=
u' * L *, uz2 8(0.03)2
29v ,s = 7p rx""f = o'147 m
Joule/N x (1 N-m/Joule)
E ='17.5 Energy Equation between A and B:
E=17.5m Ea-HLz.l+HA-HLLr=
a -,
Es
v5=L* 65'6 *3 lto2 D^ _.2
29 -;-T*r^_HLet+HA us
29v*
e.81.(z) -HLz-"= Ps +ZB
)
!- = n.so 0+0+10_2(0.0465)+ HA-70(0.147)=0+O+OO
2g ^ HA = 51.563 m
o = 14,79 m/s (velocity of flow) Power output = ey HA = 0.03(9,S10)(51.563)
=15,175 watts x (1hp/7a6watts)
Power, P=QyE Power output = 2}.B4horsepower'
a.7e)l x (e810 x 2) x't7 '5 irated power of the pump)
= tf (0.05)'z(1
Pressure heads at 1 and 2:
= 9970.92watts x (Ihp/7a6 watts)
Power, P =73.37 hp , Energy Equation between A and 1:
Ea- HL+t= Er
uo2 or' * p, +21
PA +za-HLA-1
5-1 May 1994, May 2004 2g v 29v =

The pump shown draws water from reservoir A at elevation 10 m and lifts it
reservoir B at elevation 60 m. The loss of head from A to L is twb times 0+0+ L0-2(0.0465)=0.046S * Pt +0
velocity head in the 200 mm diameter pipe and the loss of head from 2 to ll v
o.
ten times the velocity head in the 150 mm diameter pipe. Determine the r -:- = 9.86 m of water
hcirsepower of the pump and the pressure heads at 'I and 2 in meters when
discharge is 0.03 m3/sec. Energy Equation between of 2 and, B:
Ez-HLz-e=Ee
0r2 _,2
*"-HL2-B= uB
Do
Pa
E. i 29v +zB

0.'1.4V + Pz +0-1,4(0.747)=0+0+60
v
Pz
= 61.;323 m of water
v
r-

?,60 Fl##:i#,:or F,uid F,ow


FLUID MECHANICS
& HYDRAULICS
FLUID MECHANICS
& HYDRAUL'CS CHAPTEfi
Fundamentar, orrr.rialFiovui Zb I
Problem 5 - L7 (CE November 1986)
HL2 = )'lge.lj pz
A pipeline with a pump leads to a nozzle as shown. Find the flow rate when
pump develops an 80 ft (24.4 m) head. Assume head lost in the 6-inch (152 uB2 Be2
mm) pipe to be five times its velocity head while the head lost in the 4-inch
(102 mm) pipe to be twelve time its velocity head. (a) Compute the flow rate,
,s - }'rc.*F = 24so'8 Q')

(b) sketch the energy grade line and hydraulic grade line, and (c) find the 0 * 0 + 2t.3 - 723.96 e, + 24.4 _ g"160.12e2 = 2450.8 e2 + 0 +
Pressure head at the suction side 72,384.89 Q2 = 2I.g 24.4
Er 80,(24.4 m)
Q = 0.0415 m3/s ) Discharge
B
3"jet (b) Energy and Hydraulic grade
(76.2 mm) lines:
E
E a7t 8(0.0415)2
El 70' (21.3 m)
N
o ,s = ,(rs1x01st. =0.266m
uz2 _ 8(0.0415)2
,s -;irflxolorf =131 m
\

6" (r52 mm) uB, _ 8(0.0415)2


- El 30'(15.2 m) ,s - }e.Blra;;,r =4'22m
HLt = 773.96 Q2 = 1.33 m
Solution HLz= 9,1,60.13 Q2 = 15.78 m
(a) Discharge

Qt= Qz= Qa = Q'


Energy Equation between A and B:
Ea - HLt + HA - HLz= Ea
t
'!- * pA +zA-HL1 +HA-HL2= ,,u' Ps +ZB
2gr 29t
HA = ?4.4 rn

HL, = 5 "t2 =5 =sQt


29 nr gDrn

8Q2
=-
n21v.sr;1o.rsz;a
HL1= //]')$ Qz

HL,=n+=-#
- El 30'(1S.2 m)

8Q2 El. 19.97 m


="12 El. 19.704 m
n21e.ar;1o.roz;a
CHAPTER FIVE FTUID MECHANICS FLUID MECHANICS
CHAPTER FIVE
ZOZ Fundamentals of Fluid Flow & HYDRAULICS & HYDRAUTICS Fundamentals of Fluid Flow 263
(c) Pressure head at S Energy Equation between L and 2
Energy Equation between A and S (neglecting head lost and taking point 2 as
datum)
Et- HLr: Es
Et-HE=Ez
ttA')1 +PA +ze_HL.t=,.t- * Ps +zs u.2 Pr
2gr2g v
2g v
+21 -HE=or'*
2g
Pz I z^
-+ Y
as' u1-
= = 0.266 m 14
8(0.5)2 8(o.s)2 4
29 29
7(rEnorf * n^s, +2's- nr= 7ffi+ * # *o
o+o+ 27.3-1,.g3=0.266+ lL +ts.2 HE = 3.647 m
v

Ps Power, p=eyHE
= 4.504 m
v = 0.5(9810)(3.647) = 1Z88S.S wans x (irhp/746 watts)
Power, P = 23.98 horsepower
Or from the figure shown above, the pressure head at S is the vertical distanctr
from the pipe to the HGL.
lL =D.7oa-:rs.z May
v 'f 20-h-n suction p*mp operating at 70% efhciency draws water from
a suction
' line whose diameter is 200 mm-and discharges into
lt air through a line whose
= 4.gg4^ diameter is 150 mm. The verocity in the 150 ti* trnu
is 3.6 m/s. If the pressure
v at point A in the suction pipe is 34 kpa below the
aknosphere, *nu* a is 1.g m
below B on the 150 mm line, determine the maximum,erevation
above B to
which water can be raised assuming a head loss of
Problem 5 - 18 (CE November 1980)
3 * d;;;d;;;..
*Q
Water enters a motor through a 600-mm-diameter pipe under a pressure of 14
kPa. It leaves through a 90O-mm-diameter exhaust pipe with a pressure of 4 az=3.6m/s=o6
kPa. A vertical distance of 2.5 m separates the centers of the two pipes at thr' -.2
uC
sections where the pressures are measured. If 500 liters of water pass the = 0.65 m
2g
motor each second, compute the power supplied to the motor'
Qr= no(0.15)'?(3.6)
Solution Qz= 0.0636m3/s
14 kPa Q: Qr: Qr= 0.0636 m3ls
0.0636
UA=01=
1p.42
oa = 0t = 2.025 rn/ s
u^2
f
zg
=0.21 m

Qr: Q, = 0.5 m3/s 900mmz O Poweroup,s = Qy HA


CHAPTER FIVE FLUID MECHANICS FLUID MECHANICS
^, ^
ZO+ Fundamentars of Fruid Frow CHAPTER FIVE
& HYDRAULICS & HYDRAULICS Fundamentals of Fluid Flow 265
20 x746 = 0.0536(9810)HA Energy equation betweenA and M
HA = 23.91, m Ea - HLn-u - HLu - HLua
-HL,v = Erv
PA
Energy equation bqtween A and C (datum at A):
Ea+HA-HL=Ec
29 y +za-z-2--j.o-o1.aN
+. 4^ - "-E ='au2
,g
*pN
v
+zn

o!-,) *pA +zA+HA-Ht='l +pc +zc * Pn *za-1s=rr.gn,!' + !!- *7*


429y'ozgy
2gy2gy
0.21 + !!
8Q2 sso +o-15=104e#ffiorrr
h=
+ 0 + 23.91, -z = 0.6G+ 0 + (1.8 + h)
9.81
15.19 m
;,di*r. # +o+16
Q = 0.0'1.067q5 m3ls

Problem 5 - 20 (CE November 1978) uN- O o.o1o674s


AN
- f (o.o2s)2
A fire pump delivers water through a 300-mm-diameter main to a hydrant to
aN = 2'1,.74m/ s
which is connected a cotton rubber-lined fire hose tr 00 mm in diameter
terminating to a 2S-mm-diameter nozzle. The nozzle is 2.5 m above the
hydrant and 16 m above the pump. Assuming frictional losses of 3 m from the ,, _ ,N2 21.742
pump to the hydrant, 2 m in the hydrant, 10 m from the hydrant to the base of 29 2(e.81)
the nozzle, and the loss in the nozzle of 4% of the velocity head in the jef to Ir=24.\02m
what vertical height can the jet be thrown if the gage pressure right after thc
pump is 550 kPa?

Solution 25 mmo
$e pipe shown in the Figure z1 = uz = 'J,.2
m/s. Determine the total head
between l and 2.

150 mm @ fire
2,5 m

Qa=Qe=Q,u=Q

300 mm A maln pipe

4.3 m

J----.-D.atvm
^, ,
ZOO
CHAPTER FIVE
Fundamentals of Fluid Flow
FLUID MECHANICS FLUID MECHANICS
CHAPTER FIVE
& HYDRAULICS & HYDRAULICS Fundamentals of Fluid Flow 267
Solution Oil of sp. gr. 0.84 is
ut=o2=1.2m/s flowing in a pipe
Energy equation between L and2:
,Under the Et. 3.21 m
conditions shown in
Er-HL=Ez
a the Figure. If the
,,2
*P, *..-HL=u2 +!z
"2gv29 + Z2 total head loss from
v point 1 to point 2 is
)1 900 mm, find the
Sinceor =o., )ot =u2-
)o ,pressure at point 2.
-6 -d
El. 1.2 m
P"t +21 6olution 225mma
-HL:P? +zz
vv Qt= Qr= 0.056 m3ls
280 +
4.g - HL= ?oo + q.os
9.81 9.81. Energy equation between O and
HL = 3.375 m Et - HLtz= Ez
I
Pl or'
zg y +,l-HLtz=
+. '- 29 * Pz *z^
v
Problem 5 - 22 (CE November 2O00)
A nozzle inclined at an angle of 60" with the horizontal issues a 50-mur 8(0.0s6)2 44s q(o.os6)2.
diameter water jet at the rate of 10 m/s. Neglecting air resistance, what is tht' tr'g(0.1.5)a 9.81x o.gZ "'" _o.eo=n2 + p
g(0.225)a 9.81x 0.84
area of the jet at the highest point of the projectile?
D
Solution = 55.52 m of oil
,"-_ *,
Solving for the velocity of the jet at the summit (highe-st point, A) p = 457.53kpa
av=0
?, = tlo- cos 0
t, = 10 cos 60" = 5 m/s 5-
50-mm diameter siphon discharges
, o' (sp. gr. = 0.g2) from
a reservoir (elev.
*ao1/
,*l*:L"::;(:tA f Lin:
summit (point 2, elev.22m)
hdjl;; r,om ae,e,"*"i,
rp",,., rl .
is 1.5 m and fro; th;;;;;;"";:"ffi;i;:
7)= =5m/s is 2'4 m' Determine the flow rate in the plpe in lit/sec and the
absolute
, ri"u" to:ioio..
Since the flow is continuous:
lQo = Q^l
Aoao = Aa u

f (0.05)'z (10) = AA (5)

Aa = Q.QQJ)I'/ pz

Problem 5 - 23
CHAPTERFIVE FLUID MECHANICS FLUID MECHANICS
-, ^
ZOU Fundamentals of Fluid Flow CHAPTER FIVE
& HYDRAULICS & HYDRAULICS Fundamentals of Fluid Flow 269
Solution Problem 5 - 25
El.22 m Determine the velocitv and
discharge through the tiO mm Et.30 m
diameter pipe shown (a) assuming
h;ad loss and (b) considering i
1o
Qz= Qz= Q head lost of 200 mm.
HLr-z = 1.5 m El. 28 m
HL2. = 2.4 El. 27.5 m

Solution
(a) Assuming no head loss:

Energy equation between 1 and 3: Energy equation between


Et - HLp- HLzt = Ez O and G) neglecting head lost:
,) p7 p3
Er=Ez
!-)o * v + 21 - HLt-z- HLz-t - lt)o + v + z3 _
at- r.)

_Pr +r.=
)
az- _Pz +r^
-6dl
29v' z8y
o+o+ 20-7.5-2.4= t-Ql-., +o+15 -2
'
n'g(0.05)* 0+0+ 30= 02
+0+24.9
2g
Q = 0.00912 nr'/s _-2
Q = 9.12 IiVsec =5.r.^
"2g
Energy equation between 1 and 2: oz=10 m/s
Et -HLtz= Ez
Q= Azoz= i (0.15)r(10)
,r,2 -:--.1-
,) t ,r^2
L +'' Dt +-.'-
+ +.-r
2gv2gv -HLt.-
-L Q=0.1n m3ls = 777es

(b) Considering head loss of 0.2 m:


0+0+20-1.5= 8f',0.00912)2
-' ' + "Lto +22 Et-HL=Ez
n2g(0.05)a 9.81x 0.82
p2= -37 kPa . Pt +zt-HL=
tzgy29f' a22
+ p, *,^
Absolute pressure at O : 1,01,.3 + (-3i) T^
Absolute pressure at € = 64.3 kPa 0+0+ g0-0.2= +o+24'9

-L-
a.2
= 4.9 m
"
-d
vz= 9.805 m/ s
Q = Az a, = t(0.15)r(9.805)
Q = 0.173 yp/s = 173IJs
CHAPTER FIVE FLUID MECHANICS FLUID MECHANICS
ZI U Fundamentals of Flurd Flow & HYDRAULICS CHAPTER FIVE
& HYDRAULICS
Fundarnentals of Fluid Flow 27t
Problem 5 - 26 Problem S - 27
Water flows freely tronr the reservotr shown through a 50-mnr diameter prpe Neglecting head loss, determine
at the rate of 6.31 tit/sec lf the head lost in the system is 11.58 Joule/N, the manometer reading in the
determine the elevation of the water surface in the reservoir if the discharge system shown when the velocity
errd is at elevation 4 nr of water flowing in the 75_mm 25mm@
cuameter pipe is 0.6 m/ s.

E
o
rr
.i
I

I
75mma I

*-
I
750

at= 0.6m/s
[Qr= Q21 25mma
I P.075),(0.6) = f, (0.02s)2a2
uz= 5.4m/s
Solution
E
Energy equation between 0 and o
Q = 6.31 L/s = 0.00631 rnr/s r €: !f,
L1, -F
- E2 75mme oi
HL = 1-|'.58 N-m/N = 11.58 nr
_-2
a1
-:-T-tzt h 71n"
= - + Pz +22
Energy equation between O and @ zgl2g v 75mmA
I
Er-HL=Ez
0.62 * pt - 5.42
ut-)) * Pt * :,.H1 "2 (r31) ? *o= +o+2.4
-
Jr*,'
2gv2g
= * l)t
4,zt frt 7S0 mm

8(0.00631)2
l! = S.SOS m of water I

()+0*zr - ll58 v
r-g(0 05)"
zr = 16.11 m) Elevation of w s rn tlre tank Sumriring-uppressure head from O
to g in meters of water:
P' *o,Ts-hft35\= la
yy
3.868+0.75-1,g6h=0
ft=0.3395m=339.5mm
CHAPTER FIVE' FLUID MECHANICS FIUID MECHANICS
Zf Z Fundamentals of Ftuid Ftow & HYDRAULICS & HYDRAULICS
CHAPTER FIVE
Fundamentals of Ftuid Ftow Z l3
Problem 5 - 28
A lrorizontal prpe gradually reduces from 300 rnrn diameter sectiorr to 100 mnr Energy equation between 1 & 3
clianreter section. The pressure at the 300 mnr section is 100 kPa and at the 100 (Neglectinghead loss & datum
mm section is 70 kPa lf the flow rate is 15 liters/sec of water, compute the along point 3)
head lost between the two sections
Er=Eg
Solution +zt='t-+P3-ar^
{.Pl
300 mm
29 T ' zg y'"
Q = 0.015 mr/s
o+o+rc=ry:+o+o
2g
u3=14m/s ,

or = 1oo kPa
Q= Q'= 225mm b
t pi.22s)2(14)
er = ez = 0.015 *i/s, Q = 0.557 m3ls

Energy equatlon between O and €) Pressure at the throat:


Er-[-IL=Ez
t Energy equation between O and O:
l'1 ,Pt ,,n2
u = --3:- D.
+ !-:- + '1 _.2
-1 "L
2gy28y {-P2 +22='t2 +P3 +,.
zgy2gy"
8(0.015)2
+ l oo
+0-H/ 8(0.01s)
2
+ 70 +()
= s!o.ss7)2
--
n'g(0.3)' q 81
n2g(0 1)a c.81 *p2 *r=++o+o
HI
- n2 g(0.15)a y
= 2.872 m
pz= -398.75kPa

Problem 5 - 29
A divergrng tube discharges water from a reservoir at a depth of l0 nr belon
the water surface. The diameter of the lube gradually increases from 150 nrrrt
at the throat to 225 r'r'rrn at the outlet Neglecting friction, determine: (a) the
maximunr possible rate of discharge through this tube, and (b) the
corresponding pressure at the throat
-, I +,
Z
CHAPTER FIVE
Fundamentats of Fluid Ftow
FLUID MECHANICS FLUID MECHANICS
CHAPTER FIVE
& HYDRAULICS & HYDRAULICS Fundamentals of Fluid Flow 27s
Problem 5 - 33
S upplementary Problems If the water level in Problem 5 - 32 varies
and. az = 1.0 m/s, find the rate of
change dh/ dt.
Problem 5 - 30
Air is moving through a square 0.50-m by 0.50-m duct at 180 m3/nrin. What is Ans: -9 mrn/s
the mean velocity of the air?
Arts: 12 m/s Problem S - 34
yi$r"i:::l:jo; rl_o;yff::,,}; arrangement shown in Figure
Problem 5 - 31 :ylindricar 5

The piston of a hypodernlc apparatus shown in Figure 5 - 8 is beurli


;]3"*iJi?lf;x'"il',)f*,I1"-:r**-il;"f;;#J;$ll,tril::;
*t;::lt B. Assume flow,.d_etermine the ulrurug" lr.ro.t
:::"dy radial flow A rty at section
tection and at
withdrawn at 6 mm/sec; air leaks around the piston at 20 n-rm3/sec. What ir at B.
the average speed of blood flow in the needle? Ans: at = 7.47 mf s; oz = 2.46 cm/
s
Anl 498 rnm/st.r
80mmU

Figure 5 - 8

Figure 5 - 10

Problem 5 - 32
5-35
The water tank in Figure 5 - 9 rs being filled through section 1 at 6 m/s arul
a jet is inclined upward. 30o from
through section 3 at 15 L/s. If water level h is constant, determine the t,xil the horizontal, what must be its
tch over a 3-m walt at a horizont"t velocity to
yelocity o2. diri;;;;; rs *, ii"l.",
Ans:7.97 ^ugi"J"g
15 L/s
r Ans:L6.93 m/s
t-]
5-36
chng air resistance, determine the
height a vertical jet of water will rise if
ted with velocity of 21, m/ g?

Ans:22.5 m

Figure 5 - 9

l.-O.S m a-t
CHAPTER FIVE FLUID MECHANICS FLUID MECHANICS
ZI 6 Fundamentats of Ftuid Ftow & HYDRAULICS & HYbRAULICS CHAPTER SIX
Fluid Flow Measurement 277
Problem 5 - 37
High velocity water flows up an inclined plane, as shown in Figure 5 - 1.1
Wlrat are the two possible deptlr of flow at section 2? Neglect al losses.
Ans:0.775m&2.74nr
Chapter 6
llgid Ftow Measurement
There are numerous number of devices
used to measure the flow of fluids. In
any of these devices, the.,Bernoulti,, Errergy
Figure 5 - 11
Tlteoremis greatly utilized and
knowredge of the characterisrici'and
,111L:""1 coefficients of each device is
lmportant. In the absence of reriabre varues
and coefficients, a device shourd
be calibrated for the expected opuruti^g
9.806 m/s
"or,iirio*.

DEVICE COEFFICIENTS

of Discharge, C or Ca
coefficient of discharge is the ratio
of the actual discharge through
j: ail, ;;s;;;,.h wo uld occ ur witho u t lo the
J
Xmay be
n"^'_1"_1^ 11,tq"oiuuc
expressed as:
s s es

corCa= _e"ggd.:lgg9_ = a
Th"*"tt"rldtr.h"tg" = A, Eq'6-1

actual discharge may be accomplished


by series of observation, usuaty by
::rt"1*",1r^.1^amountof
fluii passinj,nro"gll
-bu the device for a known
' The theoreticar value can u.;*;iisL"d using the Bernoulli,s
'm neglecting losses.

of Velocity, C"
coefficient of velocity is the ratio of the
actual mean velocity to the ideal
retical velocity which would occur or
withoui u.,y torrur.

(- Actualvelocity
_ =_=__:_ a
fneor"ti*t"J*it11 = o, Eq.6-2
A-A CHAPTERSIX FTUID MECHANICS FLUID MECHANICS
Z l6 Fluid Flow Measurement & HYDRAULICS 6. HYDRAULICS CHAPTER SIX
Fluid Flow Measurernent 279
Coefficient of Contraction, C. rable 6 - l: Discharse c"fl::::: v:Tf4 Sharp_Edsed Circutar orifice
The coefficient of contraction is the ratio of the actual area of the contracted Discharging inro lqr
Air at rS.O.C titi.,rl
section of the stream or jet to the area of the opening through which the fluid
flows.

_ Areaof streamorjet _
-'' -. a
- A Eq. 6- 3
Aluuof op""itrg

Relationship between the Three Coefficients

Actualdischarge, Q= C"Q' ) Eq. (1)


Also
Q = Actual atea, A x Actual velocity, c'

Q= C,A x C,,u1
Q = C, C,, Aut
but Aor = Qr
Q: C,C,QI ) Eq. (2)
From Equations (1) and (2)

C- C"x C., Eq.6 - 4 tosT


head lost through Venturi
meters, orifices, tubes, and
as: nozzles may be
The coefficient of discharge varies with Reynolds Nurnber. It is not constarrl
for a given device. Table 6 - 1 gives the coefficients for verhcal sharp edged
orifice.
Flgure 6 - 2

The-ideal energy equation between


E
1 and 2 is:
Ll-E2-F
/ -aa at2 , Pt
T,Pz
7t"2
4'8 - -f
':::::'7,,,,t
Q-1'0 a! 1.0 ;:- +21 _ +22
c; -'0.86
,
c,- 0.qt 2g v
0.86 -.::.:jt .! cd=
Co = 0,91
Ath = Azaz
ti

Square shoulder Thick plate


7,t= 12 a, und.
a12
= (!.\' 'r'
Figure 6 - 1: Orifice coefficients A7 29 {.A' j 29
FLUID MECHANICS
2BO FllffL:l',trasurement & HYDRAULICS
FLUID MECHANICS
& HYDRAULICS
Fruid Frow,fiH:I"'*:fi zB l
(4t\'t'z? +Pr +zt=oz2 *Pz +zz
[Arl 29 y 2s y nr= Y'
2g
f't*J'1 + *[, t*]']
[' [f)'] + (+.,,). (?.,,)
=
* (# 'Jl't*l'l'
If the orifice or nozzle takes off directly
from
o" then the velocity or lpfrou.ha tank where A1 is very much
is negligible and Eq. 6 _ 5
Considering head lost bdtween 1 and2: ff111; ;|1"
))
u7' + I'1 + z. _ HL= ,r-, * P2 + z,
29vzgv ,r= (J:-tl') 4 --
rt Eq.6-6
This equation sirnplifies to:
lc,t -
Note: zr = acfual velocity
(,rt)".,'o,
1- (Az / Ar)'

ffi
Since ?u.su"1 = C0 z/theorerical = z,

u= Co
1- (Az / A,,)2

Squaring both sides and arranging terms: ,

*l' [#)'] +=(t.,,) (?.',) 'cling to shape, orifice


*uI circular, square, or rectangular
From (b) n' The circular sharo-crested?"orifice
i, mo'ri*raely
ln cross_
icity of its design anj construction. used because of the

figure berow shows a general case


of fluid flow through an orifice.
I'J:,T jLT:'
normat 1".:.
of the stream ::-'_':.
;;;ill Let pa
a,,", p". uvery and aa be the
to the ptane
r two points 1 and 2 such ihat ul
J,h";;*;ffiJ#:i:#::.T;
= ,^ ur,J = o and writing the
quation between thes,
equation rhocoe r.^,^ -, r
-^:-r^ neglecting
two points losses
FLUID MECHANICS
282 F'Tf,tJ':ltil.asurement & HYDRAULICS
FLUID MECHANICS
& HYDRAULICS _ CHAPTER S'X
Fluid Flow Measurement 283
Qo, Q o' where H is the total head,prod-ucing
flouin meters,or
feet of the flowing fluid.
;Xli""::j}"Lt"i"tl$:1" t"ii";"- Ro- enersv upst uu* ress the
It
"iL"
-

Values of H for Various


Conditions

Chamber A Chamber B

a
.l
Energy equation between and 2 neglechng head lost A
Et=Et I
I
u"r2 * Pt +zt= u22 + rz +z? I
2gy29y
uA2 *Pa+ylt *o=!:*P, *0
29y28Y
H=h(lta/g)
uA2 * P^ *r,- t'2 * PB
29y28y

'2 =lr* P^ - PB *
uo2
29yyZg

rl"-t-[+ +)]
Theoretical velocity, ,, = ,pgn Eq.6 7

Actual velocity, a= e, rFgH Eq. 6-i H=h+p/1,


H=hz+hr(ylyz)+plyz
Theoretical disclrarge, Qt = n ,pn Eq.6 I
Actual discharge, Q = CA ,pn Eq.6-l0

aA2
H=ir+ * Pa - Ps
2g vv
284 Filf,'J:t',tr asuremenr FLUID MECHANICS
& HYDRAULICS
FTUID MECHANICS
& HYDRAULICS CHAPTER SIX
Fluid Flow Measurement 285
Orifices under Low Heads
when the head on a verticar orifice
is sma' in comparison with
the height or
an appreciable dlrfe'-enle u.t-u"'" tr-'" J,.t.'u.,",
ffi;t*:"tlXis using rhe

sider the rectangular secf,on of length


L ancr height D as shown in the
re with both the surface and the
Submerged Orrflce iet luu;e.i io atmospheric pressure.
(Neglecting v.) reticar discharge through an elementary'roip The
of length L and height dft is
H=hr-h:=h

dQ, = (L dh)
Jrsh
dQ, = J2S L tr/z dh
Contraction of the Jet
The figure shown represents a cross-section of fluid ilow through a vertical
sharp-edged orifice from a reservoir to the almosphere. The fluid flowing ir
Q, = J2s t 0,
!,,t
coming from all direction upstream fronr the orifice and as they leave tht,
orifice, they cannot make an abrupt change in their direction and they move irr I o ,1ht
curvilinear paths, thus causil-lg tl-re jet to contract for a short distance beyond Q, = ,l2g t
the orifice. The pherromerron is referred to as the contrnction of tlrc 1et. Thr L;rt ]^.
section on the jet where the contraction ceases is called the oatn contraclt Q, = t JzS t thz3/2 - h,t3/2 )
which is approximately located at one half of the orifice dianreter (D/2) front
Q=CQ,
the upstreanr face

METER
meter is an instrumer,
"::d,: measuring
Fi;;;;"_
the drscharge through pipes
l'j""?,::::lT:.s;u!"
ln pipe at rhe inlet at
ii,"r,i.r, is connected io the
1, Q:" ii a cylindrical
section ,. .jil":
cp whiih is colnected again to the main ;l:
{,an{3nding
oat' and a diverging section
the outlet D The ancre of divergence pipe
ir t"pt sma' to reduce the head rost
tne by turbulence as tte velocity is reduced
CHAPTERSIX FLUID MECHANIC5
^A,
26O FLUID MECHANICS
Fluid Ftow Measurement & HYDRAULICS & HYDRAULICS CHAPTER SIX
Fluid Flow Measurement 287
T_-
vl/2s
The theoretical or ideal discharge ,,er,, canbe found oric€ o; ,rm
f- I
-f- I
:"::,2:,H:',:;z;^;',f;;;f
":,1:ilr,::F,,,y,ryj;;;:il
",
,"ff^:::J,ll, ,,ur,," uu

I
Note: If we neglect the heart lnqi in
9tlv
+\ theoretical ir ;;:;;;;;Z;i17i"iiill ^,,.
^^^_^..
tost, we set the actuat ualues
(actuat !:['illi'llr"l,ijji.l;li:::,fiJ: '.:::,f"IffiT"j;
""t;;;;.ffi;ilat discharqe).

NozztE
A nozzle is a converging tube
instated at trre end of a pipe
purpose of increasing the
velocity of tn" i.rrirrg;"t
or hose for the

Piezomet€r ring Z2 0r<0t

t
Figure 6 - 3: Venturi meter

Consider two points in the system, 0 at the base of the in_tet and O at the
throat, and writing the energy equation between these two points neglecting clischarge through a nozzlecan
head lost: be calculated using the
equation
ur2 * P't *
2., =
,'r' * 0o
!_2 + 7,
'2g ,y 2g Y

u22 (1"*,.\
,s -|.t*",l \ v -)
zs_r1',=(pr_.\ where:
H= total head at base of nozzle
A, = area at the nozzle tip
Ihe left side of the equation is the krnetic energy which shows an increase in value
whjle the left side of the equation is tlre potential energy which shows a decrease ln
value. Therefore, neglectilrg head lost, the increase in kinefic mergy is equal to tl* followi'g table gives the mean
values of coefficients for
dea'ease in potentinl alerry. This statement is known as the Ventui Princtple. ,tgh a nozzle having a base water crischargr'g
diameter.i 40; and C. = 1.6.
The difference in pressure between the illet and the throat is commonly measu
by means of a differential manometer connecting the inlet and throat.

lf the elevations and the difference in pressure betweeu O and O are known, I

discharge can be solved


head lost through a nozzleis
given by Eq. 6 _ 5.
FLUID MECHANICS
2BB FilffL:l',fi.asurement & HYDRAULICS
FLUID MECHANICS
& HYDRAULICS CHAPTER SIX
Fluid Flow Measurement 289
PITOT TUBE This equahon shows that the velocity
head at point .l is transformed
Named after the French physr.cist and engineer Henri Pitot, Pitot tube is a bent pressure head at point
2
into
(L-shaped or U-shaped) tube with both ends open and is used to measure the'
velocity of fluid flow or velocity of air flow as used in airplane speedometer .

(b) (c) /d\


When the tube is placed in a moving stream with open end oriented into tht'
direction of flow, the liquid enters the opening at point 2 until the surface in
the tube rises a distance of /r above the stream surface. An equilibriunr
condition is then established, and the quantity of liquid in the tube remaitrs
unchanged as the flow remains steady. Point 2 at the face of the tube facing
tlre stream is called the stngmtion point I
P/:v

h2
l
I
hr

J_
Figure 6 - 5
v:=0

Figure 6 - 4

Consider a particle at point 1 to moving with a velocity of u As the partich'


approaches point 2, its velocity is gradually retarded to 0 at point 2. Writing
tlre energy equation between 1 and 2 neglecting friction:

Et=Ez
ut2
*L*r(= u
"2' Pz *?1
2gv' 2g Y

u1 =o; Pl -n1
-t. ,
Pz
=hz
- Y

u2 *1r",=lr,
2g
u'1=2s(7) Figure 6 - 6: pitot tube rn a ptpe

o = ,fzgn Eq.6-16
29o F,TfJ,:l'#"asurement
FLUID MECHANICS
& HYDRAUTICS
FLUID MECHANICS
& HYDRAUL'CS
CHAPTER SIX
Fluid Flow Measurement 29t
GATES
A gate is an opening lr1 a dam or other hydraulic structure to control the Actual o = C,, rl 2S(d, _ d2) + a.,2 Eq.6-18
passage of water. has the same hydraulic properties as the orifice. In using
It
gates, calibration test are advisable if accurate tneasurements are to be Actual Q = CA rl zS@., _ d,r)+ u.,2 Eq.6 - 19
obtained since its coefficient of discharge varies widely
Coefficient of contractio n, C, = lZ Eq.6-20
The following illustrations show the two different flow conditions through the
sluice gate
where:
C = C, C,, (varies from 0.61 to 0.91)
A=bv
ll = width of the flunre

TUBES

6tandard Short Tube


A standard short rube is.the one with a square-cornered
e.trance and has a
aiu*"tll u,., hown in Fis;;" ;' "Fis",: ;
Figure 6 - 7 (a): Free Flow Figure 6 - 7 (b): Submerged Flow
f -Xll ;*:."ii jm ::
8 (a):jshows a condition when
.,111.:1i
the flow started suddenry
-id., i,igi i";r"Ji;
. Figure 6 - 7: Flow through a gate j"":Til_ ol.'h" pllu
-Figurerhis condruon is-,,ery rnuch
:::r:
e same as that i:.^,:I:l_,T_ 1"5orifice.
of a sharp-crested 6 - g (b)shows a condition
ln Figure 6 - 7 (a), wrihing the errergy equation betweetr 7 and 2 neglecting the rube. Th? dischargl ,nro,rgh *,]s
t one-third greater than that of the standu.d ,hu.p'-"Jg"J3rtr,." t,rUe i,
|;:|"^Y:"1hes.rhe..wails.of
head lost:
ity of flow is lesser ,h" i",
h= Et
,.2 01 u"2 Do
rar t Ll - I !-2 a7,
29v2g -
Y

where Pt = dt und, P2 = d, C. = 1.0


vy C=C"=0.82
ul2 *d,+o= _.2
u2 *dr*0
2g 2g Dr (
-4-\--lLl--"-
L\-_.2_!__- .t _t:-_=-:
ur' -!,f- =dt_dt
29 29
ttzz-ut2==29@t-dz)
uzz: 2g (dt - dz1 + o'tz

Figure6-8(a) Figure6-8(b)
- d2\+ a12
Figure 6 - 8: Standard Short tubes
-A^ Z CHAPTERSIX FLUID MECHANICS FLUID MECHANICS
Ztf Fluid Flow Measuremenr & HYDRAULICS & HYDRAUL'CS
F,uidF,ow,fH:15*:# z9J
Converging Tubes Re-entrant Tubes
Clorricral converging tubes has the form of a These are tube having
their ends projecting
l:rustunr of a right circular cone with the inside a reservoir or
tank.
larger end adjacent to the tarrk or reservoir
as slrown in Figure 6 - 9

::#{;##'!0.,:;';Jn: f a special case or.a re-enkan*ube,


C,,pgH
u=

Q = C A,FgH
Eq.6 -

Eq.6 -22
21
** :: j,n" fi
T,T :;:fi:H', :1,:" il::i # n?.1,H,*?

Tubes
Table 6 - 2: Coefflcients for Conical Converging Tubes ofsubmerged tube is
-:1,u*pl: The discharge through
a conyeying water through
Coefficient
Angle of Convergence, o ]ill""o ^.:l::1,
a submerged tube is
given by tl:;
0o 5o 100 15" 200 25" 300 400 500
cu 0.829 0.911 0.947 0.965 0.971 0.973 0-976 0.981 0.984
a 1.000 0.999 0.992 0.972 0.952 0.935 0.918 O.BBB 0.859
c 0.829 0.910 0.939 0.938 0.924 0.911 0.896 0.871 0.845
cre C is the coefficienf
of discharge, A is the
difference in elevarion area of the opening,
of the tiquilJ;;A;: and H is
Diverging Tubes
A diverging tube has the form of a frustum of a right circular cone with thl
smaller end adjacent to the reservoir or tank.

i1
--.------F0 =T- mr-) e
';;*-/
;
'-
Figure 6 - lO: Submerged
Tube (Culvert)
29 4 FiltrJ,:l',ilasurement
FLUID MECHANICS
& HYDRAULICS
FTUID MECHANICS
& HYDRAUL'CS CHAPTER
!
FtuidFtowt"rrrr.-J# 295
UNSTEADY FLOW
Asdh
'l'he flow through orifice, weirs, or tubes is said to be steady or-rly if
the total dt=
head producing flow, H, is constant. The amount of fluid being discharged for
0--4"; -
a time I can therefore be computed using the formula

VoI=Qf Eq.6-24

where Q is the discharge, which is constant or steady. ln some conditions,


When there is no inflow (ei,, 0),
however, the head over an orifice, tube or weir may vary as the fluid flows oul = the formula becomes:
and thus causing the flow to be unsteady
t= ft" A,dr,
J r, - Qou,

nging the limits to change the


sign of the integrand:

Note: If A, is variable, it must be


expressed in terms of fi.

Figure 6 - 11
e outflow is through and orifices
or tube, eout = CA
ugh any other openings, use the On. If the flow is
corresponding formula for discharge.
tanks with constant cross_
ional area and the outflow is
Consider the tank shown in the figure to be supplied with a fluid (inflow) and rugh an orifice or tube (with
simultaneously discharging through an outlet (either an orifice, tube, weir or inflow), the time for the head
pipe). Obviously, if Qin > Qout, the head will rise and if Qout > Qin, the head will change from Hr to H2 is:
fall. Suppose we are required to compute the time to lower the level from /rr to
ftz (assuming Qou, > Qin), the amount of fluid which is lost in the tank will be
r= lH' e,au
dV = (Qt^ - Q""r) r// Ju, CA",l2fi
dV
at=
Qin -Q.u,
where dV is the differenhal volume Iost over a differential timedt. If the hcnd
'=#
n'
Ii:"",
Al,tH
over the outlet is /2, then the level will drop dh, thus dV : A" dh, where A" is thr t= lru* l"'
surface area in the reservoir at any instant and may be cbnstant or varialrl€, cA"J2g L-" _ln,
then
- ,
ZtlO
CHAPTER SIX
Fluid Flow Measurement
FLUID MECHANICS FLUID MECHANICS
& HYDRAULICS & HYDRAUTICS CHAPTER SIX
Fluid Flow Measurement 297
WEIR
,= -34=(,f+
L/.o.,l Zg
-W) sq.6-27 weirs are overflow shucfures which
are buirt across an open
Purpose of measurinS or coltr$ing th" flo*. channer for the
commonly used to measure the of liquids. Weirs have been
florar"of wate
tf liquid flows through a submerged orifice or tube connecting two tanks as
measure ihe now of other riquids.
discussed on this chapter u.u
gur,"rut, i.".
il;#l|J'jfffiLi",rr;:iffii;i
shown, the time for the head to change from Hr to Hz is: (See the derivation of
these formulas in PROBLEM 6 - 36)
"pffi."frf" to any type of liquid.
Classification of Weirs

ti;z:i::z::i*i";.il"?;fi?L?:#:'K:,T^:::1ryn,*apezoidat,circurar,
the
ihe rectangutur, commonly used shapes are
J "i1: J, ;XT ;ilt".L:;
rec tan gu ar, diurr*otur
I
,---^-^,,-To,tt
an;d rho
tr i an gu la r r-r,
o ra J',
th
of the crest, weirs mai be " "p ", " ";
r,'
slnrp_crested or broaa_tesled-

flow over a weir may either be free


or subnterged. If the water surface
,h:;;:st, the now is fiee, but ir the
il$::il fff.His higher
surface il:fjj,l"::I,i.'""
than the.r"ri n" n"_'i:r;;;r;;I

of Terms
Nappe - the overflowing stream in
Tank 1 Tank 1 a weir.

""' "friiir|;"*:rflt"#top surface of a weir wirh which


the flowing
Contracted ueir - weirs having sides sharp_edged,
1.
t= --.-..--.-.: -AstA,-z g-1/2 dH Eq.6-2g contracted so that the nappe
in width or having end contr"actio;;, is
CA",,!2g fi' A4 + A,2 two ends. ;i;J.re end or
Suppressed zoeir or
full-tuidth rceir _ weirs havit
'.,f il,. rLa--^r ^^ ,, . 19 its length L being equal to
where A51 and Arz is the water surface areas in the tanks at any time, and H it the width
width'.r,r-,"" .n"""a-;",i:"rtiithe
the difference in water surfaces in the two tanks at any time. If A.r and/or A,r contractions. nappe suffers no end
will vary, it must be expressed in terms of H Drop-dorun cun)e - the downward
curvature of the riquid surface
weir. before the
[f Asr and A"2 dre constant, i.e. the two'tanks have uniform cross-sectional arcn,
Head' H - the distance between
the formula becomes: the liquid surface and the
measured be.fore the drop_dow. crest of the weir,
.rrrr"-.
Art A"z
f= Eq.6 -2e
As + Ar2 dmtrq-ftr;)
\
29a Flitffiltf.asurement FLUID MECHANICS
& HYDRAULICS
FLUID MECHANICS
6. HYDRAULICS
Fruid Frow,;H:l"t#:# 299
Ion,= JEt [t.h,)ian
e,=,fg rEA.r"trf
Q,= t JU rt(H+11,12y2-(o+11s2121

Actual Q =Ce,

,'v
t/i\r
--_<--
't
'r\-
l\.\.
a=+cJ4LL@+h,ym
___*____---,' .._l],
I

a common practice
2 t;^- _
to comhino .
a singre coefficient
in* r""",",11H111;,ffI;:: c, caned
1i,,i1gt*
;idering velocity"of through a recta.gular
Figure 6 - 12: Pathlines of flow over a rectangular sharp-crested weir upfrou.h d;;;"..arge weir

RECTANGULAR WEIR Q = C,"L l(u + tr,,)3 i2 * tr-- J


€'-" Eq.6-37
small, the verocity or
lh
jan I
I
ff#5:#{J,ji,i,Y,',0,ff"Tv
Lay be neglected.
approach
The discharge formula
o*"-;
becomes

- P Q= CruLgt
rifuations where the
,fu:1, *i,,g E; ol;r::;ilt: the verociry or
(since ;' ;:Tffi$,considerine
" - ur
velocityof aonroa.h
woulcl lead to successive
Figure 6 - 13: Section A-A of Figure 6 - 12. Q.Jsince the velocity r.
h' ^.ld
:^ is a function
kials to solve
La
"q"ui"r,;;to""tr"Ttoach "f Q). H;;;".;*,
Consider a differential area of length L and height dh to be located ft nrt
below the liquid surface. By orifice.theory, the theoretical velocity thrt
this area is: c'ruti'.o(#)'J Eq.6-33
,, = JTgIt
where the total head producing flow H' = h t llo, where ho is the velocity g*!1
C, =
of approach and is equal to tto2/28. The discharge through tlre elemt'r 22g Eq.6 -34
strip is then,
dQt= dAq = of water upstream
{ 9-uO*
d=H+p
dQ,=Latr,[zg1tr+nj
CHAPTER SIX
300 Fluid Flow Measurement
FLUID MECHANICS
& HYDRAULICS
FTUID MECHANICS
& HYDRAUTICS CHAPTER SIX
Standard Weir
tstuid Flow Measurement
30r
'l'he following specifications must be applied BAZINFORMULA
to a standard rectangular weirs For rectangular weirs
without end conkactions: of length from 0.5 m
rnm to 600 mm. to 2.0 m under heads
from 50
1. The upstream face of tl're weir plate should be vertical and smooth.
2. The crest edge shall be level, shall have a square upstream corner,
and shall be narrow that the water will not touch it agairi after
passing the upsh'eam corner.
3. The sides of the flume shall be vertical and smooth and shall extend
a short distance downstream past the weir crest.
4. The pressure under the nappe shall be atmospheric
Rectangular Weirs
5. The approach channel shall be of uniform cross section for a sufficient
effective length of
distance .above the weir, or shall be provided with baffles that a L ofa contracted weir is given by:
normal distribution of velocities exists in the flow approaching the
weir, and the water surface is free of waves or surges. L=L'-0.1NH

Standard Weir Factor (C*) Formulas


where l, = measure length of
crest
N = number or""ia-.or,t
Numerous equations have been developed for finding the discharge H = measured
Jluon
--- -. 1t or z;
\r v,
coefficient C"' to be used in Eq. 6 - 31 and Eq. 6 - 32. some of these are givcn head
below.

FRANCIS FORMULA
Based upon experiments on rectangular weirs frorn'1..07 m (3.5 ft.) to 5.1g ur
(17 ft.) long under heads from 180 mm to 490 mm.

C,,=7.841+0.26(H / 421 (S.I. Unirs) Eq.6-35


F-L
For H/P < 0.4, the following value of C,, may be used. One-end Contraction
(N=1) Two-end Contraction
(N=2)
S.I. Unit, C,, = 1.84 Eq. 6 - 36

Ij.uLAR WEIR (V_NOTCH)


the nappe of a recrangular
REHBOCK AND CHOW FORMULA .j::^Tf:,
'nsheam face. weir nas
h a tendency to adhere
A .,.,"""r1:"::: -*"t
,ccurate,esurt'o.-ui.
to

English Unit, C, = 3.22 + O.EOY. 'Eq.6-38 y of measurement "#q?;::T;:.1;n*n*;##,:


is required-
rrr" r*t"-']"gre
Detween 10o to 90o 0 0f a v-notch weir
but rarely larger. is
S.l. Unit, Co, = 1.8 + ().22!-
p.
Eq.6-39

Vous aimerez peut-être aussi